PART ONEAll about the LSAT
CHAPTER 2LSAT-diagnosetest
This diagnostic test is designed to be very similar to the real LSAT in type and number of questions, time limits, and level of difficulty. Use it to assess your test readiness as you begin preparing for the LSAT.
For best results, try to simulate exam conditions when taking this test. Complete the test in a single session, following the instructions and time limits. At the end of the test you will find an answer key for each question, as well as answers and explanations. Rate yourself using the answer key, then read through the explanations, paying special attention to questions you got wrong or found difficult to answer. Don't skip explanations for questions you answered correctly; You can learn simpler or easier ways to find the right answer.
Your score on this test reflects your current readiness to take the LSAT. Use it as a starting point for planning your preparation program. What kinds of questions did you find the easiest to ask? What were the hardest? Was there a question that you found confusing or that you couldn't answer? Instructional material for each LSAT question type is presented in the following chapters. As you read the chapters, focus on the types of questions you found most difficult. Pay close attention to the examples and solutions presented. Then hone your problem-solving skills by taking the practice tests at the back of this book. Additional LSAT exercises are available on the companion website atMHPracticePlus.com.
You can also run the diagnostic test on your tablet or smartphone, as well as on your laptop or home computer. See page 7A of the welcome booklet for more information.
answer sheet
Instructions: Before starting the test, copy or remove this answer sheet from the book. Mark your answer to each question by filling in the appropriate answer oval in the columns below. If a section has fewer questions than answer fields, leave the extra fields blank.
Section I
1.
2.
3.
4.
5.
6.
7.
8.
9.
10.
11.
12.
13.
14.
fifteen.
16.
17.
18.
19.
20.
21.
22.
23.
24.
25.
26.
27.
28.
29.
30.
Section II
1.
2.
3.
4.
5.
6.
7.
8.
9.
10.
11.
12.
13.
14.
fifteen.
16.
17.
18.
19.
20.
21.
22.
23.
24.
25.
26.
27.
28.
29.
30.
Section III
1.
2.
3.
4.
5.
6.
7.
8.
9.
10.
11.
12.
13.
14.
fifteen.
16.
17.
18.
19.
20.
21.
22.
23.
24.
25.
26.
27.
28.
29.
30.
Section IV
1.
2.
3.
4.
5.
6.
7.
8.
9.
10.
11.
12.
13.
14.
fifteen.
16.
17.
18.
19.
20.
21.
22.
23.
24.
25.
26.
27.
28.
29.
30.
SECTION I
Time – 35 minutes
26 questions
Instructions: The questions in this section are based on justifications in short statements or passages. For some questions, more than one of the options could potentially answer the question. However, you must chooseotherAnswers; that is, the answer that most accurately and completely answers the question. You must not make assumptions that are reasonably implausible, superfluous or inconsistent with the Passage. After choosing the best answer, hide the appropriate space on your answer sheet.
1🇧🇷 Since David Ellington became Chief of Police, some of our most decorated officers have resigned, recruitment has declined, the department has implemented fewer community programs, crime is on the rise, and polls show that our police force is less popular than ever. It's obvious that the City Council appointed Ellington to undermine our civic order.
The reasoning in the argument is flawed because the argument
(A) confuses quantitative results with qualitative results
(B) jumps to a general conclusion based on some anomalous cases
(C) ignores that someone can be perceived as incompetent without actually being incompetent
(D) assumes that because an action is followed by a result, the action was performed to produce that result
(E) repeats something that has already been proven
2🇧🇷 An apartment complex installs alarm systems in all apartments due to increased crime in the area. The manager justifies this by saying that he doesn't need to install an alarm system in apartment 3B because the tenant is a policeman and can protect himself.
Which of the following decisions is based on faulty reasoning that most closely resembles the reasoning of the housing manager?
(A) A courier has limited time to deliver all of his packages before returning to the office. The supplier argues that they should be delivered in order from largest to smallest, as there may not be enough time to deliver them all.
(B) A candidate for mayor must prioritize areas of the city on which to focus his campaign efforts. The candidate justifies that she is dedicating all her resources to those areas where voters are least likely to vote for her, since other voters do not need to be persuaded to vote for her.
(C) Jury members have four hours to reach their verdict, or the judge will dismiss them and declare a mistrial. The jury reasoned that since the foreman is a lawyer, they should allow the foreman to review the evidence and render a verdict.
(D) The captain of a yacht requires all of his passengers to complete safety training. He argues that one passenger did not have to attend because he was a former US Navy officer.
(E) A manager discovers that an employee has stolen groceries from the break room. He argues that he should force all of his employees to complete ethics training to ensure the culprit never steals food again.
3🇧🇷 Larry: Some laws passed by Congress are seen as bad by the citizens of our country, even though they produce positive results. A law is only good if it produces positive results. Therefore, some laws considered bad by the citizens of our nation are actually good laws.
Carrie: While I agree with your conclusion, I disagree with your reasoning behind it. Some good laws don't really produce positive results. But no laws considered bad by our country's citizens produce positive results, so your conclusion that some laws considered bad by our country's citizens are actually good still applies.
Which of the following statements correctly describes a flaw in Larry and Carrie's reasoning?
(A) Suppose that if a certain quality of a law is necessary for it to be a certain kind of law, then having that quality is sufficient for it to be that kind of law.
(B) Suppose that if a certain property is shared by two types of laws, then that property is the only property that distinguishes the two types of laws from laws of other types.
(C) Suppose that if most laws of a certain type share a certain quality, then all laws share that quality.
(D) Suppose that if a certain property is shared by laws of a certain type in a certain nation, then that property is shared by laws of that type in all nations.
(E) Suppose that if a particular property distinguishes one type of law from another, then that property is one of many properties that distinguish the two types of law.
4🇧🇷 Based on new evidence, anthropologists now believe that there was a "white revolution" during the Neolithic period, when Middle Eastern milk-drinking farmers conquered the hunter-gatherer tribes of Europe and established permanent settlements where hunter-gatherer communities collectors used to exist. 🇧🇷 Experts used to think that representatives from the Middle East just traveled around Europe and brought their ideas as evangelists and converted indigenous peoples to a new way of life. This new evidence finally proves that Europe's transition to an agricultural and dairy-consuming society was far more violent and transformative than previously thought.
Which of the following statements, if true, most weakens the argument?
(A) A discovery of several Middle Eastern villages in northern Europe with tools showing that their members ate only vegetables and fruits and never raised cows
(B) Evidence that various hunter-gatherer societies drank milk well before Middle Eastern farming communities migrated to the region
(C) the discovery of lands scarred by period battles and mass graves of Middle Eastern farmers and hunter-gatherers
(D) Evidence that villages were established by Middle Eastern people near hunter-gatherer communities and trade existed between the two types of communities
(E) the discovery of children with genetic markers from hunter-gatherers and farmers, proving that there was sexual mixing
5🇧🇷 An increase in the number of young people entering the workforce in a small country between 2000 and 2006 is correlated with an increase in the percentage of young people completing college in that country. Since young people in the small country are generally high school graduates or dropouts, the correlation suggests that the country's employability significantly depends on the number of high school graduates.
Which of the following statements, if true, most weakens the argument?
(A) The percentage of young people with at least two years of college was higher in 2006 than in 2000.
(B) Between 2000 and 2006, the proportion of early school leavers entering the labor market increased sharply.
(C) Most available jobs require the use of technology and the performance of tasks that are too complicated for early school leavers.
(D) In 2006, more young school leavers were hired in the small country than in 2000.
(E) All workers are strongly encouraged to complete at least high school before applying for a job.
6🇧🇷 Speaker: The government must recognize that modern corporations are capable of behaving as irresponsibly as they like without fear of losing power. In a modern society, Davis and Blomstrom's Iron Law of Responsibility, which stated that "those who do not use power in ways society holds responsible tend to lose it in the long run", no longer apply and it never was. History shows that, in the long run, companies are able to cover up their bad deeds with effective public relations or wait out the public's short attention span. Thus, a company that wants to act irresponsibly can do so without fear of losing power in society.
Which of the following statements, if true, most weakens the speaker's argument?
(A) Some institutions reach a critical mass where they are too big to fail and will, in a sense, live forever.
(B) Some corporations that used power in socially irresponsible ways remained powerful.
(C) The power of some institutions grows faster than the power of others, whether they are socially irresponsible or not.
(D) Government institutions are as immune to the Iron Law of Responsibility as economic institutions.
(E)Public relations are often transparent and cannot hide a company's truly irresponsible actions.
Questions 7-8
Several serious amateur photographers put the new Apheron digital camera to the test. They noted that the camera shot both high-light and low-light with much finer detail than the 10-megapixel Norwich, although 10-megapixel Norwich cameras offer higher resolution than 8-megapixel Apheron cameras. Given the observational results of these amateur photographers, every serious photographer should pick up the Apheron when shopping for a strong, low-light digital camera.
7🇧🇷 the dispute continues
(A) Describe how a test situation approximates actual conditions of normal use
(B) Use the claims of a subset of a group to make a recommendation to the larger group
(C) Evaluate the credibility of a recommendation from a specific group
(D) Distinguishing between the real reasons why a given group did a given thing and the best reasons for doing so
(E) Putting an experience into a larger context to explain it
8🇧🇷 Which of the following statements weakens the argument the most?
(A) The lens of the Apheron digital camera is made of a different material than that used in other digital cameras.
(B) In digital cameras, sensor size is more important than the number of pixels when it comes to shooting in high or low light.
(C) The quality of detail in photos is just one of several factors that together should determine whether a camera is worth buying.
(D) The amateur photographers carried out their observations and comparisons in multiple sessions and at different times of the day.
(E) Many serious amateur photographers have no intention of buying a digital camera to shoot in such extreme conditions.
9🇧🇷 Economist: The auto industry appears to be recovering from an earlier slump. Recent numbers show automakers are buying more equipment and spending more on research and development than ever before, suggesting they expect sales to pick up in the near future.
Does the fact that automakers are buying more equipment and spending more on research and development than ever before play one of the following roles in the economist's argument?
(A) It is a conclusion drawn from the premise that automakers expect sales to increase in the near future.
(B) It is a conclusion drawn from the premise that the auto industry is recovering.
(C) It is the primary evidence from which the conclusion of the argument is drawn.
(D) Its truth is necessary for the conclusion of the argument to be true.
(E) It is the phenomenon that the argument seeks to explain.
10🇧🇷 The amount of water used in Anderville on any given day during the summer is directly proportional to the heat index on that day. Since the average heat index in July was four points higher than the average heat index last August, it follows that Anderville used more water in July than it did last August.
Which of the following arguments has a pattern of reasoning most similar to the above argument?
(A) The number of physicians working in the ED in a day is directly proportional to the number of services provided by the ED that day and also directly proportional to the number of patients visiting the ED. Thus, the number of visits provided by the emergency room on a given day is directly proportional to the number of patients arriving at the emergency room on that day.
(B) The number of physicians working in the ER on a given day is directly proportional to the number of nurses working in the ER on that day. But the ER employs the same number of nurses every day. As a result, there are often more doctors than nurses working in the emergency room.
(C) The bill paid by a patient in the emergency room is directly proportional to the number of health professionals the patient sees during his consultation. As the number of patients passing through the emergency room increases, it follows that the emergency room charges a larger amount of fees paid by patients than previously.
(D) The increase in emergency room patients is directly proportional to the amount of advertising the hospital did in the previous year. The hospital is trying to attract more patients to its ER by increasing the hospital's advertising spend.
(E) The amount of analgesics prescribed in an emergency room is directly proportional to the number of patients that pass through the emergency room on a given day. Because the emergency room treated 15% more patients last year than the year before, more pain relievers were prescribed in the emergency room last year than the year before.
11🇧🇷 Jason: The solar cooker provides free energy for cooking, baking and purifying water. People using it avoid indoor smoke inhalation, fire hazards, and injuries from collecting firewood. They also have environmental benefits such as: B. reducing deforestation, reducing energy costs and reducing CO2emissions. The solar cooker is a simple solution to a variety of problems in developing countries.
Zola: While I agree with your arguments, I disagree with your conclusion. Solar cookers have all these benefits. Unfortunately, they take a long time to boil water - usually between two and three hours - and they need a lot of sunlight, which is not always available during the rainy season. In practice, they are far from a perfect solution.
Which of the following reasoning techniques does Zola use in response to Jason's argument?
(A) She argues that Jason's conclusion is just a repetition of his fallacious arguments.
(B) She argues that Jason's view of the ills of the developing world is too narrow and therefore his conclusion is inadequate.
(C) She questions the integrity of Jason's research and whether his conclusion is based on a flawed basis.
(D) It gives an alternative interpretation of the assumptions, leading to a very different conclusion.
(E) She introduces caveats that undermine several of Jason's assumptions.
12🇧🇷 Joe's Lumber only carries two types of 2×4 lumber, pine and oak. Roberto never uses pine to build his tables because he likes oak's heavier feel and believes it is more durable. When he built a dining table for Marion, he bought the wood from Joe's Lumber, so the table had to be oak.
The above argument is most similar in logical structure to which of the following?
(A) Rocky wants to go to the amusement park with three friends. His father owns a sedan and a compact car. The compact has room for four people, but even though the sedan's engine has more power, Rocky will borrow the compact if he borrows a car from his father.
(B) Roddick Corporation offers Pedzisai the option of deploying in Sweden or South Africa. Pedzisai would never accept a tour of his native South Africa if it meant turning down a tour of Europe. So if he decides to play, it has to be Sweden.
(C) If Georgia wants to ride her bike from home to work, she must take a public road or navigate a maze of neighborhoods. The streets in the neighborhood are always full of potholes and the road is very busy, so she never rides her bike to work.
(E)Antonio can take a 15-minute break in the morning or an hour and a half later for lunch. Antonio's scheduled haircut takes 30 minutes, but his barber only had morning appointments, so it doesn't matter what Antonio has to bring for the haircut another day.
(E) If the team owner doesn't fire his coach, his football team will rebel and lose more games. Losing the coach also means more games will be lost. Therefore, if there is no workaround, the team loses more games.
13🇧🇷 A building inspector is accused of ignoring serious construction defects in a building. Although records were lost and the building was demolished, his inspections of newer buildings were reviewed and found to be sound. Therefore, the claim must be rejected.
Which of the following contains questionable reasoning similar to the reasoning above?
(A) A computer scientist has been accused of spreading a computer virus to several large computers, but the charges must be dismissed as the hard disks of these computers have been wiped and there is no record of his wrongdoing. Furthermore, his work since then has not been affected by such actions.
(B) A professor was accused of allowing several students to pass a course when, in fact, they failed. This accusation must be discussed at length because, although these students were successful in later grades, the teacher may have surpassed other unworthy students.
(C) Politician A was accused of stealing parts of a speech by Politician B. This allegation is unfounded and should be ignored as Politician A was not even born when Politician B gave this speech and there is no recording of the original speech.
(D) A restaurateur is accused of using expired ingredients. The accusation seems justified, even if there is no evidence for it, because several cases of food poisoning can be traced back to the restaurateur's place of work.
(E) A financial manager is accused of stealing funds from his clients. The accusations must be ignored because, although the records show that there were wrongdoings, the identity of the accuser has not been disclosed.
14🇧🇷 Every employee who goes to and from work by public transport has lunch in the company's break room. As a result, some employees who have their own office do not use public transport to get to work.
Does the conclusion of the argument follow logically if which of the following assumptions are made?
(A) Some employees who do not have an office have lunch in the company's break room.
(B) Some employees who eat lunch in the company's break room have an office.
(C) Each employee who eats lunch in the break room has an office.
(D) Some employees who do not have an office do not have lunch in the company's break room.
(E) Any employee who eats lunch in the company's break room uses public transportation to and from work.
fifteen🇧🇷 Archaeologists have found underwater engravings near the wild city of Manaus after a drought hit the Brazilian Amazon. Previously, archaeologists studying the Amazon believed that the rainforest was too inhospitable to support a large civilization and that the only civilizations in the area were nomads. The new discovery proves that theory wrong and that the Amazon was home to great civilizations thousands of years ago.
Which of the following statements, if true, would most strongly invalidate the above argument?
(A) Remains of large established villages with well-traveled roads leading through or through them have been excavated near the site of the drawings.
(B) The engravings showed an unexpected sophistication for the period, but still showed the telltale signs of the period.
(C) Archaeologists have proven that a nomadic artist created the very similar engravings and engravings nearly 100 miles away.
(D) The drought that exposed the submerged carvings was the first in the area in over 2,000 years.
(E) The engravings included a depiction of hunters walking across a plain and people building small houses.
16🇧🇷 A new study looked at people who watch TV and their levels of happiness. The study showed that people with more channels, and therefore more choices about what to watch, are not necessarily happier than those who have fewer options about what to watch. Therefore, people in highly industrialized countries who have more options in all aspects of their daily lives are no happier than people in less developed countries who have fewer options in their daily lives.
The above argument relies on the assumption that
(A) People in less developed countries are just as likely to have as many television channels as people in highly industrialized countries
(B) The inverse relationship between happiness and number of television channels is the same for other categories of goods and services, regardless of other factors related to where people live in the world
(C) People in less developed countries are not familiar with the number and quality of options available to people in highly industrialized countries
(D) People in highly industrialized countries have visited less developed countries and found that their choices are of lower quality and less likely to make them happier
(E) The relationship between the number of TV channels and happiness is directly related to a society's stage of industrialization, and therefore an industrialized nation will feel the impact of such differences more
17🇧🇷 A high school cheerleading team held their annual bake sale on the first day of school to raise money for the upcoming year. To increase sales this year, the team decided to lower prices on everything. Despite their strategy, Cheerleaders had lower sales than any previous year.
Each of the following statements, if true, will help resolve the apparent discrepancy noted above, EXCEPT:
(A) Enrollment of high school students decreased significantly from last year to this year.
(B) Other school groups scheduled their collections for the same day, which did not happen in previous years.
(C) Due to the increase in the price of sugar and flour, the cost of manufacturing bakery products has increased this year.
(D) Teachers distributed baskets with cupcakes and other sweets on the first day of class.
(E) Due to the construction of a new auditorium, the fans were unable to set up their bake sale tables in the same known location as in previous years.
18🇧🇷 The non-profit Redville Community Center was facing a budget crunch due to rising operating costs and the costly repairs needed to keep the center running. The board decided to raise membership fees and charge a construction fee to raise the necessary funds. By the end of the year, the Board realized that costs were lower than anticipated and the fee changes left the Center with a budget surplus. The board decided to use the surplus funds to buy a new van to transport people from the care home near the center, although the old van was still operational. But before that, the board must get approval from the members who paid the additional fees.
Which of the following policies, if passed, would most likely justify the above position and still less restrict the use of funds by community centers?
(A) Beneficiaries of a charitable organization with reasonable fee-based interest should presume that they trust the directors of those organizations to use the money wisely under all circumstances.
(B) Beneficiaries of a non-profit organization with a fee-based interest commitment may not delegate to the directors of such organizations the responsibility for allocating funds received for various purposes consistent with the purposes of the organization, as the directors of the organization see the adjust.
(C) Directors of nonprofit organizations may not allocate reviews for any purpose for which the directors have not specifically earmarked funds in advance.
(D) Fees collected by a non-profit organization for a specific purpose shall be used only for that purpose or, where this is not possible, shall only be used at the express request of the person paying the fees.
(E) Directors of non-profit organizations that charge additional fees for specific purposes must return money received from such evaluations if more money is received than can practically be used for the stated purpose of the evaluation.
19🇧🇷 Scientist: My colleagues have said that my theory of molecular cohesion in glass is purely conjectural and has no experimental basis. This is not true. I based my theory on several findings recently published by reputable research organizations. Although I have not verified all the details of their experiments, I am confident in their work. Also, you may recall that five years ago I was asked to develop theories of molecular cohesion in wood fibers and I used results from the same laboratories at the time.
Which of the following criticisms is the scientist's argument LESS susceptible to?
(A) It is a conclusion about the scientific knowledge of research organizations based on uncertain memories.
(B) Experiments conducted by research organizations are assumed to be unaffected by human bias or error.
(C) Experiments carried out by research organizations are considered the only work necessary to develop a scientific theory.
(D) Hastily concludes that experiments conducted by research organizations are correct without having studied them in detail.
(E) The use of experiments conducted by research organizations in the past as a basis for the development of a scientific theory is considered to justify their use in the development of current theory.
Questions 20-21
The city conducted its annual testing for lead in drinking water and found that about 14% of samples collected exceeded federal standards for allowable levels of lead. Regardless of these results, lead levels, even in those that exceeded federal standards, were still too low to pose an immediate health risk. So it's perfectly safe for city dwellers to drink as much tap water as they want.
20🇧🇷 Which of the following is an assumption on which the argument depends?
(A) City residents often flout federal guidelines when it comes to public health issues because the standards are so strict.
(B) The most dangerous pollutant in drinking water is lead.
(C) Statistical sampling is not the most accurate basis for determining health risks, although federal and local governments should rely on it.
(D) Lead levels, even slightly above federal standards, have no long-term effects on human health.
(E) People feel safer if they heed federal government alerts, even if they are not entirely accurate.
21🇧🇷 Which of the following statements most strengthens the argument, if true?
(A) Most of the lead in municipal drinking water is absorbed by the lining of pipes.
(B) Due to budget cuts, the city had to test water coming from a central location, instead of sampling randomly across the city as in previous years.
(C) Studies show that children who ingest lead levels above federal standards experience long-term adverse effects.
(D) The local hospital is reporting an increase in cases of lead poisoning among people who regularly drink large amounts of water.
(E) Several medical associations have advocated for higher standards and tougher penalties for cities that fail to take steps to reduce lead contamination in their drinking water.
Questions 22-23
Jaime: Both African and Asian elephants live much shorter lives in zoos than they do in national parks. Studies have proven this by comparing the life expectancy of elephants in zoos with the life expectancy of elephants in wildlife sanctuaries in Kenya and Myanmar. Therefore, our government should seriously review how zoos treat their elephants and possibly introduce a temporary ban on acquiring new elephants until this problem is resolved.
Eleanor: But both of the reserves you mentioned are sanctuaries where the animals are not at risk from poaching and other unnatural dangers. Furthermore, the data from these studies go back more than 40 years and do not take into account recent advances in captive animal husbandry that will definitely extend the lifespan of elephants currently in zoos.
22🇧🇷 Which of the following is an assumption on which Eleanor's argument is based?
(A) There are genetic differences between African and Asian elephants that cause different lifespans between these particular species.
(B) Predation and disease within the boundaries of a conservation area affect an elephant's life expectancy to a lesser extent than threats faced by an elephant outside parks.
(C) The training of zookeepers and those who manage nature reserves is exactly the same; Therefore, care for elephants should be exactly the same.
(GB) A zoo keeper in 1970 was unable to prolong the lives of elephants in captivity as zoos suffered from budgetary constraints at the time.
(E) Professionals who manage nature reserves are more aware of the threat from poachers than from keepers; Therefore, they better protect their elephants from them.
23🇧🇷 Eleanor's rejection of Jaime's conclusion uses which of the following reasoning techniques?
(A) Produce a single contradictory example that establishes that a given generalization is wrong
(B) Questioning the validity of a conclusion because the supporting statements cannot be verified experimentally
(C) Noting that there is potentially more favorable evidence that has been overlooked
(D) Rejection of a problematic correlation and possible change of variance in relation to the supporting statements underlying the conclusion
(E) Reanalyze the underlying assumptions and follow a broader sample of data underlying the conclusion
24🇧🇷 A teacher decided to challenge 10 of her students to do their math homework without a calculator or computer for three months, while allowing the rest of the class to use whatever tools were available. She found that the 10 students' scores on their homework and exams improved. Based on this experiment, the teacher found that students perform better when they are forced to do their work manually rather than relying on machines.
Which of the following statements, if true, would most weaken the above reasoning?
(A) The 10 students performed poorly on the teacher's course before the experiment.
(B) The 10 students were tutored by other students who had the benefit of a calculator or computer.
(C) The 10 students were the best math students in the class.
(D) During the three months of the experiment, the class learned mathematics that only the 10 students had previously learned.
(E) The teacher taught the students to do their homework without the help of a calculator or computer.
25🇧🇷 A recent survey found that 60% of employers polled believe an employee should be fired if they are accused of a crime, while 40% believe an employee should only be fired if they are accused of a crime and convicted of a crime. As a result, more employers believe that an employee should be fired if they are accused of a crime than if they are convicted.
The above reasoning is flawed because
(A) uses a sample of the population to draw a conclusion about the general population
(B) bases a conclusion on an ambiguous term in the supporting statements
(C) uses two different beliefs to come to a conclusion about an entirely different belief
(D) is based on premises that cannot all be true
(E) confuses a sufficient condition with a necessary condition
26🇧🇷 Editorial: History has shown that while politicians promise that lottery winnings will be used for educational purposes, the funds are inevitably diverted to other, non-educational purposes. Therefore, our representatives in the state legislature must not support the proposed lottery, as they will inevitably break their promise to use the winnings to fund education and use the funds elsewhere.
The above reasoning is flawed because
(A) draws a conclusion about a specific population based on a study of a larger population
(B) historical data used to draw incorrect conclusions about a similar situation in the future
(C) assumes that a pattern of behavior under certain conditions will be repeated under completely different conditions
(D) rejects a proposed solution based on the track record of an entirely different solution
(E) attacks a lottery proposal by attacking those who support it rather than the merits of the lottery proposal
PAUSE
IF YOU FINISH BEFORE TIME IS UP, CHECK YOUR WORK FOR IT
SECTION ONLY. DO NOT PROCEED TO ANOTHER SECTION OF THE TEST.
SECTION II
Time – 35 minutes
24 questions
Instructions: The questions in this section are based on justifications in short statements or passages. For some questions, more than one of the options could potentially answer the question. However, you must chooseotherAnswers; that is, the answer that most accurately and completely answers the question. You must not make assumptions that are reasonably implausible, superfluous or inconsistent with the Passage. After choosing the best answer, hide the appropriate space on your answer sheet.
1🇧🇷 Chancellor of Mayfield Academy: Mayfield Academy must grow if it is to survive, so as we agreed, efforts must be made to attract students from all grades K-12. The best strategy for attracting students is to build new athletic, computer and arts facilities, including a new classroom. Parents demand that their children have the best facilities so that they can pursue their interests to the fullest. With other schools also investing in similar facilities, we have to keep up with the competition. Therefore, parents will transfer their children to our school when our facilities match or exceed other facilities.
The argument leads to the conclusion that
(A) Mayfield Academy should attract more students
(B) Parents who want the best facilities for their child's interests should choose Mayfield Academy
(C) Parents should consider public and private institutions
(D) Mayfield Academy should invest in new facilities
(E) Parents who have children in public school must transfer them to Mayfield Academy
2🇧🇷 CEO: Trying to create an ethical business by teaching our employees ethics is a waste of time and money, as the business structure is inherently neither ethical nor unethical when it is created. No matter what we do, people will inevitably act unethically. All we can do is create monitoring systems to prevent problems from occurring and protect the business when they do.
Ethicists: Arguing that we shouldn't train employees on ethics because they will inevitably act unethically makes as much sense as arguing that we shouldn't spend money on driver education because all drivers will inevitably cause an accident.
The method the ethics expert uses to counter the manager's argument is this
(A) argue that there are problems that time and money cannot solve, no matter how wisely spent
(B) Attacking the manager's character rather than the manager's position
(C) show that the executive's reasoning would lead to an unacceptable conclusion if applied to a different situation
(D) show that the manager needs to provide more evidence to support the manager's position
(E) Explain a dilemma central to the manager's argument
3🇧🇷 Renfield is not expected to be promoted to a managerial role. His performance as a member of our workforce, while exemplary, showed little of his managerial ability. Everything he's said and done so far has been to get that promotion, not to show us that he has the ability to lead others. As such, we cannot be certain that he will be an effective manager once in office.
On which of the following assumptions is the argument based?
(A) The duties of an employee in themselves do not demonstrate that the person is capable of managing people.
(B) Renfield is unreliable, even in his current roles as a member of the team.
(C) If Renfield receives the promotion, he does not intend to act in the best interests of the employees under his direction.
(D) The team will not follow Renfield's lead as manager once he receives promotion.
(E) Managers are rarely promoted from the workforce because they are not up to the highest responsibility.
4🇧🇷 Real progress in society is not something that can be achieved without meaningful and transformative action. It will only come when those in power have the courage to overcome significant obstacles and overthrow previously accepted norms that are harmful to society.
The reasoning in the argument is fallacious because the argument
(A) undermines its own premise that a particular attribute is present in all instances of a particular phenomenon
(B) concludes that since an influence is the most important influence on a given phenomenon, that influence is the only influence on that phenomenon
(C) denies that observing that a certain pattern is common to phenomena within society can help to observe a causal explanation of the phenomenon
(D) concludes that the characteristic of a type of phenomenon in society that occurs at a given point in time is characteristic of similar phenomena at all times
(E) selects an impact on a particular phenomenon in society as an indication that its impact outweighs any other impact on those phenomena
5🇧🇷 Educator: Scholars are concerned that the growing popularity of online universities and low residency degree programs will cloud the classroom experience. They reject this trend, saying that online courses don't offer the level of collaboration and support that the traditional classroom does. However, at Plymouth Online we offer a fully interactive experience with web video, web-based collaboration software, online chat and video conferencing sessions. Thus, it is possible to enroll in our online university and live an experience very close to the classroom, and in some situations the experience is richer and more satisfying.
The educator's argument continues
(A) Reference to a scientific authority to challenge a widely held belief
(B) question the veracity of evidence presented in support of an opposing position
(C) to offer a counterexample to a prevailing opinion among experts in the field
(D) Proposing an alternative sociological explanation for a pedagogical practice
(E) Distinction between teaching approaches
6🇧🇷 Until recently, it was believed that strength training does not help children, but actually harms them and possibly stunts their growth. A new study finds that resistance training helped children between the ages of 6 and 18 get stronger, even if they couldn't gain muscle mass like adults. Therefore, our government should require strength training for all children in public schools between the ages of 6 and 18.
Which of the following statements, if true, most weakens the argument?
(A) The school year lasts only nine months a year, and children's strength training must be consistent throughout the year. A home program is necessary for success.
(B) As the children are not gaining muscle mass, it will be difficult to measure their progress without the specialized instruments used in the study.
(C) The study was conducted with individuals from only one geographic location where strength training is culturally more acceptable.
(D) Strength gains over a nine-month school year will be minimal. It takes at least a year for children to show significant results from strength training.
(E) Children do not respond positively to government regulations, and low morale can adversely affect the results of government-imposed strength training regulations.
7🇧🇷 According to one study, an exercise program that combines aerobics and weight lifting can help people with type 2 diabetes lower their blood sugar. This combined program offers greater benefits than doing either type of exercise alone. Therefore, all people with type 2 diabetes should be involved in an exercise program that includes aerobics and weight lifting.
Which of the following statements would weaken the argument the most?
(A) People with type 2 diabetes are less able to physically cope with aerobic exercise and weight lifting.
(B) Aerobics and weightlifting are mutually exclusive and therefore cannot be practiced together.
(C) The physical demands of a combination program can have adverse physical effects that negate the benefits of lowering one's blood sugar.
(D) Some individuals with type 2 diabetes, because of their unique physical needs, require only one of the exercise activities to receive the benefits in their blood sugar.
(E) The study considered all key indicators, including blood glucose, that can be influenced by exercise.
8🇧🇷 For the first 140 million years of existence on Earth, mammals were relatively small, ranging from 2 to 22 pounds, but when the dinosaurs became extinct, mammals grew up to 1,000 times, reaching a size of 17 tons. Once the herbivorous dinosaurs were gone, mammals no longer had to compete for space to roam and vegetation for food. They could eat as much as they wanted, and the larger mammals were better able to fend off predators. That's why we have today's hippos and giraffes.
If the above statements are true, which of the following statements must also be true based on them?
(A) If dinosaurs had lived longer, today's mammals would be much smaller but still herbivores.
(B) If the hippopotamus we know today were 1,000 times smaller, other mammals would also be significantly smaller.
(C) If other larger non-mammals had survived the event that led to the extinction of the dinosaurs, modern mammals might have been much smaller due to competition for resources.
(D) If mammals had been larger at the time of the dinosaurs, the resulting competition would have killed both of them.
(E) If hippos and giraffes had evolved as carnivores instead of herbivores, the large mammals probably would not have competed with dinosaurs for space and food.
9🇧🇷 At an auction, no one wants to buy the Alberto Giacometti statue more than Jody does, but Jody won't be attending the auction, so no matter how much auctioneers lower the reserve price, no one will bid on the statue.
The flawed reasoning in the above reasoning most closely corresponds to the one in which of the following?
(A) The jockey who would most like to ride the horse Black Lightning in the next race is Cornwall, but he suddenly tells his owner that he cannot ride in the next race. This means that the other jockeys who have given up on riding Black Bolt will redouble their efforts.
(B) Larry can spot a fake Renaissance painting better than anyone, but he still hasn't found any flaws in Renny's painting. So there must be a forgery among the other paintings in his collection.
(C) If anyone can translate this ancient text it is Professor Ricardo, but he is currently involved in a project to translate a much more significant text and will probably have no interest in undertaking this minor project. So we have to wait to find a similar person to do the translation.
(D) Although Emilio is more eager to get the sales job than everyone else, he does not apply for the job. It follows that no one else will apply for the position of salesperson, whatever the salary offered.
(E)Sherry wanted to participate in an extra-credit group project for her science class, but every time they called a meeting, she was too busy with cheerleading exercises to join them. So the closer she gets to being the cheerleader, the less time she can dedicate to her studies.
10🇧🇷 If the conductor does not want to participate in the New York competition, we should consider other competitions. If the orchestra does not want to enter the New York competition, we must skip all competitions. And it must be that neither the conductor nor the orchestra want to participate in the New York competition.
If the above statements are true, which of the following must also be true?
(A) If the orchestra agrees to enter the New York competition, we should skip the competitions altogether.
(B) We should only consider other competitions when it is more likely that the conductor and orchestra will participate.
(C) We must try to convince both conductor and orchestra to enter the New York competition.
(D) If the conductor agrees to enter the New York competition, we must skip the competitions altogether.
(E) We should only consider other competitions if the conductor is more likely to participate.
11🇧🇷 It is ludicrous to suggest that a newspaper need not be newsworthy if it does not report an event, as can be seen by rephrasing: no event is newsworthy unless it appears in a newspaper.
Which pattern of reasoning in which one of the following statements is most similar to the reasoning above?
(A) The claim that every politician elected to Congress becomes corrupt is absurd, as can be seen by restating: No politician is elected to Congress without becoming corrupt.
(B) The claim that every politician elected to Congress becomes corrupt is absurd, as can be seen by restating it: No politician who is not elected to Congress becomes corrupt.
(C) The claim that every politician elected to Congress becomes corrupt is absurd, as can be seen by reformulating it: No politician can become corrupt unless he is elected to Congress.
(D) It is ridiculous to say that a student who has never been accused of cheating has never cheated, as can be seen by rephrasing the sentence: Any student who cheats is accused of cheating.
(E) It is ridiculous to claim that a student who has never been accused of cheating has never cheated, as can be seen by rephrasing it: every student accused of cheating.
12🇧🇷 PHILOSOPHER: Pragmatism is the view that meaning or value is determined by practical consequences. If an approach has the desired effect, then it's good. So if a theory works in practice, it is correct. If it doesn't work then it must be wrong. However, pragmatism is a dangerous philosophy because it is impossible to develop a general truth that applies to everyone or guide moral or ethical decisions. What is "good" for one person may not be for another.
For which of the following reasons is the philosopher's argument most vulnerable to criticism?
(A) Discrepancies between perceptions of what is practical and what is good are a matter of people getting together and deciding what actions and inactions are practical and good on a societal level.
(B) The individual cannot always successfully determine what is practical and good. Only through experience can one develop better judgment. Thus, experience is the great leveler and resolves discrepancies between perspectives on a course of action.
(C) While it is true that one person may believe that an action is practical and good, the only reason the action can conflict with another person's action is because the other person has a poor sense of what is right and wrong. of what is wrong.
(D) There is a possibility that a person's action is practical but wrong if another close person determines that the action is contrary to society's values.
(E) Other philosophies have failed to adequately establish a general truth for human action, so pragmatism is no different from other philosophies.
13🇧🇷 Press Release: A committee of physicians reviewed nearly 1,000 publications and concluded that the high levels of vitamin D and calcium recommended by physicians and testing laboratories are unnecessary for maintaining bone health. Food manufacturers add vitamin D and calcium to the foods people eat every day, not to mention people get vitamin D from sun exposure. Therefore, everyone should stop consuming large amounts of vitamin D and calcium immediately.
Which of the following points to a weakness in the reasoning of the press release?
(A) Vitamin D and calcium work together, not separately, to maintain bone health.
(B) Physicians reviewed other publications rather than designing their own study and reviewing a direct patient study.
(C) Doctors recommend high levels of vitamin D only for those who have been tested for poor bone health.
(D) High levels of vitamin D and calcium bring benefits beyond the maintenance of bone health.
(E) Many of the foods with added vitamin D and calcium are breakfast foods and people tend to skip breakfast regularly.
Questions 14-15
Company X's board of directors decided to sell its underperforming businesses that drove down its stock price. That sounds like a drastic change to some analysts, but the president of Company X has long claimed that among the troubled divisions are some with outdated systems and inferior products. He argued that upgrading its operations would be too expensive. Therefore, the board's action would not hurt but actually help the stock price of Company X.
14🇧🇷 On which of the following assumptions does the conclusion drawn depend?
(A) All businesses for sale are 100% owned by Company X.
(B) Only a very sophisticated buyer will be willing to buy such underperforming businesses from Company X.
(C) Company X cannot sell its underperforming units until it improves its operations.
(D) All companies that the Board decides to sell include those recommended for sale by the President.
(E) Corporate buyers cannot identify an underperforming division.
fifteen🇧🇷 Which of the following statements, if true, most weakens the argument?
(A) The stock price is determined not only by the company's best performing departments, but by the interaction between all its departments.
(B) High-performing departments charge a much higher price than low-performing units.
(C) Underperforming departments have always been seen as bad investments by Company X.
(D) Buyers who buy underperforming departments and then sell them at a higher price care little about the resulting inflation in the market, but large companies like Company X are very concerned about the resulting inflation.
(E) The Directors are currently investigating to find means other than selling the underperforming businesses that would increase the Company's share price.
16🇧🇷 A new restaurant that survives more than six months is popular with local customers or food critics. Last year, all the new restaurants that were popular with food critics were also popular with local guests. As a result, every new restaurant that survived for more than six months in the last year has become popular with food critics.
What pattern of reasoning in which one of the following arguments is most similar to the above argument?
(A) All auto repair shops in Caedmon Township carry out maintenance on domestic and foreign automobiles. Larry's Auto Shop is a mechanic shop that does not serve foreign cars. Therefore, Larry's Auto Shop is not located in Caedmon Township.
(B) All Willow Construction Company interns learn drywall or furniture making in their second year. This year, all interns who study drywall construction also learn furniture construction. Therefore, every Willow apprentice studies drywall.
(C) Former congressmen teach at their hometown university or write a book about their years in politics. Mary Seldon is a former congressman who teaches at Riverside University in her hometown of Riverside. That's why Seldon doesn't write a book about his years in politics.
(D) Every bestselling novel published in the last year is well written and has a compelling plot. The novel Jacob Rain, published last year, is well written and has an exciting plot. Therefore, the novel by Jacob Rain is a bestseller.
(E) To be successful, most new movies need a big publicity campaign or a big star cast. But most movies with a big star cast automatically get a big hype. Therefore, a film cast with big stars is guaranteed to succeed.
17🇧🇷 A rice plant was discovered to contain protein molecules that recognize and bind to specific molecules on invading organisms, signaling the plant to mount an immune response and fight microbial infections and diseases. This immune response is not unlike the way the human immune system fights disease. It won't be long before we find out how plants fight the common cold.
Which of the following is an assumption on which the argument depends?
(A) The same diseases that afflict humans also afflict plants.
(B) The common cold is a disease that the human immune system cannot fight off.
(C) Because plant and human immune systems are similar, diseases attack both in the same way.
(D) The immune system of a rice plant is the same as the immune system of other plants.
(E) Protein molecules bind to invading molecules in plants in the same configuration as in humans.
18🇧🇷 Joseph: A computer company's night service representatives handle complaints an average of 20% faster than day service representatives, but they provide a similar quality of service when handling complaints. That way, night reps work more efficiently.
Davis: Your conclusion is unfair. Different times of day place different demands on account managers. Since late night calls tend to be about computer problems at home, the issues are less complex than during the day when more business-oriented calls are made to the customer service center.
The point of contention between Joseph and Davis is
(A) why night shift customer service representatives can stay on the night shift as opposed to the day shift
(B) the relationship between the time of day and the time it takes a customer service representative to process a customer complaint
(C) how the company might decide to rotate customer service representatives between day and night shifts to increase efficiency in handling complaints
(D) the accuracy of the 20 percent value regarding the different speed of handling complaints between night and day customer representatives
(E) the reason why night agents are able to provide the same quality of service as day agents, spending less time, on average, with each customer
19🇧🇷 In a legislature, much of the legislation is written by industry experts, who also act as lobbyists on behalf of their respective industries, who will be affected by the legislation they draft. Because these experts know the industry much better than legislators, they are better able to craft laws that do what legislators require without harming the companies or entities that must comply with the resulting regulations. But this strategy is clearly based on bad reasoning. After all, industry experts are paid to act as lobbyists to represent their employers' interests, and any legislation they make is likely to be watered down in the interests of their employers' self-interest rather than the best interests of the legislative constituency.
The author cautions that the most common type of legislation may not be in the interest of constituents in the legislature because
(A) Many industry experts may let their self-interest as industry lobbyists influence their role as legislators, acting on behalf of elected officials
(B) Most industry experts are heavily financially influenced to write unfair laws thanks to external lobbying contracts
(C) Most legislators would be less corrupt than industry experts when making legislation
(D) Many industry professionals create watered down laws to one day gain lucrative jobs in their respective industries
(E) Many industry experts are well paid and want to keep lobbying jobs by creating weak laws that favor their respective industries
20🇧🇷 All diamonds cut by Richman are over three carats and all rubies are under three carats. Most gemstones cut by Richman have a clarity rating below SI2. All Allister cut diamonds and rubies have a clarity rating greater than SI2 and less than three carats. Ellington Jewelry, which sources its diamonds and rubies from only these two companies, only buys stones weighing less than 3 carats. Ellington is buying a large shipment of diamonds.
If the above statements are true, which of the following statements must be true based on these statements?
(A) Ellington only buys diamonds.
(B) All Allister's stones are of lesser purity than Richman's.
(C) The supply of diamonds purchased has a clarity rating above SI2.
(D) The shipment of diamonds purchased belongs to Richman.
(E) Ellington does not buy stones from Allister.
Questions 21-22
Coal-fired power plant ladder: Regardless of technological advances over the next 30 years, coal will remain the most cost-effective source of energy. Despite what some people claim, the cost of alternative energy sources such as solar and wind technologies will never drop to levels that can compete with coal. Coal is here to stay.
Environmentalists: I disagree. Over the past 10 years, there have been significant improvements in solar cell and wind turbine technology. You are more efficient than ever. These new technologies continue to evolve, while the coal industry has been slow to adopt the latest clean coal technologies. Eventually charcoal will become obsolete.
21🇧🇷 The reasoning in the environmentalist's argument is flawed because of the reasoning
(A) ignores the long time it will take for alternative energy sources to become competitive with coal
(B) does not recognize the cost advantage of coal over alternative energy sources such as solar and wind technologies
(C) misinterprets the coal plant manager's discussion of technical advances by saying that the coal industry intends to adopt clean coal technologies
(D) ignores the possibility that coal and alternative technologies can work together to create a cleaner energy future
(E) assumes that the cost of alternatives to coal will decrease with the introduction of more efficient solar and wind technologies
22🇧🇷 Which of the following statements, if true, most strongly supports the environmentalist's counterargument?
(A) Engineers have discovered limits to technological advances in the coal industry that do not exist in the wind and solar energy industries.
(B) Thirty years is a very small window to assess whether one energy technology will prevail over another.
(C) A direct link was found between technological advances in the energy sector and cost adoption drivers.
(D) A coal company in one state is in the process of converting its facilities to clean coal technology.
(E) Improvements in solar and wind technologies have accelerated, but costs have improved only slightly over the last 10 years.
23🇧🇷 Randy's boss asked him to write a management report on training new financial advisors coming into the company, but because of other projects assigned to him, he put off until the day before the scheduled date to make a presentation based on the report. . for his boss. Randy can still finish the report on time, but only if he works on it all night without stopping. However, one of his clients invited him to dinner and asked Randy to stay later to discuss the client's portfolio in detail. Therefore, Randy will be forced to choose between satisfying his boss and fulfilling his obligations as a client.
the dispute continues
(A) Provide a version of events and an alternative version of events, both with mismatch between the factors involved
(B) Explain the difficulties inherent in one situation in terms of another situation with similar difficulties
(C) show how one set of responsibilities, with its own conditions to be met, is inconsistent with another set of responsibilities and presents a puzzle to the individual
(D) describing the struggles involved as a series of events creates a different series of events, leaving the subject in a quandary
(E) Investigating how a person's failure to act in a situation leads to a difficult conundrum, and how the person's choices in that situation can result in unavoidable harm to the other parties involved
24🇧🇷 Bumblebees have been found to visit and pollinate red or striped snapdragons more often than white or pink flowers. Bumblebees are the snapdragon's main pollinators, as the pollen carried by the bumblebee is necessary for the flower to open its petals. Therefore, to increase the bumblebee population, nurseries are encouraged to grow more striped and dark-pigmented snapdragon species.
Which of the following statements can be correctly derived from the passage?
(A) The bee population is threatened by a shortage of striped or darkly pigmented snapdragons.
(B) If there are not enough striped or darkly pigmented snapdragons, all snapdragons will not be able to open their petals.
(C) When there are more striped and darkly pigmented snapdragons, other insects are forced to open the petals of the white and pink varieties.
(D) The bee population is lower than desired, and more striped, darker-pigmented snapdragons will help to increase their numbers.
(E) Red and striped snapdragon species are endangered and more bees are needed to increase their numbers.
PAUSE
IF YOU FINISH BEFORE TIME IS UP, CHECK YOUR WORK FOR IT
SECTION ONLY. DO NOT PROCEED TO ANOTHER SECTION OF THE TEST.
SECTION III
Time – 35 minutes
26 questions
Instructions: Each set of questions in this section is based on a set of conditions. When answering some questions, it can be helpful to draw a rough diagram. Choose the answer that most accurately and completely answers each question and hide the appropriate space on your answer sheet.
Questions 1-6
A mechanic is considering using coils D, E, G, H, I, J, and K to replace the old ones in a machine. The mechanic has developed a test to determine its quality, but the mechanic has limited time to test the coils. Only tested bobbins can be used in the machine. The selection process must meet the following requirements:
When G is tested, I is tested.
If E is tested, G will be tested.
D is tested.
H is not used unless J is tested.
D is not used unless H is tested.
If J is used and I is tested, then K is used.
1🇧🇷 Which of the following would be a complete and accurate list of tested coils?
(A) D, E, G
(B) D, E, I
(C) D, E
(D)D, me
(E)D,G
2🇧🇷 Which of the following statements could be true?
(A)E and three other coils are the only coils tested.
(B)E and two other coils are the only coils tested.
(C)E and another coil are the only coils tested.
(D) D, G and H are the only coils tested.
(E)I and J are the only coils tested.
3🇧🇷 If J is not tested, which of the following statements must be true?
(A)D is not used.
(B)H is not tested.
(C)D is tested, but H is not used.
(D)D is used, but H is not used.
(E)H is tested but not used.
4🇧🇷 If E and five other coils are the only coils tested, and if exactly three coils are used in the machine, which of the following would be an accurate list of coils used?
(A) D, I, J
(B) D, H, J
(C) E, G, I
(D) H, I, K
(E) E, G, J
5🇧🇷 If every coil tested is used and if I is used, each of the following coils must be tested EXCEPT:
(A)K
(B)H
(C)J
(D)D
(Z.B
6🇧🇷 If K is not used and exactly four coils are used, which of the following must be wrong?
(A) I is used.
(B)J is used.
(C)G is tested.
(D)H is tested.
(E)E is tested.
Questions 7-13
Exactly seven children - Barry, Ezra, Jaime, Karly, Pakhi, Sharon and Usef - are participating in a craft class at the same time. During class, they can choose between three activities: pottery, drawing and origami. Each child participates in exactly one of the activities. Activities only occur once and one activity at a time. The following restrictions must apply:
Exactly twice as many children choose drawing than ceramics.
Sharon and Usef participate in the same activity.
Ezra and Karly are not participating in the same activity.
Barry and Pakhi are not participating in the same activity.
Barry takes part in pottery or origami.
Jaime participates in the raffle.
7🇧🇷 Which of the following statements could be a correct assignment of children to activities?
(A) drawing of Karly; pakhi design; usef origami
(B) Karly-Origami; Pakhi-Origami; Sharon-Origami
(C) drawing of Ezra; pakhi design; Ceramics Sharon
(D) Barry-Origami; Esra-Keramik; Sharon-Keramik
(E) Drawing of Barry; Ezra origami; karly drawing
8🇧🇷 Which of the following could be a complete and accurate list of children who choose not to paint?
(A) Barry, Esra, Vogel
(B) Barry, Sharon, Usef
(C)Barry, Karly, Usef
(D)Barry, Karly
(E) Barry, Pakhi
9🇧🇷 Each of the following statements must be false EXCEPT:
(A) Exactly two children choose origami.
(B) Usef is the only child who chooses origami.
(C) Exactly three children decide to draw.
(D) Pakhi is the only child who decides to draw.
(E)Pakhi is the only child who chose pottery.
10🇧🇷 If exactly one of the children chooses origami, which of the following statements must be true?
(A)Karly chooses pottery.
(B) Sharon chooses to draw.
(C)Barry chooses pottery.
(D) Pakhi decides empathar.
(E)Ezra chooses pottery.
11🇧🇷 If Sharon chooses the same activity as Barry, which of the following statements could be true?
(A) Ezra decides to draw.
(B) Pakhi chooses origami.
(C)Barry chooses pottery.
(D)Usef chooses pottery.
(E) Sharon decides to draw.
12🇧🇷 Any of the following could be a complete and accurate list of children choosing pottery EXCEPT:
(A)Barry, Karly
(B) Karly, Pakhi
(C) Esra, Pakhi
(D) Barry, Ezra
(E)Sharon, Usef
13🇧🇷 Which of the following statements must be true?
(A) Ezra chooses a different activity than Sharon.
(B) Ezra, Jaime, and Sharon do not all choose the same activity.
(C) Ezra chooses a different activity from Jaime.
(D) Barry, Jaime, and Karly do not all choose the same activity.
(E)Barry chooses a different activity than Ezra.
Questions 14-20
A scientist must form two test groups - Group 1 and Group 2 - of six out of seven laboratory rats - three brown rats named Abby, Carl and Dennis; and four white mice named Elle, Fern, Horn and Iris. Each group has three mice. No mouse can be in more than one group. Each group must have at least one brown rat and one white rat. The composition of the groups must comply with the following conditions:
Neither group includes Abby and Iris.
Neither group includes Elle and Horn.
If a group includes Dennis, it does not include Horn or Iris.
If Group 1 contains Carl, Group 2 contains Horn.
14🇧🇷 Which of the following could be the composition of the two groups?
(A)Group 1: Abby, fern, horn
Group 2: Carl, Elle, Iris
(B) Group 1: Carl, Dennis, Iris
Group 2: Abby, Elle, Farn
(C) Group 1: Carl, Dennis, She
Group 2: Farn, Horn, Iris
(D)Group 1: Abby, Elle, Iris
Group 2: Carl, Farn, Horn
(E)Group 1: Abby, Dennis, Fern
Group 2: Carl, Elle, Horn
fifteen🇧🇷 If Carl is in Group 1, which of the following couples should be in Group 2 together?
(A) Fern and iris
(B) Abby e Horn
(C) horn and iris
(D) Elle e Fern
(E) Dennis and Fern
16🇧🇷 If Elle is in Group 1, which of the following pairs of rats could be in Group 1 along with Elle?
(A) Fern and iris
(B)Carl e Farn
(C)Carl e Horn
(E)Abby e Carl
(E)Carl e Iris
17🇧🇷 If Abby is in the same group as Dennis, which of the following statements could be true?
(A) Elle and Fern are in Group 2.
(B) Horn is in Group 1.
(C) Carl is in Group 1.
(D) Elle and Fern are in Group 1.
(E)Iris is in Group 2.
18🇧🇷 Any of the following pairs of mice could be in Group 2 together EXCEPT:
(A)Carl e Horn
(B) Abby e Horn
(C) Abby e Carl
(E) Abby e Dennis
(E)Carl e Dennis
19🇧🇷 Which of the following statements could be true?
(A) Carl is in Group 1 and Elle is in Group 2.
(B) Abby is not in any group and Carl is in group 1.
(C) Abby is in Group 1 and Elle is in Group 2.
(D) Abby is in Group 2 and Dennis is in Group 2.
(E)Abby is in group 2 and Carl is not in any group.
20🇧🇷 If Dennis is in Group 2, which of the following must also be in Group 2?
(A horn
(B) Farn
(C) Iris
(D) Carlos
(E) Abby
Questions 21-26
Of the eight loads - L, N, O, P, R, S, T and W - only six can be loaded onto the three available trucks - Truck 1, Truck 2 and Truck 3. Each load must be loaded onto one of two handles - front and rear. Loads must be carried in sequence, first at the front, then at the rear, under the following conditions:
L can only go in truck 2.
T and W cannot rear-end a truck.
If S is loaded onto a truck, then N and O go onto the next truck, unless S is loaded onto truck 3.
21🇧🇷 Which of the following could be the load register for the load?
(A) Truck 1: Front, S; back, no
Truck 2: Front, W; back, right
Truck 3: Front, T; back, right
(B) Truck 1: Front, N; go back w
Truck 2: Front, T; back, oh
Truck 3: Front, R; back, p
(C) Truck 1: Front, T; back, p
Truck 2: Front, O; back, no
Truck 3: Front, P; back, I
(D) Truck 1: Front, R; back, no
Truck 2: Front, L; back, p
Truck 3: Front, W; back, oh
(E) Truck 1: Front, O; back, t
Truck 2: Front, N; back, p
Truck 3: Front, S; back, right
22🇧🇷 If P and R are unloaded, the fronts of Truck 1, Truck 2 and Truck 3 can carry the following load:
(A)T, W and O
(B)E, W e N
(C) W, L e N
(D)N, L e S
(E)S, N e T
23🇧🇷 Which of the following is a pair of loads that must be loaded on truck 3 when loaded on the same truck?
(A)O and W
(B)L e R
(C)N e S
(D)T e P
(E)P e S
24🇧🇷 If L, S, and W are loaded onto different trucks, which of the following could be true?
(A)T is in truck 1.
(B)S is in truck 2.
(C)N is in truck 3.
(D)W is in truck 3.
(E)S is in truck 1.
25🇧🇷 If N, P, and R, not necessarily in that order, are loaded in front of the three trucks, which of the following statements must be true?
(A)P is in truck 3.
(B)O is in truck 3.
(C)P is in truck 2.
(D)N is in truck 1.
(E)O is in truck 1.
26🇧🇷 If P is at the back of truck 2 and N is at the back of truck 3, which of the following could be the list of loads at the front of truck 1, truck 2, and truck 3, respectively?
(A) W, O and R
(B) W, L e O
(C)R, L e O
(D)O, R e S
(E)S, O and R
PAUSE
IF YOU FINISH BEFORE TIME IS UP, CHECK YOUR WORK FOR IT
SECTION ONLY. DO NOT PROCEED TO ANOTHER SECTION OF THE TEST.
SECTION IV
Time – 35 minutes
25 questions
Instructions: Each passage in this section is followed by a set of questions to be answered based on what isspecifiedorit impliesin the pass. For some of the questions, more than one of the options could answer the question. However, you must chooseotherAnswers; that is, the answer that answers the question most accurately and completely, and hide the appropriate space on your answer sheet.
1🇧🇷 The main purpose of the section is to provide an answer to which of the following questions?
(A) How did the Supreme Court fail to protect the "one person, one vote" precedent, and how did voters act to correct this failure?
(B) What mistakes do states make in trying to solve a national problem at the state level, and how will their solutions impede democracy and lead to injustice in congressional elections?
(C) What are the unique problems of the electorate in California and Florida, and what efforts did voters make to resolve them?
(D) What laws did voters in two states adopt as a solution to a perceived political problem, and what key counterarguments can be used in future efforts, as illustrated by a specific case?
(E) How did reform-minded citizens misunderstand the problem of legal apportionment and enact costly and ill-advised solutions?
2🇧🇷 The passage implies that, prior to 1964, which of the following statements could be true?
(A) Political parties had no voice or control over legislative allocation.
(B) Some citizen votes counted less than a full vote.
(C) One district may be designed to contain more components than another.
(D) Gerrymandering was legal for some states and not for others.
(E) The Supreme Court could overturn any reallocation based on the “one person, one vote” requirement.
3O debate no SupremoWesberry x SandersThe case in lines 18-26 is primarily intended for this
(A) show that the Supreme Court failed to resolve the "one person, one vote" problem that has plagued the electoral system since the nation's inception
(B) Explanation of the legal bases of the proposed solutions to the problem of legislative division
(C) Explain that while one branch of the country's government tried to solve the problem, another branch undermined those efforts
(D) provide a historical perspective on a problem with democratic representation that has not yet been resolved through legal redistribution
(E) Establish that the Supreme Court is in fact the root cause of gerrymandering and other political party interference in the voter mapping process
4🇧🇷 The discussion of Proposition 27 on lines 53–59 implies that which of the following statements was true prior to the passage of Proposition 20?
(A) The Citizens' Redistricting Commission, while a good idea, was an impractical institution and needed to be eliminated for redistricting to work properly.
(B) Politicians wanted to create confusion by having both proposals on the same ballot in hopes that voters would favor Proposition 27 over Proposition 20.
(C) The Citizens' Redistricting Committee existed before the election to redistribute the state legislative map, but had no authority over the congressional map.
(D) Voters rejected Proposition 27 because it conflicted with Proposition 20, but would have accepted it if an agreement had been reached.
(E) The Citizens' Redistricting Commission was unconstitutional, and if voters did not remove it, the courts would have to do so.
5🇧🇷 Which of the following statements, if true, would most strongly invalidate the California Proposition 20 arguments discussed in the third paragraph?
(A) Evidence shows that a clear majority of voters re-elect their deputies because they are satisfied with their performance.
(B) Recently, several bills were passed because Republican and Democratic congressmen were able to unite and compromise on the legislation.
(C) The proposal would deny the commission the right to use income, race, or sex as factors in determining district shape and size.
(D) It is established that one of the eligible members of the Commission is, in fact, a member of a political party.
(E) Population movements between districts are so fluid from year to year that it is almost impossible to predict the voting behavior of any district in any given election.
6🇧🇷 Which of the following statements accurately expresses the meaning of the word "anchored" as used in line 38 of the passage?
(A) Not eligible
(B) low
(C) Imperturbable
(D) Teimoso
(E) ruthless
PASSAGE A
PASSAGE B
7🇧🇷 Which of the following statements best describes the relationship between passage A and passage B?
(A) Passage A and Passage B offer similar solutions to the same problem.
(B) Passage A presents a problem and passage B presents a solution to that problem.
(C) Passage B expressly contradicts a number of implicit assumptions made by the author of passage A.
(D) Passage A and passage B describe similar movements arising from the same circumstances.
(E) Passage B discusses a refinement of an idea that passage A outlines.
8🇧🇷 What can we deduce from passage A about theater in the early twentieth century?
(A) The advent of agit-prop theater was an entirely new concept of political theater designed to inspire action in its audience.
(B) The political ideas represented by plays like The Urge for Organized Labor were the result of people like Lee Strasburg importing ideas from Russians like Stanislavski.
(C) Broadway in the 1920s featured a musical or lighthearted comedy rather than a tragic play depicting a struggling family.
(D) The public lost interest in "serious drama" after the end of the Depression.
(E) Playwrights and actors resented the political nature of plays, forcing theater managers after the end of the Depression to revert to pre-Depression theater types.
9🇧🇷 Each of the following is mentioned as a difference between Theater Group and Shock Group, EXCEPT
(A) the use of untrained actors
(B) the filing of social complaints
(C) the physical location where the play will be performed
(D) the names of the characters in the plays
(E) the evocation of social change
10🇧🇷 According to Passage B, the Shock Troupe would be less likely to perform a play that includes:
(A) a staged discussion between "Freed-Slave Man" and "White Slave Owner" about the lasting effects of slavery and racism in America
(B) a public re-enactment of workers exiting the production line at a local car factory to protest poor working conditions
(C) a dramatic argument between Jack, an unemployed blacksmith, and his wife, Mary, about her long hours working as an assistant to a manager of a company that Jack believes has romantic intentions for her
(D) an unemployed man sells his last belongings to a bartender in exchange for a place to sleep in a warehouse, then gives a speech about how America's free market system has robbed him of his dignity
(E) an impromptu performance before a miners' union meeting which, through dramatic effect, calls on workers to stand up and speak out about low wages and dangerous working conditions
11🇧🇷 Which of the following statements would the author of section B be likely to believe about group theater as described in section A?
(A) The use of trained professional actors in group theater allowed a better connection with the audience.
(B) The Group Theater was better known to theatrical audiences in the 1930s than the Shock Troupe.
(C) The actors associated with the Shock Troupe were less idealistic than the actors associated with the Theater Group.
(D) The production of the Theater Group ofWaiting for Leftyturned out to be more popular with contemporary audiences than the Shock Troupe productionnewsboy.
(E) "The method" used by actors in group theater is the most effective means of teaching acting.
12🇧🇷 Which of the following is the meaning of the term "agit-prop" as implied in the context of both passages?
(A) Agricultural Proposal
(B) Agitation Proposal
(C) Agitation-Advertising
(D) Agitation-Propagation
(E)Self-Agitation
13🇧🇷 Which of the following statements best captures the main point of the passage?
(A) People are happier in a universal welfare state than in a partial welfare state, and therefore the United States, as a partial welfare state, will have a hard time keeping its population happy .
(B) People are happier in a universal welfare state, but happiness is a weak basis for judging a society, and therefore recent studies on the relationship between the welfare state and happiness are flawed.
(C) While the United States is locked in a struggle between those for and against welfare programs, the rest of the world has forged ahead with building welfare states that have achieved high levels of social happiness.
(D) While politicians, social scientists, and economists argue for the welfare state using arguments not supported by data, Rothstein, Pacek, and Radcliff offer unique support for this through analysis of well-researched data.
(E) Happiness as a basis for analyzing the success of a political state is valid, but the SWB measurement is flawed. Only through improved data collection and a more accurate happiness index can the study of the welfare state be accepted by today's thinkers.
14🇧🇷 Which of the following statements best describes the organization of the passage?
(A) A social issue is discussed. An alternative way of looking at this problem is discussed. The alternative method is evaluated. The social issue is discussed in relation to the alternative method.
(B) A social problem is presented. A solution is discussed. The approach is evaluated. The approach is discussed in relation to a specific nation experiencing the social problem.
(C) Alternative perspectives on a social problem are presented. An approach to solving the problem is presented. A criticism is made that discredits this approach. A specific case study aims to show that the approach is not viable.
(D) A social problem is analyzed. A solution to the problem is suggested. The strengths and weaknesses of the solution are discussed. The practical difficulties in providing the solution are presented.
(E) A social problem is discussed. A unique approach to the problem is discussed. The validity of the assumption underlying the approach is debated. A special case is discussed regarding the approach.
fifteen🇧🇷 The author of the passage points out Rothstein's bias in line 32 because
(A) Rothstein is a well-known and outspoken advocate of welfare programs
(B) Rothstein was an unnamed author of the study with Pacek and Radcliff and therefore has an interest in drawing attention to his ideas
(C) Rothstein lives in a northern European country that is a long-standing universal welfare state
(D) Rothstein invented the SWB measurement and wants it to be used in future studies
(E) Rothstein is currently running for political office in his country and preserving the welfare state is a central motive for his candidacy
16🇧🇷 Which of the following statements most refutes Rothstein's argument about the "arbiter" in lines 48-52?
(A) Political scientists and public policy experts experience happiness and unhappiness because of welfare programs like anyone else, so their criteria are not that different from those of the general public.
(B) Happiness or unhappiness is influenced by factors outside the political spectrum, such as B. the availability of camaraderie, community support, and social opportunities, while experts filter these topics and focus on specific problems.
(C) Political scientists and public policy specialists tend to view problems from a more theoretical perspective and fail to see the fundamental elements that can affect the success or failure of a welfare state.
(D) The general public tends to be influenced by waves of economic ups and downs that affect happiness, while political scientists and public policy experts are all wealthy individuals unaffected by economic changes.
(E) SWB measurement has proven to be very accurate in predicting the success of welfare programs, whereas simulation models created by political scientists and public policy experts have rarely been successful in making such predictions.
17🇧🇷 Based on the first paragraph of the passage, which of the following statements, if true, would support an argument made by advocates for a more universal welfare program in the United States?
(A) The results of a study show that unemployment benefits reduce the number of applicants for a job.
(B) A survey of welfare recipients shows that the vast majority save rather than spend the money.
(C) Research has shown that the number of registered voters increased significantly in the 10 years following the introduction of certain welfare programs.
(D) A survey of welfare program participants found that a large percentage expressed significant resentment toward the government.
(E) The results of a study show that a large part of the population is happier because of the existence of social programs.
18🇧🇷 Which of the following statements best describes the author's attitude toward Rothstein's approach to happiness and the well-being state?
(A) Cautious neutrality
(B) strong condemnation
(C) moderate defense
(D) mild skepticism
(E) reluctant acceptance
19🇧🇷 Which of these claims would Rothstein accept least?
(A) The ability of its population to maintain and form family ties is a factor that can obscure the relationship between SWB and the establishment of a universal welfare state.
(B) Adding a program to protect children from the effects of poverty will make the people of a nation happier.
(C) Northern European countries have higher SWBs than the United States and other countries with non-universal wellness programs.
(D) A long-term program that all the people of the nation trust and know will be there when they need it will increase the SWB for this nation.
(E) The discovery that a large proportion of welfare program funds have been diverted to a politician's pet project proves that the welfare state is a failure.
20🇧🇷 Which of the following statements most accurately expresses the main idea of the passage?
(A) Companies have found a way to incorporate philanthropy into their operations and sales to avoid the stigma associated with giving away money, which is seen as a cheap marketing tactic. Why they get involved in philanthropy is a more complicated question.
(B) Businesses are increasingly incorporating philanthropy into their operations, but the financial impact on customers, shareholders and employees is no different than a business making a cash donation to a charity. Why companies do this is a more complicated question.
(C) Regardless of how corporations' actions are perceived, as an integral part of society, corporations have a moral obligation not only to make charitable donations, but also to incorporate philanthropic values into their operations.
(D) Integrating philanthropic activities into Company operations amounts to extortion, forcing clients to engage in an undesirable philanthropic effort. Businesses would be better off just making a charitable donation. Why they get involved in philanthropy is a more complicated question.
(E) Companies increase their commitment to philanthropy. As a result, they find themselves in more complicated relationships with their customers, shareholders and employees. Why corporations engage in philanthropy is a more complicated question.
21🇧🇷 Which of the following statements best describes the author's attitude toward the model of corporate philanthropy discussed in lines 8-13?
(A) Confident it offers companies a new and better way to be philanthropic without sacrificing profits and shareholder value
(B) certainty that it gives customers more power over their relationship with the philanthropic transaction implicit in purchasing the company's products
(C) Convinced that it offers little new in terms of the financial equation or explanation for corporate philanthropic endeavors
(D) Satisfied to be doing much better than individuals could if they were in control of funds earmarked for philanthropic causes
(E) pleased that companies have found a more creative way to allocate financial resources to philanthropy than just writing a check
22🇧🇷 Which of the following sentences would logically start a paragraph immediately after the end of the passage?
(A) Logically, we need to continue discussing how a company committed to philanthropy can really know that its suppliers are in compliance with fair trade rules and if its production processes are actually generating more waste instead of eliminating it.
(B) So the question arises, are the billions of dollars that companies donate to the common good worth it, and if so, which philanthropic model is most effective in allowing a company to reap the benefits of that contribution to maximize?
(C) It is impossible to know why a company engages in philanthropy, as companies are more complex than individuals. Rather, the question is whether the government should prevent corporations from engaging in this unprofitable activity and leave philanthropy to individuals.
(D) Therefore, companies should stick to the philanthropic model of giving money, rather than the model that hides the donation in the cost of goods sold, resulting in inflation of the price paid by the consumer.
(E) To understand why companies engage in philanthropy, we need to examine the moral and ethical policies that have been established by companies as guiding principles for employees. After all, a company is just the sum of its employees.
23🇧🇷 The relationship of the information contained in the two sentences on lines 35-42 to the information contained in lines 8-13 can best be described as follows
(A) No significant relationship because they represent two independent statements of fact
(B) the author's opinion is consistent with another opinion expressed by the author in the previous lines
(C) a hypothetical situation that clarifies a statement made by the author in the previous lines
(D) Agree in general with the previous position, but disagree in detail
(E) essentially equivalent statements, the latter being an explicit clarification of the previous lines
24🇧🇷 From the passage it can be deduced that the author was supposed to do this for a philanthropic company
(A) inform the customer that all cash is tax deductible
(B) Allow customers to opt out of paying the portion of the premium that goes to philanthropic purposes
(C) inform your customers, employees and shareholders that your philanthropy affects price, profit and shareholder value
(D) Educating the public about corporate misuse of philanthropic funds
(E) establish and publish a code of ethics for the supervision of its customers, employees and shareholders
25🇧🇷 The main purpose of the passage is
(A) question the reasons for a company's behavior, provide counter-reasons for stopping that behavior, and summarize opposing views by asking another question
(B) argue that certain corporate behavior has no moral and ethical justification within society and should be stopped
(C) Discuss the nature of corporate behavior and provide a moral reason that undermines its justification
(D) explain a trend in the company's behavior, compare it to previous similar behavior, and ask a question based on that behavior
(E) reveal how companies manipulate relationships with their customers, employees and shareholders to justify unethical behavior
PAUSE
IF YOU FINISH BEFORE TIME IS UP, CHECK YOUR WORK FOR IT
SECTION ONLY. DO NOT WORK ON ANOTHER TEST SECTION.
LSAT diagnostic test answer key
Section I
1. D
2. D
3🇧🇷 ONE
4. D
5. B
6. E
7. B
8. C
9. C
10. E
11. E
12. B
13🇧🇷 ONE
14. B
fifteen. C
16. B
17. C
18. D
19🇧🇷 ONE
20. D
21🇧🇷 ONE
22. B
23. D
24. D
25. E
26🇧🇷 ONE
Section II
1. D
2. C
3🇧🇷 ONE
4. E
5. C
6🇧🇷 ONE
7. D
8. C
9. D
10. D
11. D
12. B
13. D
14. D
fifteen🇧🇷 ONE
16. B
17🇧🇷 ONE
18. E
19🇧🇷 ONE
20. C
21. B
22. C
23. C
24. D
Section III
1. D
2🇧🇷 ONE
3. C
4. C
5. E
6. B
7. B
8🇧🇷 ONE
9. E
10. B
11🇧🇷 ONE
12. E
13. D
14🇧🇷 ONE
fifteen. B
16. E
17. E
18. D
19. C
20. D
21. D
22🇧🇷 ONE
23. C
24. C
25. E
26. B
Section IV
1. D
2. C
3. D
4. C
5. E
6. B
7. D
8. C
9. B
10. C
11🇧🇷 ONE
12. C
13. B
14. E
fifteen. C
16. B
17. C
18. D
19. E
20. B
21. C
22. B
23. E
24. C
25. D
calculate your score
Complete the table below.
your raw score
Your approximate scale score
It's impossible to say with absolute precision which raw score will translate into which scaled score on future LSATs, but here's a rough estimate.
LSAT diagnostic test answers and explanations
SECTION I
1🇧🇷 Answers:D
STEP 1: Read the question and identify your task.
This is an error question. They look for a flaw in the reasoning or something illogical in the reasoning that challenges their conclusion.
STEP 2: Read the argument in relation to your task.
The argument takes an unrestricted leap of logic. Instead of attributing the civic disorder to David Ellington, he claims that the City Council intended to undermine civic order by hiring the incompetent Ellington.
STEP 3: Find out what you are looking for.
The correct answer points to this flawed logic.
STEP 4: Read every word of each answer option.
If you read each answer carefully, certain words will help you eliminate answers. Answer A does not at all match what you are looking for. It focuses on "quantitative" results versus "qualitative" results, but the argument does not contain quantitative results, so it cannot be the right answer. Answer B doesn't work either. Yes, the conclusion is pretty general, but the "anomalous" descriptor is inaccurate. Answer C describes Ellington's incompetence as "perceived", but specific findings are discussed and do not address the conclusion of the argument. Answer D states that the error is in assuming that an action will produce a result (hiring Ellington will produce civic disorder) and that the action was taken to produce that result (hiring Ellington must produce civic disorder). This fits perfectly with your understanding of the error and appears to be the correct answer. Before making a decision, you must go through all the answers. Answer E says the error is a "repeat", but no part of the argument serves as a replay, so this can't be the right answer.The correct alternative is answer D..
2🇧🇷 Answers:D
STEP 1: Read the question and identify your task.
This is a parallel question. The question requires you to match the reasoning in the answer and the statement, so look for a similar thought pattern between the two.
STEP 2: Read the argument in relation to your task.
The reasoning speaks of a janitor making an exception based on an assessment that a police officer can protect himself because of his special abilities.
STEP 3: Find out what you are looking for.
The correct answer contains a judgment and an exception to that judgment that matches the nature of the argument.
STEP 4: Read every word of each answer option.
For answer A, the courier decides to organize his deliveries, but does not make exceptions, so this does not correspond to his paradigm and is not the right answer. Answers B and C fail their test because both the candidate and the jury are making judgments about specific areas or one person to the exclusion of all others, not the other way around, as your caretaker does. For answer D, the captain of the yacht judges that the US Navy officer has special skills that would enable him to handle an emergency situation and therefore excludes the officer from training. This is similar to your mandatory thought pattern and is probably the correct answer, but you must complete the answer check. Answer E says the manager doesn't make any exceptions and forces everyone to participate and that doesn't fit your pattern, sothe correct option is answer D.
3🇧🇷 Answers:ONE
STEP 1: Read the question and identify your task.
This is a description question. You are asked to describe the flaw in Larry and Carrie's reasoning.
STEP 2: Read the argument in relation to your task.
Larry and Carrie discuss the relationship between positive outcomes and how this quality determines whether the law is good or bad. Larry and Carrie mistakenly assume that because a law has positive outcomes it is a good law, and therefore because a law has positive outcomes such evidence is sufficient to be considered a good law.
STEP 3: Find out what you are looking for.
The correct answer will identify the common weakness in the exchange between Larry and Carrie. Pay close attention to the words of the argument - what isconsideredgood or bad of the citizens against whatIt isgood or bad. For both arguments to be valid, they must assume that a law is good when it has positive outcomes and vice versa.
STEP 4: Read every word of each answer option.
Answer A quite accurately describes Larry and Carrie's assumption that a law of a certain quality (positive results) is necessary for it to be a certain kind of law (good or bad) and that this quality is sufficient to make that kind of law. law work. be law. But let's check the rest of the answers to make sure we're paying close attention to specific words. Quality is a determinant of law type, not a "common" attribute, and they never say that quality is the "only" quality that distinguishes between the two types of law, so answer B is wrong. Your reasoning does not involve an extrapolation from "most" laws to "all" laws, so answer C is wrong. Answer D is incorrect because Larry and Carrie do not make a comparison between a "specific nation" and "all nations". Finally, answer E is wrong because they don't extrapolate a distinguishing property to mean nothing in terms of other properties of the laws.The correct alternative is answer A.
4🇧🇷 Answers:D
STEP 1: Read the question and identify your task.
This is a weak question. He tells you to pay close attention to the fundamentals of the argument and find out how a counterargument can weaken it.
STEP 2: Read the argument in relation to your task.
This is an anthropological argument asserting that there is a new belief that the "white revolution" came about because one society conquered another in a "violent and transformative" way, whereas previously it was seen as a peaceful process.
STEP 3: Find out what you are looking for.
A weakened claim would contradict or discredit this argument and prove that, in fact, what was previously thought was not a violent but a peaceful transition to a milk-based society.
STEP 4: Read every word of each answer option.
Answer A refers specifically to northern Europe and not the wider continent. Furthermore, milk may still have been part of the Middle Eastern diet even if someone else raised the cows. But more importantly, the transition in Europe may still have been violent. To answer B, just because Europeans drank milk before people arrived in the Middle East doesn't mean their society didn't undergo the later forced conversion mentioned in the argument, so this answer doesn't weaken the argument. Answer C actually reinforces the argument and proves that there was violence at the time and you are looking for a claim that weakens the argument. Answer D states that the people of the Middle East had settlements alongside hunter-gatherers and engaged in peaceful trade. Such peaceful coexistence and trade definitely undermines the argument that the transition was violent and transformative. Finally, the evidence of "sexual mixing" mentioned in Answer E does not rule out the possibility that hunter-gatherer society underwent catastrophic conversion in this way.The correct alternative is answer D.
5🇧🇷 Answers:B
STEP 1: Read the question and identify your task.
This is a weak question. You're looking for a statement that weakens the argument, so read the argument and look for critical facts or grounds that make the argument work.
STEP 2: Read the argument in relation to your task.
The argument is based on a six-year trend in which the number of young people entering the workforce is increasing, while the percentage of young people completing secondary school is increasing. Based on this, she notes that more graduates will increase youth employment.
STEP 3: Find out what you are looking for.
A weakened argument may show a caveat or alternative interpretation that will change this logical step. The correct answer will satisfy this caveat or an alternative interpretation.
STEP 4: Read every word of each answer option.
Answer A is only about the completion of college years, while the argument is about graduation and dropout rates. As these are completely different terms, it cannot be the right answer. Answer B refers that between 2000 and 2006 there was also an increase in the percentage of young people who left school early, suggesting that the increase in employment benefited equally all young people, graduates and non-graduates. This weakens the correlation between skills and employment, and therefore the inverse correlation between employment and skills. This seems to weaken the argument considerably, but let's continue. Answer C addresses jobs and their difficulties, but says nothing about the relationship between graduate or non-graduate status and employment. It is also possible that the jobs are too complicated, even for graduates. You wouldn't know because the statement says nothing about it. Answer D is trying to confuse you by using the term "count" versus "percentage" and although a greater number of dropouts were hired, there may still be a higher percentage of graduates who were hired, so this is the case, it cannot be your argument weakening. The argument discusses percentages in its premises, as well as the weakening statement. Answer E may be common practice, but it has nothing to do with the actual results of who was and was not hired.The correct answer is answer B.
6🇧🇷 Answers:E
STEP 1: Read the question and identify your task.
This is a weak question. You must find a statement that undermines the speaker's statement.
STEP 2: Read the argument in relation to your task.
The speaker talks about how companies can use public relations to cover up any wrongdoing on their part and ultimately get away with it while they wait for the "short attention span of the public."
STEP 3: Find out what you are looking for.
You are looking for a statement in your answer that contradicts or undermines this idea.
STEP 4: Read every word of each answer option.
Answer A can be interpreted as reinforcing the argument that a company can become too big to fail despite being socially irresponsible. Answer B essentially repeats the argument in simpler terms, and speaks in the past tense and does not address what might be the case in the future. Answer C also repeats the argument, but with the caveat of faster and slower growth between companies. Answer D adds state institutions to the argument, which does not weaken it. Answer E states that public relations is often overlooked by the public and fails to mask a company's irresponsible actions. While this may not address the short attention span part of the argument, it weakens the argument and qualifies it much better than the other answers.The correct alternative is answer E.
7🇧🇷 Answers:B
STEP 1: Read the question and identify your task.
This is a variation of a descriptive question – a descriptive question of how the argument proceeds. It asks you to find the answer that describes how the argument "proceeds", that is, how it arrives at its conclusion.
STEP 2: Read the argument in relation to your task.
Based on the experience of "several" amateur photographers with a given camera, the argument urges "any" (= all) such photographers to choose that camera.
STEP 3: Find out what you are looking for.
The correct answer will describe how the argument applies the claims of a small group (the amateur photographers with the Apheron camera) to make a proposition to a larger group (all amateur photographers).
STEP 4: Read every word of each answer option.
Answer A talks about test conditions, while the argument doesn't mention any test conditions, so it's wrong. Answer B states that the argument uses a subset to make a recommendation to a larger group that meets your criteria. Amateur photographers are the subgroup and "any" photographers are its larger group. With answer C, the credibility of the amateur photographer is taken for granted and is not part of the argument. For answer D, the rationale for buying the camera is never discussed in the argument. The experience of the smaller group of photographers is not placed in a larger context. It is only used to recommend the camera to a larger group, so answer E is wrong.The correct answer is answer B.
8🇧🇷 Answers:C
STEP 1: Read the question and identify your task.
This is a weak question. Challenges you to find an answer that undermines the argument.
STEP 2: Read the argument in relation to your task.
Check out the basis for camera recommendation, in this case, despite its lower resolution, the camera offers finer details than Norwich's other competing cameras.
STEP 3: Find out what you are looking for.
The correct answer casts doubt on the relationship between the premises and the conclusion.
STEP 4: Read every word of each answer option.
For answer A, lens composition does not change the results discussed in the argument, so the basis of the recommendation is not weakened. Response B is not significantly different from response A, except in terms of sensor size rather than lens substance. For Answer C, the criterion under which the argument makes its recommendation is the finest detail the camera can achieve in both bright and low light. If there are other criteria for amateur photographers when it comes to a camera then the argument is definitely mitigated and this seems to be your correct answer. Answer D describes conditions under which test results appear more reliable than weak. Answer E may weaken another argument, one against digital cameras, but it does not weaken the recommendation that if photographers are considering a digital camera they should choose Apheron over others.The correct alternative is answer C.
9🇧🇷 Answers:C
STEP 1: Read the question and identify your task.
This is a description question. It asks you to describe how a specific fact is used within the argument.
STEP 2: Read the argument in relation to your task.
As you read the argument, pay close attention to this fact and how it works. In this case, the fact that automakers are buying more equipment and spending more on R&D seems to be a justification or evidence that supports the conclusion of the argument. This argument actually has the conclusion in the first sentence.
STEP 3: Find out what you are looking for.
The correct answer discusses the fact as justification or evidence.
STEP 4: Read every word of each answer option.
Answer A asserts that the fact is an inference from a premise when, in fact, it is just the opposite. The conclusion from the considered facts is that companies are expecting an increase in sales. Answer B is wrong for the same reason; Its fact is not a conclusion, but a premise or basis for conclusions. Answer C states that the fact is the main evidence for the conclusion and this seems to be your answer because based on the considered fact, as you expressed it from the reading of the question and the argument, the economist concludes that the industry is on the way to recovery . Answer D states that the fact is necessary for the conclusion to be true, but the conclusion may be true based on other facts that you don't have at hand, so this is not the function of the facts considered. After all, the argument doesn't explain why companies are spending more. It's just making a judgment based on that fact, so answer E can't be correct.The correct alternative is answer C.
10🇧🇷 Answers:E
STEP 1: Read the question and identify your task.
This is a parallel question. It asks you to find an argument that follows the same logical pattern.
STEP 2: Read the argument in relation to your task.
The argument made is that since there is a directly proportional relationship between greater water use and greater heat index (increase leads to increase), then as the average heat index is four points higher, more water is used.
STEP 3: Find out what you are looking for.
You need to look for a similar relationship in the answers.
STEP 4: Read every word of each answer option.
Answer A establishes two proportionalities (doctors for services, doctors for patients) in relation to the first statement (doctors). It then tries to establish a relationship between the two conditionals (services to patients). This is a much more complicated relationship than you're looking for, so it's not your answer. Answer B starts with the correct proportionality between two terms (doctors and nurses), but then adds an independent statement about nurses and draws an unsustainable relationship between doctors and nurses, so it is not your answer either. Answer C starts well with a directly proportional relationship, higher account for more health professionals seen, but the conclusion is based on the number of patients and not on the number of health professionals that the patients see during the consultation, which represents a breach of that premise. Therefore, answer C cannot be your answer. Answer D makes a recommendation based on the premise but does not reach a conclusion. Answer E describes a directly proportional relationship. If there are more patients, more painkillers will be prescribed, and as the number of patients has increased by 15% this year, the number of painkillers is also expected to increase. Therefore,The correct alternative is answer E.
11🇧🇷 Answers:E
STEP 1: Read the question and identify your task.
This is a description question. The question asks you to describe which reasoning method Zola uses to answer Jason.
STEP 2: Read the argument in relation to your task.
If you read her arguments, you'll find that Zola argues that the long cooking time and ample sunlight required to operate the solar cooker are issues, which means that the solar cooker is not such a perfect answer to "a plethora of problems in developing countries". She thinks the arguments on which the conclusion is based are sound and logical, but she thinks the conclusion doesn't take other factors into account.
STEP 3: Find out what you are looking for.
You'll look for something similar in your answer choices.
STEP 4: Read every word of each answer option.
Zola actually agrees with Jason's arguments and doesn't think they are "wrong", so answer A can't be right. Although Zola disagrees with Jason's conclusion, her problem is specific to the stove and not her inadequate definition of problems in developing countries, so Answer B cannot be correct. Answer C has the same problem as answer A. It does not contradict your arguments or your basis. With answer D, Zola doesn't reinterpret anything, so this answer can't be right. Zola makes caveats that undermine several of Jason's assumptions, and that's what you'll find in answer E.The correct alternative is answer E.
12🇧🇷 Answers:B
STEP 1: Read the question and identify your task.
This is a parallel issue. It asks you to look for a series of statements that follow a logical pattern similar to the one given in the passage.
STEP 2: Read the argument in relation to your task.
In this case, the argument is that a lumber company only sells two types of wood, pine and oak, and Joe never uses pine to build tables. Therefore, if he builds a table for Marion, it must be oak. the keywordOh nolimits Joe's choices.
STEP 3: Find out what you are looking for.
You need to look for a similar pattern in the answers.
STEP 4: Read every word of each answer option.
In Answer A, there is no definitive "single" statement that limits Rocky's choices and determines the conclusion. He just judges which car works for him. Answer B shows the correct pattern. Roddick Corporation offers Pedzisai only two options, Sweden and South Africa. Pedzisai would never choose South Africa if he could choose between the two. Therefore, when choosing a bet, he will choose Sweden. The wordOh noit is used almost equally as a limiting and determining factor in the final statement. Answer C actually sets out conditions under which Georgia will not choose any of the available options. Answer D has a conclusion that has nothing to do with choosing between a break and a longer lunch. Rather, whether or not to cut hair has to do with a third element. The pattern is not the same. For answer E, the owner has an equally bad choice, and the conclusion gives no indication of which choice was made. Rather, it merely outlines the conditions, whichever the owner chooses, unless, of course, a third alternative presents itself.The correct answer is answer B.
13🇧🇷 Answers:ONE
STEP 1: Read the question and identify your task.
This is a parallel question. This question challenges you to find the answer whose logic most closely matches the given statements. The fact that the reasoning is "questionable" is irrelevant to your task of just finding the answer that follows the same logic.
STEP 2: Read the argument in relation to your task.
In this case, a building inspector was charged with the crime, the evidence was lost or destroyed in a lawsuit, and therefore the argument recommends that the charges be dropped.
STEP 3: Find out what you are looking for.
Again, that the reasoning is "questionable" is irrelevant to your task of just finding the answer that follows the same logic, questionable or not.
STEP 4: Read every word of each answer option.
Answer A reverses the statements and gives the recommendation in the second statement, but the logic is essentially the same. The computer scientist has been accused of a crime and the charges must be dropped as all evidence of his crime (hard drives, recordings) has been removed. Answer B recommends that the teacher be investigated further and the evidence is still available, so this situation is very different from your inspector. Answer C suggests itself, but it is still not the best answer because the lack of evidence described is insufficient. People may have heard the speech and be familiar with its content. Answer D argues to support the claim, which is supported by some evidence, which is an entirely different situation. Answer E argues that the charges should be dropped due to the lack of identity of the accuser and not due to insufficient evidence against the financial manager.The correct alternative is answer A.
14🇧🇷 Answers:B
STEP 1: Read the question and identify your task.
This is an acceptance question. The question asks which answer makes the argument's logic successful.
STEP 2: Read the argument in relation to your task.
Despite the claim that the second statement "follows" from the first, the argument does not.
STEP 3: Find out what you are looking for.
So you need to find the assumption that the conclusion works when inserted between the first and second statements.
STEP 4: Read every word of each answer option.
Answer A ignores the condition of having a job or not and therefore does not help with the conclusion. Answer B seems to fit your needs. Since every employee who uses public transport has lunch in the company's break room, it follows that if some employees who eat in the company's break room have an office (as indicated in answer B), there are some employees who do not have an office Eat in the room of rest, so don't use public transport. Answer C leads to all employees with an office using public transport, which contradicts the conclusion. Answer D does not lead to any conclusions about who has an office or whether or not they use public transport. Answer E says something about who eats in the break room, but nothing about who has an office or whether or not they use public transportation. Therefore,The correct alternative is answer B.
fifteen🇧🇷 Answers:C
STEP 1: Read the question and identify your task.
This is a weak question. The question asks you to find the statement among the answers that most weakens the argument.
STEP 2: Read the argument in relation to your task.
You must read the argument and discover the central basis for your conclusion, and then articulate what kind of response will undermine that basis. In this case, the argument reveals a discovery that leads archaeologists to believe that a larger and more permanent civilization once existed in the Brazilian Amazon.
STEP 3: Find out what you are looking for.
A weakened statement could indicate that a smaller, less enduring civilization was responsible for the engravings.
STEP 4: Read every word of each answer option.
Answer A uses words likegrandeeestablishedto support the conclusion that there was a well-developed civilization in the Amazon, then this cannot be the correct answer. Answer B offers irrelevant facts and no insight into the nomadic or permanent nature of civilization. It just refers to the sophistication of each form. Answer C reveals a discovery of similar engravings 100 miles away by a "nomadic" artist. It can be deduced that the artist was a member of a nomadic civilization, as was originally thought before the discovery. This weakens the argument and seems to be his answer. Regarding answer D, the argument does not assert that the civilizations in question did not exist prior to the 2000-year flood window, nor does this answer address the nomadic or permanent nature of the civilization. Answer E is interesting, but it doesn't weaken the argument as well as answer C. While the content of the engravings is relevant, they don't necessarily prove the nature of the civilization, as they could be representations of earlier civilizations or other civilizations.The correct alternative is answer C.
16🇧🇷 Answers:B
STEP 1: Read the question and identify your task.
This is an acceptance question. He asks you to find a statement on which the logic of the passage depends.
STEP 2: Read the argument in relation to your task.
The bottom line is that people in developed countries with more options "in all aspects of their lives" are no happier than people in less developed countries with fewer options.
STEP 3: Find out what you are looking for.
The conclusion is extrapolated from television to all aspects of people's lives, so you'd expect the correct answer to support the conclusion and, most likely, the extrapolation.
STEP 4: Read every word of each answer option.
Answer A focuses only on the television aspect of daily life and not on other aspects of their lives. It also softens the comparison of nations a bit. Answer B claims that an inverse relationship – that more channels means less happiness – also exists between “other categories”. If the same relationship exists between other goods and services regardless of other factors related to where people live in the world, then people in countries with more options are less happy than people in countries with fewer options. So answer B seems to strongly support the argument and is your answer. For answer C, the argument says nothing about conscience among nations, only that the number of choices determines happiness. For Answer D, how each nation judges the decisions of the others is irrelevant to its own experience. Like Answer A, Answer E deals only with the aspect of TV in everyday life.The correct answer is answer B.
17🇧🇷 Answers:C
STEP 1: Read the question and identify your task.
This is a paradoxical question. As you read the question, you realize that it wants you to find the only answer it needs.NoExplain the phenomenon described in the argument.
STEP 2: Read the argument in relation to your task.
During the scuffle, you discover that the cheerleading squad had below-normal sales for the bake sale after deciding to cut prices.
STEP 3: Find out what you are looking for.
So you need to come up with an answer that does nothing to explain the drop in sales.
STEP 4: Read every word of each answer option.
For answer A, a smaller number of potential buyers would likely hurt sales. For Answer B, competing fundraisers would likely reduce the funds students spend on baked goods. For answer C, the cost of sugar and flour can affect profitability, but it has nothing to do with sales. Since answer C doesn't help make up for the apparent discrepancy, this appears to be your answer, but you should keep checking the rest of your answers to be sure. For answer D, students would probably not buy more baked goods if they already got free cupcakes from teachers, which would definitely hurt sales. Finally, for answer E, location can very likely have an impact on sales, especially if students have already assumed that their counter will be in a traditional location.The correct alternative is answer C.
18🇧🇷 Answers:D
STEP 1: Read the question and identify your task.
This is an issue of empowerment. It prompts you to find the policy that matches the situation.
STEP 2: Read the argument in relation to your task.
In that case, you need a policy that allows the community center to use the additional resources as it sees fit, but also respects the donors' interests.
STEP 3: Find out what you are looking for.
You'll look for something similar in your answer choices.
STEP 4: Read every word of each answer option.
Answer A gives the center complete freedom to do whatever it wants with all the funds, but ignores the interests of the donors. Answer B states that the center cannot do anything with the money except what the donors approve, which ties the center's hands and gives the donors a lot of power. Answer C paralyzes the center by saying that directors must pre-approve the purpose of all expenditures, but it does not address overfunding and is therefore useless in the situation described in the argument. Answer D states that the center should use the money for its intended purpose, unless those funds cannot be used for that purpose, in which case donors should be consulted. This policy fits the situation perfectly. It allows the center to use the money as it sees fit until allocations are met. So they have to consult donors. Answer E requires the parent company to return the money, which is completely out of line with the situation. Because of this,the correct option is answer D.
19🇧🇷 Answers:ONE
STEP 1: Read the question and identify your task.
This is an issue of empowerment. He challenges you to find the answer that least weakens the argument, or in other words, most strengthens it.
STEP 2: Read the argument in relation to your task.
The scientist argues that his colleagues' statements that their theories are "based on pure conjecture and have no experimental basis" are incorrect because their theories are based on current and reputable data. Although the scientist admits that he has not read "all the details" of the information, he trusts the sources.
STEP 3: Find out what you are looking for.
The correct answer is a criticism that has little or no impact on discrediting the argument. When reviewing your answer choices, a process of elimination works best in this situation.
STEP 4: Read every word of each answer option.
Answer A describes the basis of the conclusion as "uncertain memories", although the memory does not seem uncertain at all. It refers to a specific study and use of the same research facilities. There is nothing uncertain about these memories. Answer A seems like a good option. Assuming answer B, if the experiments are influenced by human error, then such a criticism would definitely weaken the scientist's argument that the research supports the theory. Since the scientist argues that experiments conducted by research organizations are all that is needed to support his theory, Answer C would seriously undermine the argument by saying that more support is needed. Answer D questions the quality of the research, and since the scientist uses these studies as the only support for the theory, this criticism definitely undermines the scientist's argument. Finally, when the logic of using laboratory results to support a theory does not work for the current theory, the scientist's argument is again weakened. Because answers B, C, D, and E make the scientist's argument susceptible to closer scrutiny, they can be eliminated, which meansAnswer A is the correct alternative.
20🇧🇷 Answers:D
STEP 1: Read the question and identify your task.
This is an acceptance question. For this first question, you need to find the assumption that the argument depends on to be true.
STEP 2: Read the argument in relation to your task.
The dispute is over annual testing of lead content in drinking water. The conclusion is that the water is drinkable, although some samples exceeded federal standards because the levels were still too low to pose a risk.
STEP 3: Find out what you are looking for.
The correct answer addresses the relationship between federal standards and the amount of lead that causes health problems.
STEP 4: Read every word of each answer option.
Answer A states that the standards are very strict, but lead levels can still be harmful to your health, and whether or not you break the guidelines, you are taking a risk by drinking the water. Answer B seems to weaken rather than support the argument, and that's not the point. Answer C questions the test method and leaves open the option that the situation could be worse than stated, possibly leading to the opposite recommendation that people should not drink the water. For Answer D, if lead levels slightly above federal standards are still not harmful, then it's fair to say that the water is still safe for city residents to drink. This greatly supports the argument and is crucial to the conclusion of the argument. Answer E has nothing to do with the lead content or drinking the water. Instead, there is an irrelevant observation about human behavior. Because of this,Answer D is the correct alternative.
21🇧🇷 Answers:ONE
STEP 1: Read the question and identify your task.
This is an issue of empowerment. This question challenges you to find the answer that actually supports the claim of the argument.
STEP 2: Read the argument in relation to your task.
The dispute is over annual testing of lead content in drinking water. The conclusion is that the water is drinkable, although some samples exceeded federal standards because the levels were still too low to pose a risk.
STEP 3: Find out what you are looking for.
The correct answer best supports the conclusion that the water is potable.
STEP 4: Read every word of each answer option.
Answer A states that most lead never makes it to the faucet because it is absorbed by the lining of the pipes, reducing the risk of lead exposure and injury. This seems to add great force to the argument and is probably the correct answer. Answer B raises questions about test methods but does not address actual results. Answer C states that children exposed to lead levels above federal standards will be harmed, the opposite of what you are looking for. Answer D is anecdotal at best, as the hospital deals with people who drink large amounts of water, not the normal amount. Answer E, while indicating a problem with current federal regulations, does not indicate what basis the medical associations are arguing, and it is still not as strong an answer as A.Answer A is the correct alternative.
22🇧🇷 Answers:B
STEP 1: Read the question and identify your task.
This is an acceptance question. The first question asks you to find the single answer on which Eleanor's argument is based. The wordBasedit requires that the statement be central for the argument to withstand scrutiny.
STEP 2: Read the argument in relation to your task.
The subject of this conversation between Jaime and Eleanor is the lifespan of elephants in captivity. Jaime's conclusion is based on studies showing that elephants living in two wildlife sanctuaries in Kenya and Myanmar live longer than elephants living in zoos. Eleanor's point is that zoos are frowned upon because elephants in parks don't face the same dangers as elephants in the wild, and that the studies Jaime cites about zoos are out of date.
STEP 3: Find out what you are looking for.
Eleanor postulates that poaching and unnatural hazards are the dominant threats to animals outside protected areas. The correct answer will resolve this issue.
STEP 4: Read every word of each answer option.
Answer A may be true about the genetic differences between African and Asian elephants, but it says nothing about what happens when either type is kept in a zoo rather than an unprotected reserve. Answer B states that predators and disease affect an elephant's lifespan to a lesser extent than other threats outside protected parks. This seems central to Eleanor's argument, as she suggests that the threat of poaching and unnatural dangers affects elephant lifespans far more than any other threat in protected parks. Answer C contradicts the second part of Eleanor's argument and is therefore incorrect. Answer D says why an elephant's lifespan was shortened in a zoo in the 1970s, but nothing to support Eleanor's claim that care is better today. It is also claimed that the worst offer at that time had budgetary reasons. Eleanor points out that the science of nursing has advanced, not funding. Answer E actually contradicts Eleanor's argument that elephants in reserves are better protected from poachers and unnatural threats.The correct answer is answer B.
23🇧🇷 Answers:D
STEP 1: Read the question and identify your task.
This is a description question. He challenges you to find the answer that accurately describes the reasoning technique Eleanor uses to defend her point of view.
STEP 2: Read the argument in relation to your task.
The subject of this conversation between Jaime and Eleanor is the lifespan of elephants in captivity. Jaime's conclusion is based on studies showing that elephants living in two wildlife sanctuaries in Kenya and Myanmar live longer than elephants living in zoos. Eleanor's point is (1) that zoos appear in a bad light because elephants in parks do not face the same dangers as elephants in the wild, and (2) that the studies Jaime cites about zoos are out of date.
STEP 3: Find out what you are looking for.
Eleanor dismisses the assumptions Jaime uses to reach his conclusion - namely, the link between living in a reserve park and a longer life - and the studies he cites, which she says are out of date.
STEP 4: Read every word of each answer option.
Regarding answer A, Eleanor doesn't offer such an example, so it can't be your answer. For Answer B, Eleanor does not question the conclusion, only how Jaime arrived at it. Furthermore, she does not question the accuracy of her data, only her choice of data population. Answer C states that Eleanor is trying to support Jaime's argument, which is definitely not the case. Answer D states that Eleanor rejects the correlation between the zoos and sanctuaries that Jaime chose for his argument and that altering her dataset will affect Jaime's supportive statements that led to her conclusion. These two seem to be exactly what Eleanor is doing with her reasoning. Answer E is partially correct because it challenges her assumptions, but it does not track a broader sample of data, but an entirely different sample of data. Because of this,Answer D is the correct alternative.
24🇧🇷 Answers:D
STEP 1: Read the question and identify your task.
This is a weak question. You must find the answer that most weakens the argument.
STEP 2: Read the argument in relation to your task.
On what basis does the teacher argue? In this case, based on an experiment with 10 of her students, the teacher concludes that all of her students would do math better manually than with machines.
STEP 3: Find out what you are looking for.
Most likely, the weakening statement will harm the experiment in some way.
STEP 4: Read every word of each answer option.
Answer A actually strengthens the teacher's thinking by making student performance even more impressive, so this option does not qualify. Answer B seems to indicate that the 10 students received help that may have improved their performance. This may weaken the argument, but it does not indicate whether the other students also received help; so this option might not be as strong as another answer. Answer C only indicates how the 10 students compare to other students, it does not affect the judgment that they have improved on their own past performance. Thus, the argument is not weakened. Answer D states that the students undergoing the experiment have seen the material before, while the other students have not. This indicates that they had an advantage that improved their performance, and the lack of a calculator may have had nothing to do with it. This greatly impairs the teacher's thinking and may be their strongest candidate. Answer E only indicates that the teacher gave them some methodology to work without a calculator, but this does not mean that such training will improve their performance and will not necessarily affect the teacher's reasoning.Answer D is the correct alternative.
25🇧🇷 Answers:E
STEP 1: Read the question and identify your task.
This is an error question. The question is looking for a description of the logical error rather than the error itself.
STEP 2: Read the argument in relation to your task.
The argument concludes that there are more employers who believe in one idea (those accused of crimes should be fired) than in another (only people convicted of crimes should be fired), where the result of a recent survey is used.
STEP 3: Find out what you are looking for.
The correct answer describes why the stats are misleading, specifically why one or both of the search stats might double or understate if someone thinks a person should be fired in both cases.
STEP 4: Read every word of each answer option.
Answer A is a true description of what the statement does, but it doesn't necessarily describe an error. Most surveys extrapolate a sample to draw a conclusion about the general population. For Answer B, no ambiguous term is used in the supporting statements, nor is such term used as a basis for the conclusion, so it cannot be correct. Regarding Answer C, the conclusion compares the two beliefs mentioned in the supporting statements and does not mention a third belief. For answer D, there is no doubt that all premises can be true. Concluding on Answer E: To be convicted, one must be charged, so more employers are likely to believe that an employee should be fired if convicted of a crime than those who believe the employee should be fired if only accused of a crime. A sufficient condition is confused with a necessary condition. So research and reasoning are flawed.The correct alternative is answer E.
26🇧🇷 Answers:ONE
STEP 1: Read the question and identify your task.
This is an error question. It asks for the error in a complete sentence form.
STEP 2: Read the argument in relation to your task.
Find the logical flaw that leads the editor to believe that politicians will divert funds from education.
STEP 3: Find out what you are looking for.
The statement asserts that since politicians in general have betrayed the purpose of the lottery, the current legislature will do the same. You'll look for something similar in your answer choices.
STEP 4: Read every word of each answer option.
Answer A claims that the argument draws a conclusion about a specific population (“our representatives in the state legislature”) based on a study of a larger population (“politicians”). This seems to perfectly describe the editor's error, but you should read the rest of the answers to be sure. Answer B might sound good, but the statements don't mention any historical data. He just makes general historical claims. Answer A remains the strongest answer. Answer C is wrong because you don't know the historical or current conditions, but the current option on the table, a public lottery to support education, is still the same. Both incumbent politicians and the current legislature are considering the same solution, so answer D cannot be correct. Neither statement attacks the fans or the merits of the lottery, only the resulting misconduct once passed, so answer E may not be the right choice.The correct alternative is answer A.
SECTION II
1🇧🇷 Answers:D
STEP 1: Read the question and identify your task.
This is a final question. She asks you to find a logical conclusion from the answers that can be drawn from the Chancellor's reasoning.
STEP 2: Read the argument in relation to your task.
So you read the Chancellor's statement with the expectation that it will lead you to an inevitable conclusion or assumption. Simply put, the dean argues that the school needs to invest in new facilities to attract new students and beat the competition.
STEP 3: Find out what you are looking for.
The correct answer sums up the argument. In this case, the Chancellor argues that Mayfield Academy must grow to survive and that building new facilities is one way to generate that growth. So you must jump to the conclusion that Mayfield Academy needs to invest in new facilities in order to grow.
STEP 4: Read every word of each answer option.
Answer A seems to work, but it only mentions attracting more students and not how this should be achieved. Answer B seems to be something parents can say if or if the school does in fact agree and implement the details of the dean's recommendation, but that's not exactly a conclusion to be drawn from the argument. Answer C addresses a parental consideration and something unrelated to the school's need to attract more students. Answer D states that the academy should invest in new facilities, a conclusion the school could very well reach based on the dean's reasoning. This appears to be your answer, but you should check the final answer to be sure. Similar to answer A, answer E indicates what the school would like to see as the parents' inevitable decision to transfer their children to Mayfield, but does not discuss how Mayfield Academy might achieve that outcome, which is the subject of the argument.The correct alternative is answer D.
2🇧🇷 Answers:C
STEP 1: Read the question and identify your task.
This is a description question. The question asks you to describe the method the ethics expert uses to respond to the manager's argument
STEP 2: Read the argument in relation to your task.
In essence, the CEO argues that teaching employees ethics is a waste of time because they will inevitably act unethically.
STEP 3: Find out what you are looking for.
First, by the tone of voice, you can tell that the ethicist is indicating disagreement with the executive ("makes as much sense as...") and that the ethicist is using an analogous situation ("spending money on driver education") and that the analogous situation is for the ethicist who considers an absurd justification (“all drivers inevitably cause an accident”) to be an equally bad result. You'll look for something similar in your answer choices.
STEP 4: Read every word of each answer option.
Answer A seems to describe the thinking of the CEO and not the ethics expert. Your question is about the method of the ethics expert, not the executive. With answer B, the ethicist is not attacking the leader's character, just his reasoning, so this is not the correct answer. For answer C, the ethics expert uses a different (analog) situation to show that the executive's reasoning is flawed and would lead to a bad outcome, so this answer seems like your best option, but let's continue with the rest of the options. Answer D does not seem correct because the ethics expert does not ask for more evidence. For answer E, the ethics expert believes that there is no dilemma to explain. There is no dilemma for the ethicist, as you can see from the use of an analogous situation, which is entirely black and white.The correct alternative is answer C.
3🇧🇷 Answers:ONE
STEP 1: Read the question and identify your task.
This is an acceptance question. You must describe the assumption on which the argument depends.
STEP 2: Read the argument in relation to your task.
In this case, it is argued that, despite his qualifications and due to certain disqualifications, Renfield is not qualified for a managerial position.
STEP 3: Find out what you are looking for.
The correct answer will likely discuss qualifications or disqualifications as they are the basis for concluding that Renfield should be a manager.
STEP 4: Read every word of each answer option.
Answer A describes the acceptance of the argument related to Renfield's only qualification that 'his performance as a member of his team, although exemplary', does not by itself prove that the person can be a manager. This is definitely an assumption the arguer is arguing about and is very likely to be your answer, but you should read the rest of the answers to be sure. Answer B claims that even in his regular roles, Renfield is unreliable, which actually contradicts the argument that his performance was "exemplary". Regarding Answer C, nothing in the statement is based on what interests Renfield will be represented when he is manager. Rather, it's based on your managerial skills. Answer D addresses the behavior of the team and not Renfield, so it is not appropriate for your answer. Finally, answer E is a very general statement about the organization's general practices. It says nothing about the specifics of Renfield's case, so it can't be your answer.The correct alternative is answer A.
4🇧🇷 Answers:E
STEP 1: Read the question and identify your task.
This is an error question. You need to figure out why the argument is wrong, but he structures the question in full sentence form, so the answer will likely be a description of the logical error rather than the error itself.
STEP 2: Read the argument in relation to your task.
In this case, the statement makes a simple point - breaking accepted norms is the only way to make real progress.
STEP 3: Find out what you are looking for.
The argument singles out one specific influence on the issue of real progress - the overthrow of accepted norms - and does not consider the possibility that another factor might have an impact. You'll look for something similar in your answer choices.
STEP 4: Read every word of each answer option.
Answer A seems to describe an entirely different argument, since the argument does not undermine its own premise and the notionAttributeseems to be a rather inaccurate description of what is being discussed. Answer B may seem like the correct answer to the question, but the problematic word iswellsince no causal relationship is offered between the first claim and the second claim. Answer C is problematic with the first word,nega, because the argument actually confirms, not denies, such a contribution. Answer D doesn't work because the argument doesn't isolate your observation to a specific point in time. It's more generic than that. Answer E is your last remaining option. The argument makes the mistake of saying that one influence trumps all other influences in progress.The correct alternative is answer E.
5🇧🇷 Answers:C
STEP 1: Read the question and identify your task.
This is a variation of a descriptive question – a descriptive question of how the argument proceeds. The question is in full sentence form, and so the answer is a description of how the argument is formed, not necessarily anything specific within the argument.
STEP 2: Read the argument in relation to your task.
The educator outlines what critics say about online universities and low residency degrees, then discusses how Plymouth Online is the exception to these objections.
STEP 3: Find out what you are looking for.
You'll look for something similar in your answer choices.
STEP 4: Read every word of each answer option.
Answer A is incorrect because the educator does not reference any scientific authority during the challenge. Answer B is incorrect because the educator does not actually question the validity of the critics' objections. The educator merely points out how Plymouth Online overcomes these problems. Answer C may be your answer because the educator uses Plymouth Online as a counterexample to what scholars say are prevailing beliefs about distance learning programs. You'll need to check the rest of the answers to make sure you're done. Answer D is wrong because the termsociologicalis an inaccurate description of the educator's discussion. The educator makes no sociological observations about why remote programs are used. Also, this answer doesn't mention using the teacher's own program as an example. Finally, Answer E is not accurate because the educator is not comparing two approaches. Furthermore, this response ignores the dynamic between critical scholars and educator discussion of Plymouth Online.The correct alternative is answer C.
6🇧🇷 Answers:ONE
STEP 1: Read the question and identify your task.
This is a weak question. You must find the statement among the answers that weakens the "harder" argument.
STEP 2: Read the argument in relation to your task.
The statement argues that the government should mandate strength training in public schools for all children between the ages of 6 and 18, based on a new study.
STEP 3: Find out what you are looking for.
They expect the correct answer to be a statement that seriously undermines some aspects of this recommendation to the government.
STEP 4: Read every word of each answer option.
Answer A seems to be your most likely answer, as it states that a school-based program is ineffective and that a home-based program is necessary for real success. You need to read the rest of the answers to be sure. Answer B undermines the ability to measure progress, but does not undermine the usefulness of the program or recommendation. Answer C adds facts about the study, but the details are not the type that would detract from the study's findings. Answer D only addresses the time required to see measurable results, which does not change the fact that this strength training is beneficial and therefore does not invalidate the argument. Finally, the answer E might give you food for thought. Yes, strength training at such a young age can pose some health risks, and adding these tests to the study could have been helpful, but the results could have been positive as well. You don't have enough information to know that, and so this answer doesn't weaken the argument any more than answer A.Answer A is the correct alternative.
7🇧🇷 Answers:D
STEP 1: Read the question and identify your task.
This is a weak question. This question calls for a statement that weakens the argument.
STEP 2: Read the argument in relation to your task.
The statement argues that, according to a study, all people with type 2 diabetes should do aerobics and weight lifting. Key terms are the health issue type 2 diabetes and the recommendation for both types of exercise.
STEP 3: Find out what you are looking for.
The correct answer casts doubt on the relationship between the premises and the conclusion.
STEP 4: Read every word of each answer option.
Answer A sounds good, but just because people with the condition are "less likely" to deal with it doesn't mean they can't or that it doesn't benefit them, the study shows. Answer B is a preposterous statement because even if they can't be done simultaneously, that doesn't mean a training program can't be designed to alternate between them. Also, the statement makes no mention of type 2 diabetes. Answer C sounds good, but pay close attention to words that hurt your chances. Claims that physical demands "could" have negative physical effects. This is speculative and contradicts the study cited in the argument. If the answer stated that another study suggested such a disadvantage, that answer would qualify as your choice. Answer D is a strong candidate because if some people with type 2 diabetes only need one of the activities to get the same benefit, the recommendation for both activities is watered down. Finally, Answer E leaves no doubt as to the relationship between the premises and the conclusion; In fact, it provides support for the study's conclusions.The correct alternative is answer D.
8🇧🇷 Answers:C
STEP 1: Read the question and identify your task.
This is a deductive question. It asks you to find a statement that must be true based on the given statements.
STEP 2: Read the argument in relation to your task.
The argument offers a paleontological discussion of the survival of large mammals such as hippos and giraffes after the extinction of the dinosaurs.
STEP 3: Find out what you are looking for.
The correct answer will most likely be a conclusion you can draw based on how that survival took place, particularly the competition for food and shelter.
STEP 4: Read every word of each answer option.
Answer A tries to link a later date of extinction to the smaller size of the surviving mammals, but nothing in the argument leads you to believe that there is a relationship between the two, just that their size changed when that happened. One can only assume that the size increase would have been delayed. Regarding Answer B, the statements do not indicate a size relationship between large mammals such as the hippopotamus and other mammals. He talks only about large mammals and how they increased in size. Answer C is a very good option for your question. The statements discuss how large mammals increased in size because they no longer had to compete with dinosaurs for space and food. So you can assume that if the larger non-mammals had survived the extinction event, they would have been competing for space and food, and the larger mammals would have been smaller. Answer D may be true, but nothing in the statements leads you to believe that the size of dinosaurs had anything to do with their extinction. Answer E says something that cannot be verified by the statements. It is possible that they competed with carnivorous dinosaurs and the same evolutionary process would have occurred.Answer C is the correct option.
9🇧🇷 Answers:D
STEP 1: Read the question and identify your task.
This is a parallel question. He asks him to find among the answers a situation “parallel” to the one given.
STEP 2: Read the argument in relation to your task.
The given situation is that of an auction that Jody does not attend; because no one wants to buy the Giacometti statue more than she does, so "no one person" will bid on the statue no matter how low the bid starts.
STEP 3: Find out what you are looking for.
Most likely, the correct answer will use extreme all-or-nothing like terms like "most", "nobody", or "not even one person", creating an equally absurd notion. The correct answer will have equally flawed logic.
STEP 4: Read every word of each answer option.
Answer A starts off well and says that one jockey wants something "more", but then backs off on the statement saying the other jockeys will just redouble their efforts. This is not the all-or-nothing statement you need. Answer B begins by saying that Larry is best qualified to detect a counterfeit, but the rest of the statement is wrong because he says that since one item is faultless, the others must be counterfeits. This is not an all-or-nothing result of the first statement. Answer C does not say that Professor Ricardo is the only one who can translate the text. It leaves open the possibility that there are others, it's definitely not an all-or-nothing outcome. In answer D, Emilio has the "strongest" intention to get the sales job, and since he doesn't apply, no one else will apply, regardless of salary. You have your extreme outcome words ("more") and all or nothing ("nobody else"), and the statement is also logically flawed. This appears to be your answer, but you need to read your last option. For answer E, Sherry is not particularly motivated to join the group, and her timing conflicts only result in a gradually deteriorating situation, not an all-or-nothing situation.The correct option is answer D.
10🇧🇷 Answers:D
STEP 1: Read the question and identify your task.
This is a deductive question. It asks you to find among the answers the one statement that must be true, provided the statements in the passage are true.
STEP 2: Read the argument in relation to your task.
The argument provides a set of conditionals, one for the conductor, one for the orchestra, and one that ties them together. Use shorthand to note these conditions: if C not NY then other competitions. If O is not NY, then there are no contests. Finally C or O not NY. You can conclude from the last statement that if C yes NY then O no NY, or if O yes NY then C no NY.
STEP 3: Find out what you are looking for.
The correct answer will likely test your understanding of how these terms interact. When you start reading the responses, you realize that they are a series of "if...then" statements. You must try the same shorthand with everyone to evaluate them.
STEP 4: Read every word of each answer option.
Answer A is O yes NY, so no contests. This completely contradicts your second condition, so this is not your answer. Answer B cannot be abbreviated because it is a probability ("most likely") and your provided conditions are not based on probabilities. They are certainties, so this cannot be your answer. Answer C cannot be abbreviated either, as it is a recommendation and not a statement. The wordhe mustis the key to identifying the problem with this option. Answer D says C yes NY so no contests. You know that if C is NY then O is not NY. So if O is not NY, there are no contests. This seems to be your answer. Answer E, similar to answer B, deals with probabilities ("probable") and disqualifies this option.The correct option is answer D.
11🇧🇷 Answers:D
STEP 1: Read the question and identify your task.
This is a parallel question. You must find the answer that follows the same pattern of reasoning as the statement.
STEP 2: Read the argument in relation to your task.
In this case, the statement leads you into some mental gymnastics about the news coverage of an event, or lack thereof, and whether that determines the journalistic value of the event. Overall, the argument goes, however phrased, is a ridiculous claim. The statements use complicated language to confuse the matter a bit, but can be simplified to "if...then" statements. The first statement is: “If a newspaper does thisNocover an event, so be itNointeresting." The second version is just a paraphrase, but could be rephrased as "if an eventIt isinteresting andtutappear in a newspaper.
STEP 3: Find out what you are looking for.
The key is to look for similar patterns, including logical word usage (negative-negative, positive-positive) and sentence order (newspaper worthy, current newspaper).
STEP 4: Read every word of each answer option.
Answer A is: “If a politician is elected to Congress, then he or she WILL be corrupt.” You don't need to go to the second sentence because the first pattern doesn't match. Keywords are in the positive and not the negative. Answers B and C start out the same as Answer A, so you might as well knock those out quickly. Answer D states, "Any student who is NEVER accused of cheating NEVER cheated" and "Any student who REALLY cheats WILL be accused of cheating." Logic words follow the same pattern (negative-negative, positive-positive) and sentences do the same inversion of sentences (accused-misled, mistaken-accused), but you should check the last option to be sure. Answer E begins the same as answer D, but the second statement does not reverse the sentences. Instead, it says, "Every student accused of cheating has cheated" (accused-cheated, accused-cheated). Are you looking for the option thatthe majoritysimilar to the argument, and in this case,the correct option is answer D.
12🇧🇷 Answers:B
STEP 1: Read the question and identify your task.
This is a weak question. You have to find a basis on which the argument loses its effectiveness.
STEP 2: Read the argument in relation to your task.
The philosopher argues that pragmatism leads to relativism, making it difficult to develop a general guide to moral and ethical choices.
STEP 3: Find out what you are looking for.
This question will require you to review the answers and decide which one best weakens the argument, but likely the correct answer will remove the issue of relativism and provide a basis for general guidelines for moral and ethical decision-making.
STEP 4: Read every word of each answer option.
Answer A goes beyond the scope of the argument and says that people should come together and decide what is right for society as a whole. This cannot be your right answer. Answer B argues that the individual's judgment is in doubt and that experience in the world removes relativism ("the great leveller") and resolves discrepancies to develop better judgment. This option looks very promising as it removes relativism and provides a basis for general judgment of actions, but you should check the remaining answers. Answer C actually reinforces the point that pragmatism does not lead to any general truths and individuals disagree about what is practical and good, so C cannot be your answer. Answer D, similar to answer C, argues in favor of supporting the philosopher's claim. Answer E is a "throw your hands in the air and give up" statement that has nothing to do with the details of the philosopher's argument.The correct answer is answer B.
13🇧🇷 Answers:D
STEP 1: Read the question and identify your task.
This is a weak question. You must find the answer that indicates a weakness in the argument.
STEP 2: Read the argument in relation to your task.
The argument goes that people should stop taking vitamin D and calcium supplements because they get enough vitamins from their diet and sun exposure.
STEP 3: Find out what you are looking for.
The correct answer casts doubt on the relationship between the premises and the conclusion.
STEP 4: Read every word of each answer option.
Answer A is an interesting fact about the interaction between vitamin D and calcium, but it has nothing to do with how people absorb the two nutrients. Answer B may well weaken the basis of the argument, but this statement challenges you to make the unfounded judgment that looking at other publications is not as valid as designing a study and conducting a study directly in patients. After all, 1,000 releases is still pretty impressive. They continue to look for a stronger answer. Answer C contains an interesting fact about a common medical practice that is not relevant to the Committee's analysis. Answer D claims that taking supplements may have benefits other than bone health, and this may well weaken the argument, as the basis of the study was to assess whether supplements helped maintain bone health. This could be your answer. Answer E is a very small objection because you don't know whether non-breakfast foods with added vitamin D and calcium contain enough nutrients to make up for the lack of breakfast. Therefore,the correct option is answer D.
14🇧🇷 Answers:D
STEP 1: Read the question and identify your task.
This is an acceptance question. It asks you to find a statement that the conclusion depends on, then look for something that must be true for the conclusion to work.
STEP 2: Read the argument in relation to your task.
In this case, the argument is that Company X will sell underperforming divisions, which will drive down the stock price. Analysts are worried. As proof that this is a good move, you are told that the President has long recommended that some departments are outdated and too expensive to update. The bottom line is that the board's move will help the stock price.
STEP 3: Find out what you are looking for.
The correct answer will be something that makes you believe that, given the facts, the sale will indeed benefit the stock price.
STEP 4: Read every word of each answer option.
Answer A is an interesting fact, but not necessarily crucial to the conclusion. Answer B refers to the chances of success or failure of the sale, but not the conclusion that it will benefit the stock value after the sale. Answer C might mean a delay in the sale, but whether the sale will help the stock price depends not on when the sale takes place, but whether it takes place at all. Answer D seems very important. The reasoning is based on the assertion that the president has long supported the sale of some companies. Therefore, these companies must be part of the sale or the stock price improvement may not occur. This looks like a promising option. Answer E refers to the buyers, and whether or not the buyers recognize an underperforming department seems irrelevant to the sale and its impact on the stock price.The correct option is answer D.
fifteen🇧🇷 Answers:ONE
STEP 1: Read the question and identify your task.
This is a weak question. It asks you to find a statement among the responses that weakens the argument.
STEP 2: Read the argument in relation to your task.
In this case, the argument is that Company X will sell underperforming divisions, which will drive down the stock price. Analysts are worried. As proof that this is a good move, you are told that the President has long recommended that some departments are outdated and too expensive to update. The bottom line is that the board's move will help the stock price.
STEP 3: Find out what you are looking for.
They are looking for an answer that undermines the recommendation to sell the underperforming trades.
STEP 4: Read every word of each answer option.
Answer A suggests that all departments are essential to maintaining the stock price. If all businesses are required to stay together, selling some businesses could affect the "interaction" between all businesses and therefore the stock price. This is a strong candidate for your choice, as it suggests that the sale of some companies may actually drive down the stock price further, rather than sustaining it. You'll have to read the other answers to make sure it's the strongest among them. Answer B tells an interesting fact about selling different types of departments, but since the money received by underperforming departments is of little value to completion, this cannot be your choice. Answer C says something about the company's history, but again, past "thinking" about departments is of little importance to the conclusion. Answer D is about inflation, and inflation isn't even discussed in the argument. Answer E states that the company is looking for alternative ways for company X to support its stock price. There may be an alternative way, but we don't know if they work. Either way, that's not to say that selling the business isn't still a good option, or even a preferable option. Therefore,the best option is answer A.
16🇧🇷 Answers:B
STEP 1: Read the question and identify your task.
This is a parallel issue. He asks you to find among the answers a statement that follows the same pattern of reasoning as the argument.
STEP 2: Read the argument in relation to your task.
As you read the argument, pay close attention to the structure of the argument. The structure of the argument can be simplified as follows: IF restaurants older than six months, THEN popular with guests OR food critics. Last year, IF restaurants are popular with food critics, THEN popular with regulars. THAT'S WHY last year, IF a restaurant has been around for more than six months, THEN it was popular with food critics.
STEP 3: Find out what you are looking for.
This is a rather complicated logic pattern, but it must be repeated in the answer.
STEP 4: Read every word of each answer option.
Answer A is IF the garages are in Caedmon THEN they service foreign AND domestic cars. You can stop there because the first sentence does not contain the "or" construction. This cannot be your choice. Answer B is: IF there are interns at Willow THEN the interns are studying drywall OR furniture making. IF apprentices learn dry building this year, THEN they learn to make furniture. THEREFORE, this year, IF there is an apprentice in Willow, the apprentice is studying drywall. This is exactly the pattern and is probably your answer. Check the rest of the answers to be sure. Answer C starts well with IF a congressman is no longer in Congress THEN the congressman teaches OR writes a book. This answer then gives a specific example of Mary Seldon, and this does not fit the pattern of her reasoning, which remains general in her terms. So this cannot be your choice. Answer D makes the same mistake as answer C by going into a concrete example in the second part. Answer E is problematic from the start because it speaks in terms of "most" new films when its argument speaks in absolute terms ("all" or "all"). Therefore,the correct option is answer B.
17🇧🇷 Answers:ONE
STEP 1: Read the question and identify your task.
This is an acceptance question. It asks you to find the assumption on which the argument depends.
STEP 2: Read the argument in relation to your task.
In this case, the argument claims that a plant's immune system behaves similarly to the human immune system, and then the bizarre claim that you can figure out how plants fight the common cold.
STEP 3: Find out what you are looking for.
You read the responses to the only statement that seems to support this comparison.
STEP 4: Read every word of each answer option.
Answer A states that the same diseases that affect humans also affect plants. This is deceptively simple and you might think it's too simple to be the right answer. Still, this might be your best option, as the bizarre conclusion is based on the idea that the common cold attacks both plants and humans. You need to check the rest of the options to be sure. Answer B is definitely correct, but it is not a statement on which to base the comparison between plants and humans. It also says nothing about plants. Answer C says much the same thing as answer A, but it establishes a causal relationship ("because") that does not necessarily support the argument. It also creates unnecessary complications when discussing how the disease attacks any of them. However, if answer A is not as effective, you may want to consider this option. Answer D is definitely important to the conclusion of the argument that generalizes about all plants based on the rice plant, but this option says nothing about the common cold or disease and is still not as strong as answer A. Answer A. And it improves this Comparison between plant and human immune systems, but that doesn't support the whole cold and disease argument. Because of this,the best option is answer A.
18🇧🇷 Answers:E
STEP 1: Read the question and identify your task.
This is a variation of a descriptive question - an issue identification question. You have to find out the main disagreement between two people.
STEP 2: Read the argument in relation to your task.
Based on certain performance statistics, Joseph makes a value judgment that night customer service agents are more efficient than morning agents. Davis disagrees, arguing that each shift has different requirements and reps face different types of problems.
STEP 3: Find out what you are looking for.
You might expect the issue to describe how Joseph and Davis differ in their definition of efficiency.
STEP 4: Read every word of each answer option.
Answer A states that they disagree about why the night shift can be on this shift and not on the day shift. This has nothing to do with the efficiency question and cannot be the right answer. Answer B states that the disagreement is about the relationship between the time of day and the time it takes to process a claim. That's almost a good description, but it seems a little inadequate. You should read the rest of your answers to see if there's a better option. Answer C cannot be your answer, as none of them discuss whether the company moves reps between shifts. Answer D is wrong because they don't disagree about their own accuracy statistics. Rather, they disagree on why the statistical discrepancy exists in the first place and whether it should be used to gauge their efficiency. Answer E states that the disagreement is because both levels are able to provide quality service, but one level is faster than the other. This seems like a perfect description of the disagreement, and much better than Answer B, your second best answer, which only focuses on one aspect of how the time of day relates to the time it takes to process a complaint.The correct option is answer E..
19🇧🇷 Answers:ONE
STEP 1: Read the question and identify your task.
This is a final question. Essentially, the question asks you to find an answer that indicates why the legislation is not working in the best interests of voters.
STEP 2: Read the argument in relation to your task.
The first part of the argument is a description of the current way legislation is drafted. Starting with “But clearly this strategy…” you learn where the problem lies, namely that industry experts as lobbyists are paid by the industry they represent and therefore act in the interest of the industry, not in the interest of voters.
STEP 3: Find out what you are looking for.
Are you looking for something similar among the answers.
STEP 4: Read every word of each answer option.
Answer A states that industry experts will allow their own interests as industry lobbyists to influence their legislation. This is almost exactly the answer you were hoping for, but you should read the remaining options to be sure. Answer B has several words that speak against it, notably "heavily financially impacted" and "unfair". The argument is that they get paid, but you have no idea how much their compensation affects them. Again, the argument only says that the legislation is weakened, but gives no indication of how unfair it is. These conditions complicate the selection of this option. Answer C argues that legislators are less corrupt in making laws. Although this is a corollary of the argument, this does not mean that the current method does not serve the interests of voters. Answer D may be true, but it builds new information and attributes a reason to industry experts that isn't even mentioned in the argument. They have no idea if lucrative jobs await them later in their respective industries. This cannot be your answer. Similar to answer B, answer E contains words that disqualify it as your choice. The words 'generous' and 'want to keep' provide information not apparent in the argument. There is no mention of how much they are being paid or that their jobs are in jeopardy in that regard. Therefore,The correct alternative is answer A.
20🇧🇷 Answers:C
STEP 1: Read the question and identify your task.
This is a deductive question. The issue is figuring out which answer must be true based on the argument statements.
STEP 2: Read the argument in relation to your task.
The argument specifies a set of conditions that you can simplify into simple logical statements. The first sentence says diamonds (R) > 3 carats and rubies (R) < 3 carats. The second set: Most diamonds (R) and most rubies (R) < SI2. The third set: Diamonds(A) and Rubies(A) > SI2. Also diamonds (A) and rubies (A) < 3 carats. Since Ellington only buys stones <3 carats, you can imagine that E might buy Rubies (R), Rubies (A) and Diamonds (A). In the last sentence, you learn that Ellington only buys a shipment of diamonds. Therefore, Diamonds(A) is the only option. Allister is the sole source for this broadcast.
STEP 3: Find out what you are looking for.
With that in mind, read the responses.
STEP 4: Read every word of each answer option.
Answer A is too absolute and cannot be true. Ellington may only buy diamonds now, but nothing tells us that the company never buys rubies and only diamonds. Answer B says the opposite of what is said in the statements. In fact, Allister sells stones of higher purity than Richman. Answer C states that the diamond shipment has a clarity rating greater than SI2. It's working. Since Ellington must purchase this supply from Allister and Allister only sells stones with a clarity rating above SI2, the supply of diamonds must be rated above SI2. As for the other answers, answer D cannot be true as Ellington only buys stones < 3 carats and you know that (R) diamonds are > 3 carats. Answer E cannot be true because Allister is the only source Ellington can buy the shipment of diamonds from.The correct option is answer C.
21🇧🇷 Answers:B
STEP 1: Read the question and identify your task.
This is an error question. The first question is to find the error in the environmentalist's response to the coal plant manager.
STEP 2: Read the argument in relation to your task.
The manager of the coal-fired power plant says that coal will remain a dominant energy source because of its cost-effectiveness. The environmentalist argues that the technology will improve the efficiency of alternative energy sources, allowing them to cut coal.
STEP 3: Find out what you are looking for.
You can see that the environmentalist talks about technology and its efficiency benefits, but does not directly address the manager's discussion of the cost advantages of coal over alternatives. You'll look for something similar in your answer choices.
STEP 4: Read every word of each answer option.
With Answer A, the environmentalist does not address how long it would take to become competitive, but time is not a priori the center of the discussion and this is a weaker option. Answer B states that the environmentalist does not recognize the cost advantage of coal over alternatives. This lines up with what impressed you about the environmentalist's statement and is likely his response. You should check the remaining options to be sure. Answer C is incorrect because the environmentalist makes no such claim. In fact, the environmentalist says the coal industry has been slow to adopt this type of clean coal. Answer D discusses a scenario that neither the coal manager nor the environmentalist discusses and therefore cannot be your answer. From what the environmentalist said, you can deduce an answer of E, but since the environmentalist doesn't mention any cost advantage, you can't choose.The correct answer is answer B.
22🇧🇷 Answers:C
STEP 1: Read the question and identify your task.
This is an issue of empowerment. This question asks you to find a statement among the responses that supports the environmentalist's argument.
STEP 2: Read the argument in relation to your task.
The environmentalist's argument is based on technology and efficiency, and the coal industry's slow adoption of new technologies
STEP 3: Find out what you are looking for.
The correct answer will match these terms, and you should expect it to help the environmentalist overcome the weakness uncovered in the previous question and address the cost-benefit issue.
STEP 4: Read every word of each answer option.
Answer A seems to contradict the environmentalist's statement about clean coal and does not help the environmentalist solve the cost-effectiveness problem. Answer B weakens the coal plant manager's argument but does not strengthen the environmentalist's argument. Answer C states that there is a direct relationship between technological progress and cost acceptance factors. This meets your requirements by saying that the environmentalist's focus, technological advancement, affects value for money. You should check the rest of the answers. Answer D covers only one company, and one company's experience does not necessarily weaken or strengthen any of the arguments. Finally, answer E actually seems to weaken the environmentalist's argument by saying that, despite technological advances, the cost benefits were minimal. Because of this,The correct alternative is answer C.
23🇧🇷 Answers:C
STEP 1: Read the question and identify your task.
This is a description question. The question asks you to find the answer that describes the method used to make the argument.
STEP 2: Read the argument in relation to your task.
In this case, the argument specifies a scenario. Essentially, Randy has to spend the entire night working on a report for his boss, but a client has invited him to dinner and a discussion that same night. The last statement states that he has a difficult choice to please both his boss and his client, but he cannot do both.
STEP 3: Find out what you are looking for.
You need to look for a similar pattern in the answers.
STEP 4: Read every word of each answer option.
Answer A cannot be correct because the argument does not provide alternate versions of Randy's night. Talk about decisions. Answer B cannot be correct because the argument does not provide any other comparable situation. There is only one situation that Randy faces. Answer C looks like his answer. The argument talks about one set of responsibilities (the boss) and shows how it is inconsistent with another set of responsibilities (the customer), leading to a dilemma (can't do both). This appears to be your answer, but you should read the remaining options to be sure. Answer D cannot be correct because the boss's requirements do not translate to the customer's requirements. They are concurrent circumstances. Answer E starts well. Randy failed to work on the report in a timely manner, which led to the mystery, but nothing in the arguments suggests that harm will inevitably befall any of the parties involved.The correct alternative is answer C.
24🇧🇷 Answers:D
STEP 1: Read the question and identify your task.
This is a deductive question. As you read the question, you will learn that you must choose an answer that can be inferred or inferred from the content of the argument.
STEP 2: Read the argument in relation to your task.
The argument goes that bumblebees prefer red or striped snapdragons. Then you will be told that bumblebee is vital for snapdragon survival. Eventually, the nursery is growing more striped, dark-pigmented snapdragons to encourage the growth of the bee population.
STEP 3: Find out what you are looking for.
They are waiting for your correct answer to discuss the relationship between snapdragons and the bee population.
STEP 4: Read every word of each answer option.
Answer A cannot be inferred, as nothing in the reasoning suggests that the bee population is threatened by the absence of a particular snapdragon. The argument only suggests that the growth of snapdragons will help increase the population. Answer B cannot be deduced because the argument says that the bumblebee visits striped or dark-pigmented snapdragons more often, but does not say that without this type it will completely stop pollinating snapdragons. Answer C cannot be deduced because the argument makes no connection between the bumblebee's behavior and that of other insects. Answer D looks promising. From the argument that the bee population needs to increase, it is clear that concluding that the population is smaller than desired is not very far-reaching. They may also conclude that more streaky, darker pigmented snapdragons will help increase their numbers, as nurseries would not pursue such a strategy unless they believe this to be the case. Read the last option to make sure it's not better. Answer E says the opposite of what the argument says, that bumblebees are needed to save snapdragons, which is not the case.Answer D is the correct option.
SECTION III
Questions 1-6
As with all logic games, you follow the six-step process.
STEP 1: Identify the type of game.
The wording of this logic game is tricky. It asks you to keep an order: a coil cannot be used unless it has been tested first. It also prompts you to group the coils into untested, tested, and used. The game also doesn't tell you how many coils there are in each group, and a coil can be in both the tested group and the used group. This is one of those rare hybrid games that cannot be easily classified. As part of your exam strategy, you can save this game until you take the exam. With that assumption, proceed to step 2.
STEP 2: Start with your diagram.
You imagine a process of coils moving from left to right, from untested to tested, unused to used. Here is your chart.
STEP 3: Symbolize the clues.
By symbolizing the clues, you couple the reel to your status. Techniques for symbolizing are explained inChapter 3
The first clue is: “If G is tested, I is tested.” This is a simple "if...then" statement. You use the equals sign to indicate that a coil is at a certain stage in the process. This warning can be symbolized as follows:
G = T → I = T
The second clue is "If E is tested, G is tested", which is another simple "if...then" statement. They use the same format as the previous note:
E = T → G = T
The third clue is a clear definition and can be simply symbolized as follows:
D = T
The next two clues are formulated as complex conditional statements that need to be translated into "if...then" language. They are:
H is not used (~H) unless J is tested (J)
D is not used (~D) unless H is tested (H)
You negate the first terms (~H and ~D) and put the last terms (J and H) after the arrows. The final results are as follows:
Finally, you get the hint "If J is used and I is tested, K is used." This is an "if...then" compound statement that can be symbolized as follows:
This completes your symbology and you move on to step 4.
STEP 4: Double-check your symbology.
To check your symbology, translate your symbolized clues back into plain English and verify that they match the original language of each clue.
Your page should look like this:
STEP 5: Make deductions.
Finally, before delving into the questions, see if you can draw any conclusions from the setup and clues. Examine each type of deduction.
1. It cannot be the first or last deduction
This is not a sorting game, but it does contain some sorting elements. Based on your knowledge, you can draw the following conclusions:
If G is tested, then I is tested, so G can never be tested without also testing I.
When E is tested, G is tested, so E can never be tested without also testing G.
When H is used, J is tested, so H can never be used without also testing J (J will always come before H).
If D is used, H is tested, so D can never be used without also testing H (H always precedes D).
If J is used and I is tested, then K is used, so you can conclude that if J is used and I is tested, then J and K are used together.
2. Deductions for repeat items
You can immediately see that the first two "if...then" statements share the term G = T (G is tested). You see that E = T → G = T and G = T → I = T. Therefore, using the reflexive law (if a = b and b = c, then a = c), you see that E = T → I = T
From this you deduce that if E is tested, then G and I are tested. You can also add the "if...then" statement:
E = T → G = T & I = T
There are no other repeating elements that can be used to make additional deductions.
3. Down-to-two-Abzüge
There are no restrictions on how many there can be in each phase, and deductions don't pose limiting factors with words like "only" or "required". So you don't have deductions reduced to two.
4. Block Division Penalties
Note that a coil may appear in both the Tested and Used columns.
STEP 6: Answer the questions in the smartest order.
On test day, answer the questions in this order:
1. Answer the question List complete and accurate.
2. Answer the questions that provide more information about how to work.
3. Answer the remaining questions.
In this example, questions 1 and 4 are complete and accurate list questions and must be answered first. Questions that will give you more information include:
Question 3 (“J is not tested”)
Question 4 ("E and five other coils are the only coils tested, and if exactly three coils are used on the machine...")
Question 5 ("Every coil that is tested is used, and if I am used...")
Question 6 ("K is not used, what if exactly four coils are used...")
After answering these questions, it remains only to answer question 2.
THE ANSWERS
1🇧🇷 Answers:D
This question asks which of the answers could be a complete and accurate list of tested coils. They only deal with tested coils. Since you are only dealing with a group, you can resolve this issue without your photo. In that case, it's best to go through the answers one by one and see if they match the rules. You know that D is being tested. Each answer contains D, so the disqualification must come from the other clues. The answer A tests E, and you know that if E is tested, G must be tested. As good as G is listed below. But thanks to your conclusions, you also know that if E is checked, I must be checked too. Since this answer does not have I in the list, it can be deleted. Answer B has a similar problem. It has D and E and I, but excludes G. Answer C is even worse. It contains E, but both G and I are missing. Answer D states that the mechanic tests both D and I. There is no indication that D or I need to be tested with a different coil, so this may very well be your complete and accurate list. Answer E has D and G, but you know that if G is tested, I must be tested as well, and I is not listed in Answer E, so you can drop it as your choice.The correct alternative is answer D.
2🇧🇷 Answers:ONE
This question is about which statement can be true. The key is the wordcould, which tells you that the answer should be "possible" given the configuration and clues. The other answers will be impossible given the same facts. The way to answer this question is to review the answers and compare them to your clues and conclusions. If you check all the answers you can quickly see that all the answers relate to which coils are tested so you don't have to worry about which coils are used. You know that D will definitely be tested, so that will be a factor in your considerations. Answer A states that E and three other coils are used. They use the only certainty that the E coil is being tested to find out if this answer could be true. You go through the notes in Coil E. You know that G is tested when E is tested. That leaves one last reel to find out. You know from your deduction that if E is tested, then I is tested. This gives you three more coils (D, G and I). This scenario is not only possible, but also necessary when E is tested. You look at the other leads and realize that none of them require you to test another coil. You could stop there, but let's look at the other possible answers. Answer B is not possible because you have already proved that with E there are three other coils to be tested. Answer C has the same problem; with E three other coils must be tested, so it may not be possible to test E and another coil. Answer D states that D, G, and H are the only coils tested. The keyword isonly, which means that only these three coils are tested and no others. That can't be true, because your rules say that if G is tested, then I must be tested. As I is not on the list, this answer can be omitted. Finally, Answer E says that I and J are the only coils tested. Again is the keywordonly🇧🇷 This cannot be the case as your rules say that D is tested and this answer has no D in its list.The correct option is answer A.
3🇧🇷 Answers:C
This question provides new information to work with. It says that J has not been tested. The question asks you to figure out which answer must be true. The keyword ishave to, which means that since J is not tested, some requirement is created for other coils. You look at your clues and conclusions and see what that condition might be. The J-coil appears on the first track as follows:
H = U → J = T
Since the given fact is the negative of the term on the right side of the arrow, it may be useful to develop the contrapositive of this clue. You switch sides:
J = T → H = U
Then you negate both sides:
J ≠ T → H ≠ U
And now you have a new version of your lead that fits this question. Asserts that H is not used if J is not tested. You can add this to your tip list in case it helps you with later questions. The last clue also includes J and can be symbolized like this:
J = U & I = T → K = U
This tip is based on using J, but your question asks if J has not been tested, so this tip cannot help you.
With your new information, evaluate your options. Answer A states that D is not used. In your situation, untested J just tells you if other coils have been tested or not. It says nothing about which coils are used. You cannot know whether D is used or not, so this cannot be your answer. Answer B states that H is not tested. The only thing you can deduce from your hints is that H is not used since J is not tested. H may or may not be tested. You cannot be sure of your clues, so answer B is not your choice. Answer C states that D is tested, but H is not used. The first part is given by your tips. D is tested independently of the scenario. The second part is confirmed by the second part of its contrapositive above. This appears to be your answer, but you should go through the last two answers just to be sure. Answer D states that D is used, but that doesn't have to be true, and the question asks what needs to be true. Answer E states that H is tested but not used. The second part of this answer is confirmed by its contrapositive, but the first part is not. Given the clues, you don't know whether H was tested or not.The correct alternative is answer C.
4🇧🇷 Answers:C
The question says that coil E and five other coils are the only coils tested and exactly three coils are used in the machine. The keywords areonlyeexactly🇧🇷 These words create a constraint that will likely help you determine your answer. As this question requires accounting for tested and used coils, you will use your visual tools to represent the different scenarios. They also note that he asks what an exact list "may" be. This means that you only need to determine what is possible, not what must be true. You know that D is tested and you get that E is tested by the question. Record both in the tested column of your graph. The question states that there are five other coils tested besides E. D counts as one, so you need to figure out four more.
From the hints to the right of your diagram, you know that when E is tested, G is also tested. You can also use your conclusion to deduce that if E is tested, then I will be tested. You add G and I to your tested column. As shown below, H, J and K are left as options to fill the remaining two tested slots. One of the three will remain untested and also unused, as only tested ones can be used.
There's nothing else you can do to populate the grid, so take a look at your answers and see if any of them work in the scenario you've created. Answer A states that D, I, and J are used. You test this scenario. When D is used, H is tested, leaving another test point for J or K. Then you have to deal with I and J. Coil I works as already tested. J must be tested before it can be used, so the last two test slots are occupied by H and J, leaving K untested and unused. However, you have a problem. If J is used and I is tested, your last clue says that K should be used. K cannot be used as only three coils can be used and including K would give four. That's enough to disqualify this answer, but you could also say that if K is used, it needs to be tested, and you've run out of test slots for K, which is a double disqualification. This scenario is shown below:
Answer B states that D, H, and J are the three coils used. As before, if D is used, H gets one of his two remaining test points. Another tip is that J is tested when H is used, so J occupies the last remaining test slot. Again you have a problem because if J is used and I is tested then K is used and K would use four coils for that which is impossible in terms of the question. Answer C states that E, G, and I are the three coils used. You already know that E, G and I are tested, so they can be used. None of those trigger conditions that break your limitations, so this is probably your answer. You'll check the rest of the answers to be sure. Answer D states that H, I and K are the coils used. First, if H is used, it must have been tested, so it occupies one of the two remaining available test slots. From the hints, you know that when H is used, J is tested and occupies the last of your test slots. They're out of test slots, which is a problem. If K is used, it must also be tested and no test slots are available for it. This cannot be your answer. Answer E states that E, G, and J are the three coils used. You know that E and G are tested, so they can be used. If J is used then it will trigger its final lane and K must be used but this is not possible as only three coils can be used and K would do four in turn.The correct alternative is answer C.
5🇧🇷 Answers:E
This question states that all tested coils are used and coil I is used. It also asks which coil does not need to be tested. In other words, four of the five options need to be tested and one does not. You have to find the one exception. You are using your image again. Because every tested coil is used and you know that a coil must be tested to be used, both D and I occupy your tested and used columns.
Based on what you already know, review your clues to see if they tell you more. According to your fifth clue, H must be tested when D is used. Therefore, answer B is eliminated. Since H is tested, it is used, since according to the question, all tested coils are used. From Note 4, if H is used, J must be tested. Thus, answer C is eliminated. As J is tested, it is used accordingly. From Note 6 you know that if J is used and I is tested, then K is used and therefore needs to be tested. This eliminates answer A. You are left with answers D and E. Thanks to your hints, you know that coil D needs to be tested, so answer D is eliminated. This leaves the answer E and should be your answer. Of course, you can follow the tips and find that G doesn't necessarily need to be tested or used. The second hint is never called. E is never tested, so G doesn't need to be tested.The correct option is answer E..
6🇧🇷 Answers:B
This question tells you that K is not used and exactly four coils are used. The wordexactlyit is important because it will be a limiting factor that will help you to rule out possibilities and determine the correct answer. Also, the question asks you to find the answer that is wrong. You must eliminate the true answers to find your choice. They use your photo to help you.
The term "K is not used" is the negative of the last term in the sixth clue, so you must form the opposite of that statement to help you with this question. First you turn the pages:
K = U → J = U & I = T
Then you turn the signs around:
K ≠ U → J ≠ U & I ≠ T
Finally, flip the connectors:
K ≠ U → J ≠ U / I ≠ T
You add this new contrapositive to your list of tips next to your image.
Thanks to your new clue, you know that since K isn't used, J isn't used either.orI am not tested. You evaluate your answers based on the given situation. Answer A states that I is used. If used, it must have been tested. According to your new contra-positive note, or I was not testedorJ is not used. So if I is tested, it must be that J is not used. That leaves D, E, G, and H to fill the remaining three spaces (I takes up the first). Nothing in your lanes will stop you from using one of these rolls to fill the remaining three slots. Answer A is not wrong and cannot be your choice. Answer B states that J is used. According to his new contrapositive, if J is used, it must be so that I is not tested. This is the negative of a term in the first clue, and creating another contrapositive can help. We won't go through all the steps here, but this is the result:
I ≠ T → G ≠ T
This contrapositive leads to another contrapositive based on the second clue:
G ≠ T → E ≠ T
They say that G is untestable if I is untestable, and if G is untestable, then E is untestable. This also leads to the conclusion that if I, G, and E cannot be tested, they cannot be used either. That leaves just D and H, just two coils to fill the last three slots and that's not enough. Answer B must be wrong and therefore is your answer to this question. The last three answers refer to specific coils being tested. Regarding testing, the only coil affected by not using K is I, which is not in the C, D, or E answer. Just because a coil is tested doesn't mean it will be used, and your only limitation is the column used. So none of these answers can be wrong.The correct answer is answer B.
Questions 7-13
As with all logic games, you follow the six-step process.
STEP 1: Identify the type of game.
This is a group game. You know it's a grouping game because it asks you to group the seven children into the three activities. It also states that a child can only participate in one of the activities and the activities only happen once.
STEP 2: Start with your diagram.
Create a grid with three areas for each group - pottery, drawing and origami. Then shorten the children's names to their initials (B, E, J, K, P, S, and U) and place them in the upper right corner. Groups can also be abbreviated to C, D, and O.
STEP 3: Symbolize the clues.
The game description doesn't give you any information to work with, except that each child must participate in one of the activities and cannot participate in more than one at a time. It also tells you that the activity only occurs once. You go tip after tip.
Use 1:Exactly twice as many children choose drawing than ceramics.
You can use the abbreviated letter for each activity to represent the number of children in each group. Therefore, this note can be presented as follows:
D=2C
Use 2:Sharon and Usef participate in the same activity.
They use children's initials as symbols for children. The words "participate in the same activity" are really saying that if Sharon then Usef and if Usef then Sharon, you symbolize this clue with two representations as follows:
S → U
U → S
Use 3:Ezra and Karly are not participating in the same activity.
The phrase "no show" is the same as "if Ezra, then not Karly" and vice versa.
E → ~K
K → ~E
Use 4:Barry and Pakhi are not participating in the same activity.
This is similar to the third clue and looks like this:
B → ~P
P → ~B
Use 5:Barry takes part in pottery or origami.
This clue can be represented directly on the chart by appearing B over pottery and origami with a "/" before or after it to indicate that it can appear in any column.
Use 6:Jaime participates in the raffle.
You can display this information directly on the diagram and do not need to symbolize it.
STEP 4: Double-check your symbology.
To check your symbology, translate your symbolized clues back into plain English and verify that they match the original language of each clue.
Here's your image again, now with your cues symbolized next to it:
STEP 5: Make deductions.
Finally, before delving into the questions, see if you can draw any conclusions from the setup and clues. Examine each type of deduction.
1. It cannot be the first or last deduction
This is not a game of order, so you won't find this kind of deduction in this game.
2. Deductions for repeat items
There are two clues about Barry. The only thing you can deduce from these rules is that if Barry is in Pottery then Pakhi will be in Origami or Drawing and if Barry is in Origami then Pakhi will be in Drawing or Pottery. There are no other repetitive elements to help you make any deductions.
3. Down-to-two-Abzüge
As the size of each group is not limited, this type of deduction does not come into play. The first rule, D = 2C, may impose some restriction on the size of these two groups, but this restriction cannot be computed at this time.
4. Block Division Penalties
Tips 2, 3 and 4 create situations where blocks can cause constraints, but since you don't know the group sizes yet, you can't deduce anything yet. These limitations become apparent when questions add information to the game.
In this case, your attempt to find deductions didn't lead to any additional clues, but you may find more deductions as you work through your questions. You continue to answer questions.
STEP 6: Answer the questions in the smartest order.
Address the questions in this order:
1. Answer the question List complete and accurate.
2. Answer the questions that provide more information about how to work.
3. Answer the remaining questions
Questions 8 and 12 are full and accurate list questions and must be answered first. Questions that provide more information are as follows:
Question 10 (“Exactly one of the children chooses origami”)
Question 11 ("Sharon chooses the same job as Barry...")
The remaining questions are 7, 9 and 13 and can be answered in that order.
THE ANSWERS
7🇧🇷 Answers:B
This question asks you to judge which answercouldbe a correct assignment of children to activities. Responses are specific to children and do not ask for full attribution of children to activities. Therefore, you should go through the responses and use your grid to test each one.
(A) drawing of Karly; pakhi design; usef origami
See diagram below. Place each child in the appropriate group. You are left with B and E to assign to an activity. You know that E cannot be in the same group as K, and you know that B cannot be in the same group as P. This leaves you with an odd number of children in the lottery group. This arrangement cannot be possible due to your first clue, D=2C, which can be rephrased as C=1/2D. The drawing group must be divisible by 2. So this answer cannot be correct and you can add your deduction to your hint list later.
(B) Karly-Origami; Pakhi-Origami; Sharon-Origami
This scenario leaves B, E, and U assigned. Since B cannot be associated with origami P, B must be in pottery. U is assigned to origami because of clue 2. Therefore, E must be assigned to drawing. This scenario aligns with your clues and with the game, so it's a possible attribution of the children to the activities. This is your answer. You can stop here, but let's look at the other answers for learning purposes.
(C) drawing of Ezra; pakhi design; Ceramics Sharon
This scenario leaves B and K assigned. Neither B nor K can be assigned to the drawing, and you still need one when drawing for clue 1 to work, so this cannot be your answer.
(D) Barry-Origami; Esra-Keramik; Sharon-Keramik
This scenario leaves K and P assigned. Your character group must have at least six children to complete the first clue, as you already have three children in the pottery group. Since you only have two children to match the group of characters, you can't reach that number and that can't be your answer.
(E) drawing of Barry; Ezra origami; karly drawing
You don't need to use your chart for this, as Barry cannot be assigned to the drawing group. According to clue 5, he should be assigned to the pottery or origami group. This cannot be your answer.
The correct answer is answer B.
8🇧🇷 Answers:ONE
This question asks for a complete and accurate list of children whodo not dochoose the design. Look at each scenario again. This means that all the other kids choose to draw, so you have to find the answer that allows everyone but those listed to draw together and still follow your cues.
(A) Barry, Esra, Vogel
Use your chart to test this scenario. You only need one setting to work, so for the sake of argument say that B chooses pottery. P can't be in the same group, but she can't choose Draw either, so she chooses Origami. And it can be ceramic or origami, so choose one, in this case ceramic. They have S and U left and they can go in any group, but they must go together because, following your hints, they will choose the same group. Since you need to make your first ruler work, insert it into the drawing. This scenario works and this is your answer. Let's explore the other options for learning purposes.
(B) Barry, Sharon, Usef
This answer doesn't work because E and K cannot be together in the drawing group.
(C) Barry, Karly, Usef
You don't even need your chart for this. Usef and Sharon must be in the same group. Sharon cannot draw and she is not included in this answer, so this answer breaks the rule on the second clue.
(D) Barry, Karly
Again, you don't need your chart for this. As only two are not painting, five children remain in the drawing group. This means that the drawing to ceramic ratio is 5 to 2 (assuming B and K choose ceramic), which is greater than 2 to 1 and breaks with your first clue.
(E) Barry, Pakhi
This one has the same problem as answer B. E and K must move together, and that breaks the rule for clue 3. Even if you ignore that clue (which you can't), it also breaks the relationship with clue 1. That doesn't work. could be your answer.
The correct alternative is answer A.
9🇧🇷 Answers:E
The question asks you to find the oneIs correctstatement between responses. You can evaluate them without a diagram. Answer A cannot be true because it gives five children a choice between drawing and pottery. The ratio between these groups is 2 to 1, and there is no five-child configuration that gives an exact ratio of 2 to 1. Answer B cannot be true because you know that S and U must be in the same group. Usef cannot be the only child in any group. Answers C and D cannot be true for the same reason as answer A: the number of children drawing must be an even number and be able to give a 2 to 1 ratio to pottery. The process of elimination makes the E answer your choice, but you can test it to be sure. See diagram below. All the other children can be arranged to follow your suggestions and leave P at the pottery alone.The correct option is answer E..
10🇧🇷 Answers:B
That's what this question tells youexactlyA child chooses origami and asks which of the answershave tobe true. You've already created a similar scenario in a previous question, and in a test situation, you want to use that knowledge to quickly answer all subsequent questions that create the same scenarios. Of course, you know that with only one child in origami, there are six children left for the remaining two groups. For tip 1 to work, you need two kids to draw pottery and four kids giving you the 2 to 1 ratio you need. You need to see which groups of children work 2-4-1 from left to right. The next graph is the same one you created in question 8. You can use this example as a guide. Look at the possible answers and figure out which one must be true. Answer A says that K chooses ceramics. You see in the diagram that K could do the drawing, but you could also make K the child who is alone in origami and have P draw without disturbing the game. Answer B states that S chooses to draw. S and U must choose the same group, so S cannot be the only child of the origami. Also, you cannot swap S and U for B and E because E and K cannot be in the same group. You can swap B, P, E, and K in all sorts of configurations, but it remains true that S must choose to draw. This is your correct answer, but let's review the remaining answers for study purposes. Answer C states that B chooses pottery, but that cannot be his answer, as B could easily switch with P and be his only origami participant. Answer D says that P chooses to draw, but P could easily trade with B and get pottery, so this can't be your answer. Answer E states that Ezra chooses pottery. You can see from the diagram that it's possible, but it's also possible to swap E for K in the drawing, so that's not necessarily true.
The correct answer is answer B.
11🇧🇷 Answers:ONE
This question tells you that S chooses the same activity as B. This fact leads you to some new conclusions relevant to this question and this question alone. Your clues say that S and U choose the same activity. So you know that B, S and U choose the same activity. You also know that B, S, and U do not choose the same activity as P. Also, according to clue 5, S and U participate in pottery or origami. Finally, not all three can choose pottery because clue 1 would require six children to choose drawing, but there are only four other children, so all three must choose origami. This new deduction actually negates the previous deduction. Now you have these additional clues:
These rules apply only to this question; Keep your new clues separate from previous ones so you don't accidentally use them in future questions. The question asks which of the answerscouldbe true (nohave to🇧🇷 With your new information, look at your chart to evaluate the options. You can see that you only have three more children to combine - E, K and P. Tip 1 says that the character group should be twice the size of the pottery group K or P, so you can add a maximum of one to the group character. In fact, you can only assign one to each group. Answer A states that Ezra chooses to draw, which may be true. Ezra could choose pottery or drawing, so this is your likely answer. You know that P cannot have the same group as B, so P cannot be in origami. This eliminates answer B. Answers C, D, and E cannot be true because their conclusions make all three impossible.The correct alternative is answer A.
12🇧🇷 Answers:E
The question asks you to find among the answers the one that does not work as a complete and accurate list of children in the pottery group. Each answer has two names, so according to tip 1 you need four children in the character group. You already have a J in the character set, so you just have to ask yourself if there are three kids who might have a J in the character set that match your clues. Whoever fails this test is your answer. You rate each answer option.
(A) Barry, Karly
That leaves E, P, S, and U. You can group E, S, U, and P, S, U together in the drawing. So this cannot be your answer.
(B) Karly, Pakhi
That leaves B, E, S, and U. You can group B,S,U, and E,S,U together in the drawing. So this cannot be your answer.
(C) Esra, Pakhi
That leaves B, K, S, and U. You can group B,S,U, and K,S,U together in the drawing. So this cannot be your answer.
(D) Barry, Ezra
That leaves K, P, S and U. You can group K,S,U and P,S,U when drawing. So this cannot be your answer.
(E) Sharon, Usef
That leaves B, E, K, and P. There are no functional triples, mainly because B → ~P and E → ~K. There are many conflicts. Also, you run out of options. That must be your answer.
The correct alternative is answer E.
13🇧🇷 Answers:D
Your last question in this game asks which of the answershave tobe true. You don't need your diagram to solve this question. You just have to test each claim against your clues, and you can also use your experience from the previous questions. Of course, you can use your graph to design situations that test each one.
(A) E chooses an activity other than S.
Nothing in the rules says that E and S must choose a different activity. For example, the following scenario shows them sharing origami, so this cannot be your answer.
(B) E, J, and S do not choose the same activity.
You can come up with the following scenario to refute this. Since E, J, and S can choose the same activity, this cannot be your answer.
(C) E chooses an activity other than J.
The above scenario shows that E and J can choose the same activity, so this cannot be your answer.
(D) B, J, and K do not all choose the same activity.
This means that B and K must be with J when drawing, due to hint 6. This is impossible because it conflicts with clue 5, which requires B to choose pottery or origami, so it must be true that B, J, and K do not can choose the same activity. This is your answer, but let's check the last answer for learning purposes.
(E) B chooses a different activity than E.
You can come up with the following scenario to refute this. Since B and E can share the same activity, this option need not be true.
The correct alternative is answer D.
Questions 14-20
As with all logic games, you follow the six-step process.
STEP 1: Identify the type of game.
Several clues say that this is a squad game. Of course, the game says that there are two test "groups". It also states that no rat can be in more than one group and states a condition under which the rats must be divided into two groups ("at least one brown rat and one white rat").
STEP 2: Start with your diagram.
Create a graph that represents the two test groups. In this case, a two-column grid is perfect for your purposes. Leave a space to the right of your chart to symbolize your clues. Also, shorten your mice's names and group them by color (B for brown and W for white).
STEP 3: Symbolize the clues.
Before going through the clues, you need to understand what the setup says, that each group must have at least one brown rat and one white rat. This cannot be easily symbolized, but you can represent it in your diagram by creating slots for the required mice in each group. Now you're going to take clue by clue and simplify them into easy-to-understand equations.
Use 1:Neither group includes Abby and Iris.
This clue is designed to confuse you, but it basically says that A and I cannot be in the same group together. You can symbolize it like this:
A → ~I
I → ~A
Use 2:Neither group includes Elle and Horn.
The wording is the same as in Track 1, so you can symbolize it the same way:
E → ~H
H → ~E
Use 3:If a group includes Dennis, it does not include Horn or Iris.
Again, the wording is intended to make it more difficult to understand what the notice is actually saying. In this case, it says that if a group contains D, then H and I cannot be in that group. This can be symbolized as follows:
D → ~H & ~I
Due to the complexity of this clue, it may be useful to form the contrapositive of this clue as well. This is symbolized as follows:
H/I → ~D
Use 4:If Group 1 contains Carl, Group 2 contains Horn.
This is a simple "if...then" statement. If Carl is in group 1, Horn is in group 2. This can be symbolized as follows:
C = 1 → H = 2
STEP 4: Double-check your symbology.
To check your symbology, translate your symbolized clues back into plain English and verify that they match the original language of each clue. When finished, your page should look like this:
STEP 5: Make deductions.
Before delving into the questions, see if you can draw any conclusions from the setup and clues. Examine each type of deduction.
1. It cannot be the first or last deduction
Since this is not a game of order, this deduction does not apply.
2. Deductions for repeat items
You see that H is repeated on several tracks. Of particular interest is the repetition between lane 4 and the contrapositive of lane 3. You can conclude that if C is in group 1, then D cannot be in group 2 because H is in group 2. This can be symbolized as follows :
C = 1 → D ≠ 2
3. Down-to-two-Abzüge
Each group consists of at least two mice (one brown, one white). You notice that D is a brown mouse. Due to clue 3, if D is in a group, then the only white rats he can be grouped with are E and F. You also noticed that F is the only rat that is a free agent. There are no restrictions on where you can be grouped and who you can be grouped with.
4. Block Division Penalties
These deductions should not be made.
STEP 6: Answer the questions in the smartest order.
Answer the questions in this order:
1. Answer the question List complete and accurate.
2. Answer the questions that provide more information about how to work.
3. Answer the remaining questions
Although not worded as such, question 14 is a complete and accurate list question and should be answered first. Questions that provide more information include the following:
Question 15 ("Carl is in Group 1...")
Question 16 ("Elle is in Group 1...")
Question 17 ("Abby is in the same group as Dennis...")
Question 20 ("Dennis is in Group 2...")
Questions 18 and 19 remain, which can be answered in that order.
THE ANSWERS
14🇧🇷 Answers:ONE
This question asks which answer gives a possible grouping of mice. The question keyword iscould, which means that four out of five answers are not possible. You must go through each answer and compare it to your graph and the clues listed to the right.
(A) Group 1: Abby, fern, horn
Group 2: Carl, Elle, Iris
As you can see in the diagram, no lanes were violated and each group has one brown rat and one white rat. This is your answer and you can move on to the next question, but we'll evaluate the other answers for learning purposes.
(B) Grupo 1: Carl, Farn, Horn
Group 2: Abby, Dennis, Elle
You can see the problem here without looking at your diagram. According to the fourth clue, if C is in Group 1, then H must be in Group 2. Since H is in Group 1, this cannot be your answer.
(C) Group 1: Carl, Dennis, She
Group 2: Farn, Horn, Iris
This cannot be your answer because group 2 does not contain a brown rat and each group must have at least one brown rat and one white rat.
(D) Group 1: Abby, Elle, Iris
Group 2: Carl, Farn, Horn
Abby and Iris can't be in the same group according to the first clue, so this can't be your answer.
(E) Group 1: Abby, Dennis, Fern
Group 2: Carl, Elle, Horn
Elle and Horn cannot be in Group 2 together after the second clue. This cannot be your answer.
The correct alternative is answer A.
fifteen🇧🇷 Answers:B
This question gives you a fact that C is in group 1. Based on this fact, you need to figure out which of the pairs is listed in the answers.have tobe in group 2 together. One conclusion you can draw before looking at the responses is that since C is in group 1, H must be in group 2 (Note 4). By the contrapositive of lane 3, you know that D cannot be in the same group as H, so it cannot be in group 2. This leaves rat A as the only brown rat available for group 2. You look at your answers and see that B says that A and H must be together in group 2. This is your answer. There is no need to check the remaining options.The correct answer is answer B.
16🇧🇷 Answers:E
This question tells you that E is in Group 1 and asks you to find the pair thatcouldbeing in group 1 along with E. This means that four out of five couples cannot be in Elle's group. You immediately see that E is a white mouse and that one of the two mice must be brown, otherwise the game conditions will not be met. You view your responses and rate them one after the other.
(A) Fern and iris
They're both white mice, so that can't be your answer.
(B) Karl e Fern
If Carl is in group 1, then H is in group 2. Since H is in group 2, rat D cannot also be in group 2, so the brown rat for group 2 is A. That leaves H and I to do this remainder of group 2 to fill in, but hint 1 says that A and I can't be in the same group, so that doesn't work. This cannot be your answer.
(C) Karl e Horn
You know this can't be your answer, because clue 4 says that if C is in group 1, H must be in group 2.
(PT) Abby e Karl
Since this scenario calls for two brown mice, only D is left as a brown mouse in Group 2. Thanks to your conclusion that D cannot be in group 2 when C is in group 1, you know that this cannot be true. and this option may not be your answer.
(E) Karl and Iris
This scenario makes A and D available as a brown rat in Group 2, but you know that D cannot be in Group 2 since C is in Group 1. So A automatically becomes the brown rat in Group 2. You can simply put E and F in Group 2 and stay consistent with the game and lanes. This is your answer.
The correct alternative is answer E.
17🇧🇷 Answers:E
The question informs you that Abby is in the same group as Dennis and asks you to find the true statement from the options. This means that four out of five options are impossible, given the setup and tracks. For the purposes of this question, you now have the clue A → D and D → A. This also tells you that C will be the brown mouse for the group where A and D are not. This produces the following warnings:
C → ~D & ~A e D/A → ~C
You should test each answer to see which ones might be true.
(A) Elle and Fern are in Group 2
Elle and Fern are white mice, so you'll need a brown mouse to complete the party. Since D and A must be together, it must be C. This scenario leaves H or I to fill the last remaining space in group 1, but D cannot be in the same group as H and A cannot be in the same group as I. So , this cannot work and is not your choice.
(B) Horn is in Group 1
According to the contrapositive of lane 3, if Horn is in group 1, D and A must be in group 2. This forces C to be the brown rat of group 1 and that is impossible. As per tip 4, if C is in group 1, then H must be in group 2. This cannot be your answer.
(C) Carl is in Group 1
According to tip 4, if C is in group 1, then H is in group 2. D and A must also be in group 2, but this contradicts its contrapositive - D and H cannot be in the same group - so this can don't be your answer.
(D) Elle and Fern are in Group 1
Since each group must have three mice and this arrangement leaves only one spot in group 1, you can determine that D and A must be in group 2. C must fill this space in group 1 so that there is a brown mouse. This scenario leaves H or I to fill the last remaining space in group 2, but clue 3 says that neither H nor I can be in the same group as D, so this cannot be your answer.
(E) Iris is in Group 2
This is your last remaining option and should be the correct answer, but let's go through it to be sure. With I in group 2, you know that D and A must be in group 1 and C in group 2. In this scenario, E, F, and H must allocate the remaining three open spaces, and there are several configurations that work. Below is one of them. This is your answer.
The correct alternative is answer E.
18🇧🇷 Answers:D
The question asks you to find among the answers the pair that couldNobe in group 2 together. You can use your experience on question 17 and quickly choose D as the answer. Because? Working on this question, you discovered that if A and D are both in group 2, this forces C to be in group 1, since it's your last remaining brown mouse. As per tip 4, if C is in group 1, then H must be in group 2, and D and H cannot be together in group 2. There is no reason to check other answers.The correct alternative is answer D.
19🇧🇷 Answers:C
The question asks which of the answers you will findcouldtrue, then four out of five answers are impossible. Answering this question is eliminating impossible answers to find the possible one.
(A) Carl is in Group 1 and Elle is in Group 2
This cannot be true because Clue 4 says that H is in Group 2 if C is in Group 1. Clue 2 says that E and H cannot be together in group 2, so you can eliminate that choice.
(B) Abby is not in any group and Carl is in group 1.
If A is completely eliminated and C is in group 1, then D must be the brown rat in group 2. Hint 4 says that if C is in group 1, then H is in group 2. Hint 3 says that D and H can't be together in group 2, so this can't be your answer.
(C) Abby is in Group 1 and Elle is in Group 2.
You can see from the example diagram that under the conditions of this option you can create any possible scenario. So this is your answer. Let's look at the rest of the options for learning purposes.
(D) Abby is in Group 2 and Dennis is in Group 2.
This option has the same problem as question 18. If A and D are in group 2, then C must be the brown mouse in group 1. If C is in group 1, mouse H must be in group 2. Mouse D and H cannot be together in group 2, so this cannot be your answer.
(E) Abby is in group 2 and Carl is not in any group.
This option forces an unsustainable situation. With A in Group 2 and therefore D in Group 1, you must divide the remaining white mice between the two groups. Unfortunately, clue 1 says that A and I can't be in the same group together, and clue 3 says that D and I can't be in the same group together, so you don't have to go anywhere to put EU. This cannot be your choice of answer.
The correct alternative is answer C.
20🇧🇷 Answers:D
The question tells you that D is in group 2 and asks you to find among the answers that mouse which, if assigned to a group, must also be in group 2. You already know from your previous work that if Carl is in group 1 and D is in group 2, the scenario cannot work. Carl in group 1 forces H to be in group 2 and D and H cannot be in the same group. Answer D must therefore be correct. All other options are easily assigned to group 1 and need not necessarily be in group 2, and answers A and C are impossible as D cannot be in the same group as H or I.The correct option is answer D.
Questions 21-26
As with all logic games, you follow the six-step process.
STEP 1: Identify the type of game.
If you pay close attention to the game's language, you'll see that this is a two-level sorting game. Includes grouping and sorting language. For example, you need to group six out of eight loads into three trucks. This is definitely a grouping requirement. It then says that the cargo must be loaded in a certain order - first at the front, then at the back - and that is definitely the language of the order. Each truck has two elements - front and rear - which is another feature of the two-level order game. Finally, the two slots in each group are specified. The front and rear of each truck are final and not interchangeable. So you know you're up against a two-tier ordering game.
STEP 2: Start with your diagram.
You can imagine a loading dock at the bottom of the diagram with the trucks lined up from left to right. (This might sound confusing, but remember that dock loaders have to go through the back to load the front, so the loading ramp is at the bottom.)
STEP 3: Symbolize the clues.
There are only three lanes in this game. Go through them one by one.
Use 1:L can only go in truck 2.
This can be shown in the diagram. The clue doesn't say whether L needs to be in the front or rear load area, so put the letter L above the column for truck 2.
Use 2:T and W cannot rear-end a truck.
These are really two different tracks. You can represent both using the box representation method:
Use 3:If S is loaded onto a truck, then N and O go onto the next truck, unless S is loaded onto truck 3.
This hint is a complex conditional statement. Essentially, if S is on truck 1, N and O are loaded on truck 2. When S is loaded on truck 2, N and O are loaded on truck 3. But if S is loaded on truck 3, N and O can be loaded on any truck. You can set this up as two separate conditions to represent the first part of the statement:
S1→ N2& Ö2
S2→ N3& Ö3
The contrasts are as follows:
~N2/~O2→ ~S1
~N3/~O3→ ~S2
The last part of the statement is implied; when S is in truck 3, no restrictions are imposed on N and O.
STEP 4: Double-check your symbology.
To check your symbology, translate your symbolized clues back into plain English and verify that they match the original language of each clue. When finished, your page should look like this:
STEP 5: Make deductions.
1. It cannot be the first or last deduction
At first glance, you might think that the third clue gives you the opportunity to draw that conclusion. Because if N and O have to be loaded onto the next truck after S, then S can't be last, but it isn't. The "unless" clause in this notice eliminates that opportunity. S can be loaded onto truck 3 because the clue says that N and O can be loaded anywhere if that happens. Only if S is loaded on truck 1 or 2, N and O should be loaded on the next truck. Therefore, you cannot make that kind of deduction for this game.
2. Deductions for repeat items
You can't draw that kind of conclusion, as there are no repeat items between clues.
3. Down-to-two-Abzüge
You definitely cannot place cargo in a specific hold. You need more information to make this type of deduction.
4. Block Division Penalties
If S is on truck 1 or truck 2, the next truck will be fully loaded with N and O. This creates a block that cannot be split and will likely limit cargo loading. Since T and W cannot be loaded onto the back of a truck, they obviously cannot be loaded onto the same truck. You can add these clues to your list:
T → ~W
W → ~T
Finally, before delving into the questions, see if you can draw any conclusions from the setup and clues. Unfortunately, there aren't many clues in this particular two-level sorting game, and none of the clues taken together lead to any further conclusions.
STEP 6: Answer the questions in the smartest order.
Answer the questions in this order:
1. Answer the question List complete and accurate.
2. Answer the questions that provide more information about how to work.
3. Answer the remaining questions.
Question 21 is the complete and accurate list of questions for this game and must be answered first. Questions with more information include the following:
Question 22 ("P and A are not charged...")
Question 24 ("L, S and W are loaded in different trucks...")
Question 25 ("N, P and R ... loaded on the fronts ...")
Question 26 (“P is in the bed of truck 2 and N is in the bed of truck 3…”)
This leaves question 23 to be answered as the last one.
THE ANSWERS
21🇧🇷 Answers:D
The question asks you to determine which answer produces a valid load record. You don't need your chart to answer this question. You can use your hints to test each answer. Fortunately, you only need the first clue to find the correct answer to this question, as the only L answer on the second truck is a D. You don't have to look any further, and there's no reason to compare the answer with the other clues.The correct option is answer D.
22🇧🇷 Answers:ONE
The question informs that P and R are unloaded and then requests a possible listing of loads loaded in the front holds of the three trucks. The given facts tell us that L, N, E, S, T and W are left as possible cargos and you should have six holds filled. Thus, all these elements must be loaded. Tip #1 says that T and W need to be preloaded, so the only answer that works has T and W in it. Fortunately, answer A is the only option that includes both T and W.The correct option is answer A.
23🇧🇷 Answers:C
This question requires you to determine which pair of loads, when loaded together, should go into truck 3. Note that the question does not say that the pairs are in a specific order, front and back, so order is not the issue, just the pairing. Again, your graph is not needed for this question. You know from the third clue that if S is paired with N or O, they must be in truck 3. Answer C pairs S and N and they can only be in truck 3. So,The correct alternative must be answer C.
24🇧🇷 Answers:C
The question says that L, S, and W are each loaded onto a different truck, then asks you to choose one of the answerscouldbe true. This means that four out of five answers must be wrong. Use your chart to review each scenario. You know that L is in truck 2, so S and W must be in trucks 1 and 2, but you don't know which is in which truck. You have to test each response and see what happens.
(A) T is in truck 1
Hint 2 says that T must be loaded before truck 1. Therefore, W cannot be in truck 1. W cannot be in truck 2 because track 1 requires L to be in that truck, so W must be in truck 3. This leaves truck 1 for S. Track 3 requires N and O to be loaded in truck 2, but it's not enough space and you can't share this block. So this cannot be your answer.
(B) S is in truck 2.
The question states that L, S, and W are loaded on different trucks, but hint 1 says that L must be loaded on truck 2, so if S is on truck 2, the conditions in the question cannot be met and it cannot be your answer.
(C) N is in truck 3.
This situation will not trigger any conditions defined by your hints and as you can see from the diagram you can create a viable loading scenario so this is your answer. But let's go through the rest of the answers for learning purposes.
(D) W is in truck 3.
Again, L must be in truck 2 and S in truck 1, leaving no room for N and O in truck 2. Since you cannot do tip 3, this cannot be your answer.
(E) S is in truck 1.
As with the previous answers, you know that L must be in truck 2. If S is in truck 1, then N and O must be in truck 2, but L makes it impossible for N and O to be loaded together. This cannot be your answer.
The correct alternative is answer C.
25🇧🇷 Answers:E
This question tells you that N, P, and R are loaded on the fronts of the three trucks, but the order is undetermined. You have to find the answer to this.have toare true, no matter what order they are in. You know right away that the T and W cannot be carried, as there are no more front grips available. So only L, O, and S need to be loaded into the remaining three holds. Clue 1 says that L must be in the back of truck 2. You look at the graph to find out which answer must be true under the conditions. To test the answers, put N, P, and R in no particular order to see what might happen. You quickly see that S cannot be in truck 1 because in truck 2 there is no room for N and O together. Also in truck 2 there is no place for S, so it must be in truck 3. So O must be in Truck 1. You can quickly see that the answer E must be the right choice. You can rearrange N, P, and R, but O must be in truck 1 regardless of order, as truck 2 and truck 3 will be populated with L and S, respectively.The correct alternative is answer E.
26🇧🇷 Answers:B
You will get that P is in the back hold of truck 2 and N is in the back hold of truck 3. You must choose the answer that contains a possible list of loads on the fronts of trucks 1, 2 and 3. This means that four out of the five options are impossible, given the terms of the question. The list of loads in each answer is in order (1, 2, 3). You look at your graph and test each scenario.
(A) W, O and R
This scenario leaves L, S, and T for loading and only one hold, the back of truck 1. None of these loads can enter this hold. Clue 1 says that L must be on truck 2. Loading S requires loading N and O together on truck 2, and that's not possible. Finally, clue 2 informs that T cannot be loaded on the back of a truck.
(B) W, L e O
This scenario works. All cargo in the front holds matches your lanes, and you're left with R, which can be loaded in the back of truck 1 without conflicting with your lanes. This is your answer, but let's go over the rest of the answers for learning purposes.
(C) R, L e O
This option leaves S, T, and W as possible loads to carry in the remaining hold in the back of Truck 1. All three fail their checks. S requires N and O to be loaded onto truck 2, but there is no room for that. According to clue 2, neither T nor W can be loaded in the back of a truck. Note that just because N and O are together in truck 3 does not necessarily mean that S must be in truck 2. The "if...then" condition goes from left to right, but not vice versa. Either way, that can't be your answer.
(D) O, R e S
This scenario leaves L, T, and W as possible cargo to load in the remaining hold in the back of Truck 1. All three fail their tests. Clue 1 says that L must be in truck 2. Clue 2 says that both T and W cannot be carried in the back of a truck, so this cannot be your answer.
(E) S, O and R
This is just a reordering of Answer D and the same terms apply. This is not your answer.
The correct answer is answer B.
SECTION IV
PASSAGE 1
1🇧🇷 Answers:D
STEP 1: Read the question and identify your task.
This is a variation of a main idea question – a main purpose question. You must choose from the answers the one that best describes what the passage is trying to achieve.
STEP 2: Go back to the passage to find the answer.
Since this is a main idea question, see the summary of the passage. Then describe that content in slightly more general terms and find the answer option that matches that description.
STEP 3: Read every word of each answer option.
Answer A focuses too heavily on the role of the Supreme Court. As the passage discusses a Supreme Court decision,Wesberry x Sanders, the passage focuses much more on proposals approved by referendums. Answer B speaks of errors, but the passage makes no judgments about whether states made an error or not. LSAT passages almost always avoid such extreme judgments about people or subjects. Answer C speaks of "unique" problems, but the passage makes it clear that these states are dealing with problems that exist across the country and are by no means unique. Answer D states that the passage is about legislation passed by voters to solve a problem, and that is what the passage in the first two paragraphs does. The passage uses these paragraphs to discuss proposals for resolving the problem of political manipulation. The response then says that the excerpt discusses arguments put forward by the opposition that might be used in the future, such proposals being raised in other states, which is exactly what the second and third paragraphs discuss. This should be your answer, but you should check the remaining options to be sure. Answer E has the same problem as answer B. She attributes too much biased intent to the passage, saying she judges the reform effort as "expensive and ill-advised."The correct option is answer D.
2🇧🇷 Answers:C
STEP 1: Read the question and identify your task.
This is a question of inference. The question is about a specific part of the passage and what is implied in the language.
STEP 2: Go back to the passage to find the answer.
In this case, the keyword or term is the year "1964". You need to look back in the passage and reread the sentences that cover that year. They discover that 1964 was the year of the Supreme Court decisionWesberry x Sanders, which stated that all congressional districts must contain the same number of people and established the "one person, one vote" requirement in determining congressional districts. His question concerns the situation before the Supreme Court decided this case. This likely indicates an imbalance in voting between districts.
STEP 3: Read every word of each answer option.
Answer A states that it is implied that political parties had no influence prior to 1964, but in fact the opposite is implied in the passage. Political parties had even more control before 1964, which is why the decision was necessary in the first place. Answer B states that some citizens voted less than one full vote. The passage says nothing about the value of a less than full vow. Instead, it discusses the district and its impact on elections. Answer C looks promising as it deals with the design of a neighborhood and how this can create an imbalance in favor of a certain group. This may be your answer, but you must evaluate the remaining options. For Answer D, there is no evidence that the Supreme Court case was about a state incongruity or that one state has an advantage over another in this regard. Answer E has turned things around. After 1964, the Supreme Court gained the power to overturn redistricting plans, not the other way around.The correct option is answer C.
3🇧🇷 Answers:D
STEP 1: Read the question and identify your task.
This is a line ID question. The question asks you to put a specific section of the section in context and find among the answers the best description of how it fits in that context.
STEP 2: Go back to the passage to find the answer.
In this case, the section concerns the one you already looked at in question 2, theWesberry x SandersDecision. You can use what you know from question 2 to answer this question. Analyze each response and assess its potential.
STEP 3: Read every word of each answer option.
Answer A cannot be your correct answer as the decision created "one person, one vote". It was not a problem the decision was intended to resolve. Answer B is tempting because the Supreme Court decision was an early process that led to the most recent proposals, but the direct link is weak. You'll still keep that answer in mind as you go through the remaining options, and if no better answer comes up, this might be your choice. Answer C cannot be correct because there is no mention of the Supreme Court decision regarding the efforts of the executive or legislative branch of the United States government. Answer D is pretty accurate. The passing section gives a historical perspective. There are several keywords that make this an ideal choice, such as "from the early years", "1964", and "continued to be influenced". These are historic sounding phrases. Answer E cannot be correct because the Supreme Court tried to resolve the gerrymandering but failed to do so. While answer B looks promising, answer D is much better, soThe correct answer is d.
4🇧🇷 Answers:C
STEP 1: Read the question and identify your task.
This is a line ID question. This question asks you to refer to specific lines (53-59) in the passage and asks you to find among the answers a conclusion you can derive from these lines regarding Proposition 20.
STEP 2: Go back to the passage to find the answer.
You find the lines in the passage and discover that they discuss Proposition 27 and its attempt to eliminate the Citizens Redistricting Commission. With that in mind, consider your choices.
STEP 3: Read every word of each answer option.
Answer A reflects the opinion of the proponents of Proposition 27, but has nothing to do with Proposition 20. Answer B attributes a motive to the proponents of Proposition 27 that is not evident from the lines. Answer C begins by saying that the commission existed before the election. Since Proposition 27, if passed, was intended to eliminate the commission, it should exist prior to the election. Also, Proposition 20 should allow the commission to draw districts for the congressional map, so you can conclude with certainty that they didn't have that power before the election. Since both elements of the C answer are implied on the lines, this is probably your answer, but you should check the rest of the options to be sure. Nothing in the lines in any way implies what voters would have done in any other scenario, so D cannot be correct. Finally, in answer E, nothing in the lines implies the unconstitutionality of the commission or how the courts would react if Proposition 27 does not pass. Also, this answer has nothing to do with Proposition 20 the question is being asked about.The correct alternative is answer C.
5🇧🇷 Answers:E
STEP 1: Read the question and identify your task.
This is a weak question. You must find the statement among the responses that most undermines support for Proposition 20.
STEP 2: Go back to the passage to find the answer.
Refer to the passage and analyze the third paragraph and its arguments in favor of Proposition 20. In this case, the paragraph discusses the fact that non-Proposition 20 politicians are guaranteed tenure due to gerrymandering. The population of their constituency is quite limited and therefore they only have to satisfy that specific constituency to get re-elected. The correct answer will present a fact that proves that, even with such maneuvers, politicians are not secure in their mandates.
STEP 3: Read every word of each answer option.
Answer A is tempting. Why would people want to change the system when most voters are satisfied with their representatives? But Proposition 20 is designed to help the minority and underrepresented, not the majority. While this option may be tempting, it's not the strongest one. For answer B, bipartisan compromises and successes in some cases don't speak much against the thesis. Answer C actually seems to make a stronger case for the claim, saying it will be blind to income, race, and gender. Answer D is a technical question, and even if all commissioners are members of a political party, that doesn't mean it can't be balanced to prevent one party from having more influence over the other. Finally, Answer E points out that population movements can make the implementation of the proposal unfeasible. This would definitely undermine the arguments of Proposition 20, because if Answer E is true, even if it is, it would not be effective.The correct option is answer E..
6🇧🇷 Answers:B
STEP 1: Read the question and identify your task.
This is a line ID question. The question asks you to find a synonym for the word among the optionsentrenchedwhich gives the meaning used in the passage.
STEP 2: Go back to the passage to find the answer.
Refer to the passage and read the sentence that contains the wordentrenchedto determine how to use it. In this case, it is perceived that the established politicians do not need to make political concessions because they are sure of keeping their jobs. The correct answer will most likely be a word similar to such a secure position.
STEP 3: Read every word of each answer option.
Answer A can be seen as the opposite of what the word means, as politicians are elected to the same office over and over again. Answer B is promising because politicians are able to maintain their position despite any omissions on their part. They are “deeply rooted” in their position. You check the rest of the answers to see if there's a better option. Answer C seems to be more of an adjective for a person under pressure. If politicians were more vulnerable but more principled in their stances on issues, they might be called unshakable, but that is not the case here. Answer D is similar to answer C, but politicians are not stubborn or opposed. The answer E would mean that politicians would become active and not be persuaded to rethink. This is also an imprecise correspondence with its definition.The correct answer is answer B.
PASSAGE 2
7🇧🇷 Answers:D
STEP 1: Read the question and identify your task.
This is a main idea question. However, for paired passages, look for possible answers that articulate the main ideas of both passages as they relate to each other.
STEP 2: Go back to the passage to find the answer.
Consult the summary of your ticket.
STEP 3: Read every word of each answer option.
Answer A discusses solutions to problems not covered in any of the passages. Although the two theatrical movements discussed here arose in response to social ills, there is no sense in the passages that they were "solutions" to the problem of the Great Depression. For similar reasons, answer option B can be eliminated. Passages A and B deal with the same circumstance: the Great Depression. And both address theatrical movements that arise from these social and economic realities. But passage B is not a solution to a problem in passage A. There is also no contradiction between the passages, so eliminate answer choice C. However, answer choice D largely corrects the relationship. The passages are very similar, both describing theater groups that arose in response to the circumstances of the Great Depression. This is probably the correct answer. Answer option E cannot be correct. There is no indication that Shock Troupe is a refinement of group theater. In fact, Passage B suggests that the opposite is true, as the shock troupe later adopted some of the methods of group theater.The correct answer must be D.
8🇧🇷 Answers:C
STEP 1: Read the question and identify your task.
This is a question of inference. The question asks you to evaluate each answer to see if it can be inferred from the passage based on what you read.
STEP 2: Go back to the passage to find the answer.
They relate to the passage and look closely in regards to new theater groups. By understanding the new groups, you can make some assumptions about what existed in the early 20th century. As the new groups were more serious and more interested in social change, the earlier groups must have been less serious and less interested in bringing about social change.
STEP 3: Read every word of each answer option.
If you look back in the passage you can see that answer A cannot be correct as it states that agit prop theater was an evolution of an earlier form of theatre. So agit-prop was not a brand new concept. Answer B makes an unwarranted connection between a political movement and Strasburg's efforts that were artistically, not politically, motivated. Answer C is an excellent candidate for your choice. Since the passage is almost entirely about how the Great Depression turned Broadway into a more serious offering, you're likely to find 1920 more like a musical or lighthearted show, as it was before the Great Depression. This is probably your answer, but you should check the remaining options. Answers D and E cannot be found in the passage.The correct answer is c.
9🇧🇷 Answers:B
STEP 1: Read the question and identify your task.
This is a line ID question. Possible answers contain four points that are explicitly mentioned as differences in the passage. The correct answer is not mentioned or is a point of similarity.
STEP 2: Go back to the passage to find the answer.
In that case, go back to passages A and B and look for any discussion of the differences between these theater groups and others.
STEP 3: Read every word of each answer option.
Passages A and B specifically mention the fact that the theater group used trained actors while the shock troupe did not, so eliminate answer option A. However, answer option B seems immediately promising. Both groups wanted to present pieces that portrayed social claims. This will likely be your correct answer. Answer option C is another difference between the two. The passages mention that the Teatro de Grupo used conventional theater productions, while the Troupe de Choque sometimes performed in the streets. Character names are another difference explicitly mentioned in both passages, so omit answer option D. Finally, answer E may be tempting, but passage B states that the Group Theater only wanted to portray social grievances, while the Shock Troupe wanted to continue and really advocate for social change.The correct answer is b
10🇧🇷 Answers:C
STEP 1: Read the question and identify your task.
This is a question of inference. This question asks you to use the shock troupe information as a basis for making a judgment, particularly about which play the theater is likely to perform the least.
STEP 2: Go back to the passage to find the answer.
You come back to the part of passage B that describes the Shock Troup's theatrical philosophy.
STEP 3: Read every word of each answer option.
Shock Troup was a politically oriented theater company, so the less political answer is the right choice. Answers A, B, D and E are politically or socially flashy games. The play described in Answer C seems to be rooted in the drama of everyday life. There is little social or political content in it.The correct answer is c.
11🇧🇷 Answers:ONE
STEP 1: Read the question and identify your task.
This is a question of inference. The correct answer will correctly identify the author of passage B's attitude towards group theater as this attitude is portrayed in the passages.
STEP 2: Go back to the passage to find the answer.
Passage B mentioned group theater only in passing, noting at the beginning of the passage that group theater only dealt with social grievances, while the shock troupe also wanted to advocate for change. At the end, the author also mentions that the Shock Troupe wanted to train their actors with the group theater method to better connect with the audience.
STEP 3: Read every word of each answer option.
Answer A is immediately promising. Group theater was brought in to train the Shock Troupe actors so that these actors could better engage with the audience. Answer option A is probably correct. Option B discusses which theater company was better known, which is never discussed. Delete answer choice B. Answer choice C discusses comparative idealism, which is not discussed here. Furthermore, the information in the passage seems to support the opposite claim, as Shock Troupe theater could transform society, not just portray society's ills. Similar to answer option B, you have no way of knowing which piece was most popular. Finally, while you know that the Shock Troupe believed Method was an effective way to teach acting, you may not know that they believed it was the most effective way to teach acting.The correct answer must be A.
12🇧🇷 Answers:C
STEP 1: Read the question and identify your task.
This is a line ID question. This question uses a technical term and asks you to choose the answer that best reflects what it means.
STEP 2: Go back to the passage to find the answer.
They look back on the passage and conclude that agit-prop theaters were politically motivated and that these theaters took extreme measures to get their political ideas in front of the masses. So the answer will be based on this kind of description. All answers expand the abbreviated words on either side of the hyphen, requiring you to evaluate both words.
STEP 3: Read every word of each answer option.
For answer A, both words are insufficient. The Agit-prop theater did not deal with agriculture and did not make suggestions through its performances. Answer B does fine with the first part of the term. Agit-prop theater definitely used their performances to stir things up, especially when they performed on picket lines or in front of workers as they commuted to work. But the second term, "suggestion", seems to be an underrated description of what they did. Answer C is much better. As discussed, the first term fits well. The second half also works because they definitely used their performances to spread propaganda or politically charged ideas. This is probably your answer, but you should also evaluate the remaining options. The second term in answer D is problematic. Reproduction is a term that could be used to describe the dissemination of ideas, but it also has a more general meaning related to reproduction. Such ambiguity undermines the chances of this option. Answer E is very wrong, as the word "right" does not fit in a busy stage theater. They weren't interested in being a real theater, especially as they performed their plays outside of the theater.The correct answer is c.
PASSAGE 3
13🇧🇷 Answers:B
STEP 1: Read the question and identify your task.
This is a main ideas question that essentially asks you to find among the answers that statement that summarizes the author's argument or thesis.
STEP 2: Go back to the passage to find the answer.
Consult the summary of your ticket.
STEP 3: Read every word of each answer option.
Answer A focuses on Bo Rothstein's essay, but ignores the rest of the passage that places Rothstein's claims in the context of a broader discussion of the merits of a welfare state. Answer B starts with Rothstein's essay, then adds context, an assessment of happiness as a measure of success, and a perspective on whether the studies help you with the larger argument. While this answer isn't perfect, it's your best candidate so far. You must evaluate the remaining options. Answer C gives a good description of the political part of the passage, but ignores the essays and studies discussed, so this is probably not your choice. Answer D is problematic because the passage does not say that politicians, social scientists, and economists are not supported by data, but that they do not focus on the happiness factor. Most likely they have their own data, but not the Rothstein, Pacek, and Radcliff data that take luck into account. Answer E starts well. The passage notes some flaws in the SWB measurement, but the passage does not claim that improving the data driving this measurement would make it more acceptable to "today's thinkers". You are left with the only answer that works, answer B, no matter how flawed it may be.The correct answer is answer B.
14🇧🇷 Answers:E
STEP 1: Read the question and identify your task.
This is a matter of argument style. You are asked to choose the answer that best describes the structure used in reasoning the passage.
STEP 2: Go back to the passage to find the answer.
To do this, you look back in the passage and read the first or second sentence of each paragraph to get an idea of how the passage moves from one topic to the next. You notice that the first paragraph begins with a very general discussion of the current welfare state conflict. The second paragraph moves on to a discussion of how to measure happiness. The third paragraph deepens the fundamentals of Rothstein's work on happiness. Finally, the concluding paragraph brings the discussion back to the United States. Now evaluate your decisions.
STEP 3: Read every word of each answer option.
Answer A is a very good description of the section and you could stop there, but the description doesn't seem to do paragraph three justice. Paragraph three does not simply "rate" the measure of happiness. Still, you'll keep that option as a possibility, and read through the remaining options to see if there's a better one. In answer B, the word "solution" in the second sentence is inappropriate for this passage. The author of the passage does not offer any solutions, since the existence of the welfare state is not a social problem (like crime) that needs to be "solved". Rather, the passage discusses the merits of the universal welfare state and how to assess its success or failure. Answer C is incorrect because your organization does not match the passage. The third paragraph actually provides more information based on Rothstein's work on happiness. Do not "discredit" this work. Answer D also states that a solution to the problem is suggested and no solution is offered, so this cannot be your choice. Finally, Answer E offers some competition with Answer A. In this case, the answer seems to be more significant, especially with regard to paragraph three, which says it is about the validity of measuring happiness, something that Answer A ignores . Therefore, between answers A and E, answer E is the best descriptionthe correct option is answer E.
fifteen🇧🇷 Answers:C
STEP 1: Read the question and identify your task.
This is a line ID question. This question asks you to look at a specific phrase or comment within the passage and explain the author's purpose in using it.
STEP 2: Go back to the passage to find the answer.
Look again at the identified line and read the sentence and possibly its context to understand the reasons for the author's inclusion. Your options probably depend on how this comment fits in this context.
STEP 3: Read every word of each answer option.
Answer A repeats the author's comment in more explicit terms, but does not explain why that comment appears at that point in the passage. Answer B may be true, but that refers to the next paragraph about Pacek and Radcliff, not the paragraph where the comment appears. Answer C is very promising. The sentence before the biased comment discusses how Rothstein focused his study on northern European countries, which they say have long had welfare states, and you already found out that he is a Swede, i.e. from a northern country from Europe. This explanation fits your context and explains the comment very well. You should check the remaining options in case there is a better answer. Answer D may be true, but this explanation is related to a bias very different from the one the author is referring to in this context. Finally, Answer E reflects an entirely unrelated fact, and in turn relates to a different kind of bias that does not fit the context in which the comment appears in the passage.The correct option is answer C.
16🇧🇷 Answers:B
STEP 1: Read the question and identify your task.
This is a weak question. This question is very similar to a question you might find in the Arguments section of the LSAT. It is asked which answer would most undermine or weaken a given Rothstein argument.
STEP 2: Go back to the passage to find the answer.
Rothstein believes that the SWB measurement is a better measure of society because it is a "generalized" arbiter rather than the elitist one that experts make it out to be.
STEP 3: Read every word of each answer option.
Answer A argues that experts make judgments based on their own biases that are not significantly different from the measure of happiness. This weakens Rothstein's testimony somewhat, but it's not a very strong attempt. You should expect that the other options might require more effort. Answer B actually argues that experts are superior because they filter out the cultural factors that cloud the ability to focus on the policy issue at hand. This is stronger than answer A as it suggests the experts are better, but you should continue to explore the remaining options. Answer C reiterates Rothstein's argument to some extent and does nothing to weaken it. Answer D may be correct, but it also supports Rothstein's argument that his measurement is better able to judge the success of the welfare state. Indeed, if experts are not affected by economic changes, they may remain an elite arbiter and continue to misjudge political progress. Finally, answer E supports Rothstein's argument rather than undermines it. If the SWB measurement can actually make such predictions more accurately than the models created by the experts, then it is also a better arbiter.The correct answer is answer B.
17🇧🇷 Answers:C
STEP 1: Read the question and identify your task.
This is an issue of empowerment. This question is very similar to the one you can find in the LSAT Arguments section.
STEP 2: Go back to the passage to find the answer.
You must find an explanation that supports the arguments presented in the first paragraph, made by those who support universal welfare programs in the United States.
STEP 3: Read every word of each answer option.
Answer A actually supports those in the opposing camp who believe that welfare harms the nation, so this cannot be your answer. Answer B also supports the rejection of welfare programs, saying that welfare grantees do not use the funds the federal government gives them. Answer C states that new social assistance programs increase political participation. This strongly supports the argument made in the passage that welfare programs are important for maintaining a functioning democracy. So this is probably the correct answer. You should check the remaining answers to be sure. Answer D may be correct, but whether or not recipients like the government seems to be irrelevant to either supporters or critics of welfare programs. Answer E would certainly be helpful to proponents of welfare programs, but it argues with an argument made by Rothstein later in the passage. The question requires you to base the answer on paragraph one only. Since this doesn't really address the specific arguments in the first paragraph, it's not as strong as answer C.The correct alternative is answer C.
18🇧🇷 Answers:D
STEP 1: Read the question and identify your task.
This is a matter of tone. This question requires you to use your judgment as to the author's tone or attitude toward Rothstein's approach.
STEP 2: Go back to the passage to find the answer.
Certain words in the passage will give you clues about this scenario. For example, the comment about Rothstein's bias in the second paragraph indicates that the author may have some doubts about Rothstein's conclusions. Think of the phrases "He goes so far as to say" and "SWB is the most controversial basis of these studies". This and other phrases, however lukewarm, testify to the author's doubts about Rothstein. With that in mind, weigh your options.
STEP 3: Read every word of each answer option.
These observations about the author's language make it impossible to believe that "cautious neutrality" could be correct, so answer A cannot be your choice. The author does not use strong language to consider answer B. Regarding answer C, the author acknowledges that Rothstein's work could be useful for welfare state discourse, but "moderate endorsement" would mean that the author recommends it , and this is not the case. Answer D is very likely that your choice of author shows some skepticism about Rothstein's work. Finally, in answer E, nothing in the language shows a grudging acceptance of Rothstein's ideas. This would require the author to admit error in judgment and words of contrition if Rothstein's ideas were to prevail. You don't see any of that in the passage. Therefore,the correct option is answer D.
19🇧🇷 Answers:E
STEP 1: Read the question and identify your task.
This is an information retrieval question. You have to find the one statement among the responses that Rothstein would disagree with.
STEP 2: Go back to the passage to find the answer.
You refer to the passage and read the sections that reflect Rothstein's views. Paragraphs two, three and four cover all of his views. The only answer that does not align with these views of social happiness will be the correct answer.
STEP 3: Read every word of each answer option.
In paragraph three, you learn that Rothstein admits that cultural factors such as the A answer can affect happiness. Despite dismissing such concerns, he might agree that the capabilities listed in Answer A would obscure the relationship between SWB and the establishment of a universal welfare state. For answer B, Rothstein would certainly agree that a child protection program would increase national happiness. Answer C refers to Rothstein's bias, which, as you've already seen, favors northern European states, so he would probably agree with that statement. See answer D below. As the passage states, Rothstein believes that people need lasting confidence in their institutions to find happiness, so he would definitely agree that such an enduring program would increase their SWB. Finally, in Answer E, Rothstein would never agree that embezzlement proves the failure of a welfare state in general. He would only say that the special welfare state has missed one of the key elements needed to bring happiness to its beneficiaries.The correct option is answer E..
PASSAGE 4
20🇧🇷 Answers:B
STEP 1: Read the question and identify your task.
This is a key question that asks you to find among the answers the single statement that sums up the passage.
STEP 2: Go back to the passage to find the answer.
Consult the summary of your ticket.
STEP 3: Read every word of each answer option.
Read the answers, paying close attention to the language. Answer A assigns a reason to the companies not supported by the ticket. The passage says nothing about a "stigma" attached to monetary donations or that they are seen as a cheap marketing tactic. In fact, the passage states that they are not different from each other. Answer B is promising. The passage argues that whether companies incorporate philanthropy into their operations or just write a check, the effect is the same for all parties, and the passage examines the reasons for corporate philanthropy without reaching any specific conclusions. This is probably your answer, but you'll need to check the remaining options to be sure. Answer C is too strong an opinion to summarize the main idea of this passage, which preserves a certain level of objectivity. The passage does not make as strong a recommendation to do either or both. For Answer D, blackmail is an idea explored in the passage, but it is not the whole idea of the passage. This answer makes it overly central, and the passage definitely doesn't make a direct recommendation that companies should just make a charitable donation. Finally, answer E isn't bad, but it's pretty general in its discussion. There is no mention of integrating philanthropy into operations and sales rather than simply writing a check, which is central to the passage's argument. Therefore,the correct option is answer B.
21🇧🇷 Answers:C
STEP 1: Read the question and identify your task.
This is a line ID question. You need to look at the lines given in this question and assess the author's position or tone in relation to the model of philanthropy being discussed.
STEP 2: Go back to the passage to find the answer.
In this case, the speeches discuss the philanthropic model, in which companies integrate philanthropy into their "supply, manufacturing and distribution activities so that it becomes an integral part of their corporate and product identity". As you read the lines, you can tell that the author believes this model is "no different than just giving."
STEP 3: Read every word of each answer option.
With that in mind, read your answers. You learn later in the passage that Answer A is not true, but you don't need the rest of the passage to know that nothing on the indicated lines indicates such a positive outlook on the philanthropic way. Answer B also discusses elements later in the passage that contradict what they say. However, the speeches considered do not give any indication that the author sees this model as empowering for clients. Answer C is probably your answer. The author definitely doesn't seem to believe that the new model is really anything new, nor does he offer any insight into why companies are engaging in philanthropy. You should check the remaining options to be sure. Answer D ventures into areas explored later in this section that are outside the scope of these questions. And as for answer E, the author doesn't seem very happy with the new way of being philanthropic, as the language indicates that the author sees no difference between that and writing a check. Because of this,the correct option is answer C.
22🇧🇷 Answers:B
STEP 1: Read the question and identify your task.
This is a question of inference. This question is one of the rare ones that asks you to imagine that the passage continues in another paragraph and you have to choose the answer that best fills that role. Essentially, it asks you to deduce what comes after the last paragraph.
STEP 2: Go back to the passage to find the answer.
Return to the passage and examine the last paragraph. The last sentences give a clue where the author might get his thoughts. The last paragraph discusses the justifications for corporate philanthropy, and the last sentences compare a company to an individual in society and how this view affects the analysis.
STEP 3: Read every word of each answer option.
Answer A might be a good paragraph to include after the second paragraph that discusses integrating philanthropy into supply lines, but not at the end of the section that moved on to other considerations. Answer B could very well be the next topic, moving from corporate philanthropy to whether the dollars they donate to charity are going to any good and which method works best. This isn't a perfect option, but it might work. You'll have to read the other options to see if there's a better one. Answer C starts off well, but the second sentence veers off and makes the odd logical leap to government intervention. Nothing in the passage mentions government involvement, so this is probably not the right choice. If the section ended with paragraph three, answer D might be correct, but the fourth paragraph makes choosing it impossible as you move on to address the broader question of whether corporations should be philanthropic. Finally, answer E is tempting, but it really belongs as another point in the last paragraph, not in a new paragraph that is supposed to lead to a whole new topic, but which logically follows the previous paragraph.Answer B is the correct option.
23🇧🇷 Answers:E
STEP 1: Read the question and identify your task.
This is a line ID question. You are asked to compare two different sections of the section and choose the answer that most accurately describes the relationship between them.
STEP 2: Go back to the passage to find the answer.
You look at the indicated parts of the passage and find that the last passage is almost a repetition of the previous passage, except that the later passage seems more explicit and therefore may be considered a clarification.
STEP 3: Read every word of each answer option.
You can quickly eliminate Answer A as you can see that both sections address the lack of difference between the two options and both address the financial equation involved in integrating philanthropy with operations and sales. It cannot be true that there is no relationship. There is no opinion other than what the author has revealed in both sections, so Answer B can be eliminated. There are also no what-if situations offered in any of the sections. No specific examples are discussed, so answer C cannot be the right choice. Regarding answer D, there is agreement with the position, but there is no disagreement on the details. You've eliminated all other answers, so answer E should be correct, but you should read it to be sure. The sections are equivalent claims that integration and mere donation are not much different, and the last section explains the financial aspect of why they are not different, specifically how they both reduce profitability and shareholder value.The correct option is answer E..
24🇧🇷 Answers:C
STEP 1: Read the question and identify your task.
This is a question of inference. This question asks you to draw a conclusion based on the author's views on corporate philanthropy.
STEP 2: Go back to the passage to find the answer.
This question asks for a conclusion based on your impression of the content of the entire passage and the author's opinion of corporate philanthropy. You must have read the entire passage to answer this question, but if you wish, you can skim the passage for clues that can help you verify the answers.
STEP 3: Read every word of each answer option.
Regarding answer A, informing the customer that the company is getting a tax deduction is not exactly helpful for the customer or the company. The author is unlikely to agree with such an idea. For answer B, such a suggestion could discourage the company from engaging in philanthropic activities, and the author is unlikely to want this, as there is evidence that the author supports corporate philanthropy, although some practices are questionable. Answer C appears to be a valid idea that the author could agree with. The author suggests that the customer will not be informed that the purchase price increase is a result of philanthropic efforts, so the author could help inform the customer about the implications of integrating philanthropy into operations and sales. This is probably your answer, but you should check the remaining options to be sure. Answer D is a good idea, but seems outside the scope of the passage. The author is not concerned with the misuse of funds, but with philanthropic endeavors in general. Answer E is also an interesting possibility, but the author really doesn't care about morals or ethics within companies or that customers know about company ethics.The correct alternative is answer C.
25🇧🇷 Answers:D
STEP 1: Read the question and identify your task.
This is a variation of a main idea question – a main purpose question that asks you to find a more general description of what makes the passage logical.
STEP 2: Go back to the passage to find the answer.
Read the first sentence of each paragraph and possibly the last sentence to understand the author's conclusions. Then evaluate the options and look for the one that seems most accurate.
STEP 3: Read every word of each answer option.
When reading Answer A, certain words become problematic. The author does not question the rationale for integrating philanthropy into operations and distribution. The author just goes into the effect of such behavior and assesses the differences to a simple monetary donation. Also, he ends up asking why they are involved in philanthropy, so this answer contains several clues that make them wrong. You cannot recommend answer B because the author never argues that corporate philanthropy must stop. In fact, the author supports the activity even if it questions certain practical aspects of the behavior. The first part of answer C seems correct, but the second part is not. The author makes no moral case against corporate philanthropy. The author questions the basis of this, but never reaches a definitive conclusion or recommendation against it. Regarding answer D, the author explains a tendency in corporate behavior to integrate philanthropy into operations and sales and compares it with the previous behavior of just donating. Finally, the author asks a question related to the idea of corporate philanthropy in general. This answer is spot on. Answer E suggests that the author discusses unethical behavior, but this is not the case. The author simply says that companies do not give full disclosure.The correct alternative is answer D.
FAQs
Should you take an LSAT diagnostic test? ›
When you start studying for the LSAT, your first step should be to take an LSAT diagnostic test. This test will help you understand which areas you need to improve on when studying and give you a feel for the test.
What is the average score on the LSAT diagnostic test? ›Your LSAT score is the most important factor for admission to law school. The highest LSAT score is a 180. The average LSAT score is about a 152.
Which LSAT should I take for diagnostic? ›You should take your cold LSAT diagnostic under test-day conditions, including the 4th LSAT “experimental section” that you will have to take on your real exam. The new LSAT format only includes 3 scored sections. Therefore, to simulate a full 4-section LSAT preptest, simply take any preptest 89 or below.
What is a good first LSAT diagnostic score? ›According to U.S. News, law school admissions experts recommend striving for at least a 150; however, for a top-ranking law school, you should aim for a 160 or better. For a Top 10 law school, a 170 or more is desired. Of course, this all depends on which schools you are applying to.
Should I take a diagnostic LSAT without studying? ›Taking a “Cold Diagnostic” LSAT preptest without studying
Even though it doesn't tell you much about where you will end up, it is a great way to build quick familiarity with the test format and give you a yardstick to measure your later progress.
You'll look at my LSAT PrepTest Raw Score Conversion Charts and calculations of what it takes to get an LSAT score of 160 or 170. Using that data, you'll find that the December exam consistently has the easiest "curve," and the June exam consistently has the hardest.
What was Obama's LSAT score? ›The easiest to predict, by far, is President Barack Obama's score, mostly because we have some data. Based on admissions records, we can deduce — somewhat reliably — that Barry-O scored between the 94th-98th percentile on his LSAT. Using today's grading system, that'd place him somewhere around a 170.
Is 150 a good diagnostic score? ›3. Is 150 a Good Diagnostic Score? Since the average LSAT score is about 151, a 150 without any studying is a great diagnostic score!
What is a 72% on the LSAT? ›...
Estimated Score Conversions.
Raw score* | Scaled Score | Est. Percentile** |
---|---|---|
74-75 | 180 | 99.97% |
73 | 179 | 99.94% |
72 | 178 | 99.87% |
71 | 176 | 99.64% |
Generally speaking, a score of 160 and above is necessary to guarantee admission into top-100 schools like Tulane, UC Hastings, or Case Western Reserve. For top-50 law schools such as Fordham, University of North Carolina–Chapel Hill, or George Washington, 165 is a good number to reach for.
Is 155 a good LSAT diagnostic score? ›
It is a very good starting score. Although 151 is the average score, most people have some prep before the test and you can expect to go up at least 10 points from your diagnostic if you study correctly.
Is a 154 LSAT diagnostic good? ›The average LSAT score is about 150. To get into a top 14 law school, you need to score above 162, and to get into a top 50 law school, you need 154 or above.
How many people pass the LSAT on the first try? ›Each June (which is considered the start of the testing cycle), about 80% of the test takers are first-times. This percentage drops off in successive tests: 73% of September/October testers are first-timers, 58% of December test takers are LSAT virgins, as are 57% of February test takers.
What LSAT score do I need for a full scholarship? ›You can also improve your LSAT scores to increase your odds of getting a scholarship. Students with an LSAT score close to 170 have a great chance of earning a scholarship for law school. Some other ways to increase the odds of getting a scholarship for law school are listed below.
Can you get into law school with a 155 LSAT? ›It's not going to be sufficient for top programs, but there are plenty of law schools that will be happy to see these scores! The following table represents the school with median LSAT scores in the 155-159 range for admitted full time student populations at these schools as of Fall 2021/Spring 2022.
What was Kim Kardashian LSAT score? ›In a clip from the show, Kim revealed to sisters Khloé and Kourtney that she failed the important test. In order to pass, Kim needed a score of 560. She scored 474.
Do you need high IQ to do well on LSAT? ›Contrary to popular belief, the LSAT does not measure intelligence. Therefore, the test does not render those with higher scores smarter than those with lower scores. The LSAT is one of many factors relied upon by law schools to predict a person's chances of first-year success.
Can you become a lawyer without taking the LSAT? ›The short answer to this question is yes. As of 2022, at least 100 ABA-accredited law schools accept the GRE, or the graduate record examination, in lieu of the LSAT.
What score is a 70% on the LSAT? ›Note that a score in this range places you, on average, in the 98th percentile, meaning that only 2% of all those who take the LSAT score a 170 or above. To get a score in the 160s you should aim for getting 70-85 of the questions correct, or around 70%.
What is the lowest LSAT score a law school will accept? ›Quite frankly, if your LSAT score is below 147, it will be difficult to be admitted to an accredited law school, not impossible but very difficult. Your GPA will have to do some heavy lifting. If your LSAT score is 150 or above, your chances increase if you choose prospective law schools wisely.
How many times can I fail the LSAT? ›
Three times in a single testing year (LSAT testing years run from July 1 through June 30). Five times within the current and five past testing years (the period in which LSAC reports scores to law schools). A total of seven times over a lifetime.
What is the lowest LSAT score ever? ›To make it easier to compare scores earned across different LSAT administrations, your “raw score” is converted to an LSAT scale. This is the score you receive in your score report. The LSAT scale ranges from 120 to 180, with 120 being the lowest possible score and 180 being the highest possible score.
What is the lowest LSAT score for Harvard? ›As you can see from these numbers, an LSAT score of 170 or higher and a GPA above 3.75 will give you a chance of gaining admission to Harvard Law School. If you have a GPA of 3.94 or higher and above a 175, you are pretty much a lock for admission, particularly given the class size of ~560.
Has anyone ever aced the LSAT? ›Getting an LSAT score of 180 or a “perfect score” is extremely rare. According to data published by the Law School Admissions Council (LSAC), from 2006-2009 of all LSATs administered, approximately 144,000 per year, only 0.1% received a 180.
Is 472 a good diagnostic score? ›Your score falls within the range of scores, from 300 to 579, considered Very Poor. A 472 FICO® Score is significantly below the average credit score.
Is 637 a good diagnostic score? ›Your score falls within the range of scores, from 580 to 669, considered Fair. A 637 FICO® Score is below the average credit score. Some lenders see consumers with scores in the Fair range as having unfavorable credit, and may decline their credit applications.
Is 624 a good diagnostic score? ›A FICO® Score of 624 places you within a population of consumers whose credit may be seen as Fair.
How many LSAT questions can I miss and get a 170? ›To achieve a score of 170 requires a test taker to correctly answer 90 out of 101 questions.
Is 148 a bad LSAT score? ›Typical LSAT score ranges include: 120-147 Low. 148-156 Mid. 157-164 High.
Can I get into law school with a 165? ›Top-Tier Law School Admission
In fact, this score range puts you within the median range at some of the top 14 law schools in the country. The following table represents the school with median LSAT scores in the 165-172 range for admitted full time student populations at these schools as of Fall 2021/Spring 2022.
How many questions missed is a 160 on LSAT? ›
Every LSAT throughout the year is different, but on a typical LSAT, you can still get 25 wrong and end up in the 160s— or about 20 wrong and get a 164, a 90th percentile score. Even a perfect score of 180 often allows for a question or two to be missed.
Can you get into Harvard with a 165 LSAT? ›Schools like Harvard and Yale, which are the top two, rarely accept applicants with less than 172 on the LSAT.
Is 146 a bad LSAT score? ›...
Law School Enrollment.
Risk Band | LSAT | |
---|---|---|
Score | Percentile | |
High Risk | 147-149 | 33 - 40.3 |
Very High Risk | 145-146 | 26.1 - 29.5 |
Extreme Risk | 120-144 | ≤ 22.9 |
An LSAT score of 142 places the test-taker in the bottom 18% of all candidates who took the exam. It is a strong indicator that the candidate does not belong in law school at all.
Will a 180 LSAT get you in anywhere? ›Whelp, you can apply to pretty much anywhere with confidence. Scoring in this range makes you extremely competitive for a variety of great law schools.
What is the average LSAT score without studying? ›The LSAT is scored on a 120-180 scale.
From our independent research, we've found that students who take the LSAT without studying achieve scores between 145 and 153.
Scoring a 175 means you missed 5 questions on the test, which can be the equivalent of an entire logic game. Scoring a 170 means you missed 10 or 11 questions, which is nearly half of an entire section. The point of all this is that there is room to make mistakes.
What did Elle Woods get on her LSAT? ›As you probably know, the LSAT is scored from 120 to 180. Elle Woods was able to raise her score from a 143 to a 179 just by diligently preparing.
What's the average LSAT score at Harvard? ›As of the most recent application cycle, Harvard Law's median LSAT score is 174. Assuming the rest of your application is perfectly “average” for Harvard Law, if your LSAT score is below 174, your chances of getting in are below average. If it's above 174, your chances are above average.
Is there math on the LSAT? ›No, the LSAT does not explicitly test math. You may see some questions about percentages in the logical reasoning (arguments) section, and some analytical reasoning questions (i.e., logic games) may resemble math problems, but these questions are designed to test your logical reasoning, not your mathematical knowledge.
What happens if you fail LSAT? ›
It Comes Down to Retaking the Test
Still, there is no other way to increase your score other than re-doing the test. Particularly if illness or anxiety prevented you from performing as well as you might have expected, you have everything to gain from doing it again.
Instead of testing content, the LSAT is an analytical exam that tests critical thinking skills across three subjects: logical reasoning, analytical reasoning, and reading comprehension. What makes the LSAT so challenging is that it approaches these subjects in a manner that is best described as counterintuitive.
Do law schools give full-ride? ›Do law schools offer a full-ride scholarship? Some law schools offer a “full-ride,” although this can mean a lot of different things. In law school, full-ride programs usually cover the full cost of tuition — not living expenses, books, etc. These full-ride scholarships are highly competitive.
When should I start studying for the LSAT? ›Although the June LSAT is ideal, most applicants don't start preparing for the LSAT until the summer before their senior year—after the June LSAT is over. These applicants usually end up taking the test in September or October.
What is the cheapest law school to attend? ›School | In-State Fees | Out-of-State Fees |
---|---|---|
North Carolina Central University, Durham | $11,387 | $23,222 |
CUNY – Queens College | $21,802 | $13,802 |
Southern University Law Center | $10,990 | $18,590 |
University of Alabama School of Law | $15,760 | $28,070 |
The cost of the LSAT is the same for both first-time and repeat test takers. For the August 2022-June 2023 testing years, LSAT registration costs $215.
What is the lowest GPA you can have in law school? ›Law schools generally require that you have specified minimum collegiate GPA and LSAT scores to qualify for admission. Harvard, Yale, and the other top five-ranked law schools require that you have a GPA of at least 3.50 and an LSAT score of 170.
Is 153 a bad LSAT score? ›The LSAT is scored between 120 and 180, with 153 being the average score.
Is a 151 LSAT diagnostic good? ›Since the average LSAT score is about 151, a 150 without any studying is a great diagnostic score!
How important is a diagnostic test? ›Diagnosis can improve the effectiveness of treatments and avoid long-term complications for the infected patient. Undiagnosed patients can unknowingly transmit the disease to others. Early diagnosis can help to prevent or stop an outbreak.
What is a 75% on the LSAT? ›
...
Estimated Score Conversions.
Raw score* | Scaled Score | Est. Percentile** |
---|---|---|
74-75 | 180 | 99.97% |
73 | 179 | 99.94% |
72 | 178 | 99.87% |
71 | 176 | 99.64% |
The average LSAT score is around 150. The LSAT has a margin of error, but 145 is considered a symbolic line by legal education experts and school administrators.
Can I get into law school with a 155? ›A score of 155 on the LSAT is a classic 'in-between' score. While the score is not too low, it will also not put you in the cream of LSAT test takers. An LSAT score of 155 can at best be classified as an average score which will put you in the hunt for a decent law school. The LSAT is scored on a scale of 120-180.
How do I prepare for a diagnostic test? ›Avoid exercise such as jogging or weightlifting for 24 to 48 hours prior to your PET/CT scan. Avoid caffeine and tobacco for 24 hours prior to your exam. Continue to take any medication prescribed by your physician.
Does diagnostic test affect your grades? ›Diagnostic assessments are “low-stake” assessments because they are non-graded and do not determine whether the student moves to the next educational level. Rather, they show a student's strengths and weaknesses regarding a specific field of study.
What is checked during a diagnostic test? ›Diagnostic tests will report back codes for almost all of the major components of your vehicle like the engine, brakes, transmission, and exhaust system as well as the fuel injection system, coolant and air flow, the various sensors throughout your vehicle, and more.